NPTE Practice Test 1

Réussis tes devoirs et examens dès maintenant avec Quizwiz!

Correct Answer: 2 1. Stretching of hip flexors is important for patients who exhibit limited hip extension. This patient has full passive range of motion. 2. Strength training has been shown to improve gait and muscle performance in patients who have cerebral palsy. The physical therapist should seek to create a balance of muscle activity across a joint. In this case, addressing quadriceps and gluteal muscles will be beneficial for improving knee and hip extension by counteracting the forces potentiating flexion. 3. Stretching exercises should be followed by active movement to maximize gains; therefore, stretching of the hamstrings and gastrocnemius muscles without strengthening muscle groups to promote improved extension would not optimize long-term gains. 4. Although strengthening is beneficial, strengthening of the hamstrings would not counteract the forces contributing to hip flexion.

A 12-year-old patient who has spastic diplegic cerebral palsy has full passive range of motion of the lower extremities, but demonstrates crouching with hip and knee flexion angles of 20° each in standing position. Which of the following interventions is BEST to achieve sustained improvements in lower extremity alignment during walking? 1. Stretching of the iliopsoas 2. Strengthening of the quadriceps and gluteals 3. Stretching of the hamstrings and gastrocnemius 4. Strengthening of the hamstrings and gastrocnemius

Correct Answer: 3 1. Palpation is not likely to reproduce symptoms for a capsular dysfunction. Palpation is more likely to identify a muscle or soft tissue condition or bursitis. (p. 901) 2. The case presentation indicates hip dysfunction, specifically slipped capital femoral epiphysis. Special tests of the knee are not likely to provoke symptoms. (p. 1002) 3. The stem describes a case of suspected slipped capital femoral epiphysis. Signs and symptoms are typically found in adolescent patients (10-16 years old) and include leg shortness, knee pain, and pain when the hip is medially (internally) rotated. Groin pain will be triggered with the anterior impingement test (hips flexed to 80° to 90° and medially [internally] rotated with adduction) if slipped capital femoral epiphysis exists. (pp. 1580-1581) 4. The stem describes a case of suspected slipped capital femoral epiphysis. The flexion, abduction, external rotation test (FABER) described in this option is used to indicate lumbar, sacroiliac joint, or posterior hip dysfunction associated with the hip capsule. Although it is a femoroacetabular impingement test, it is a better indicator of posterior hip dysfunction than anterior hip dysfunction. (pp. 1549-1550)

A 13-year-old patient reports moderate knee pain persisting more than 3 weeks, with no trauma noted. The patient exhibits an out-toeing gait pattern, leg length discrepancy, and restriction in medial (internal) rotation of the involved leg. Which of the following test findings would MOST likely be present? 1. Pain with palpation of the trochanteric region 2. Pain and instability during the application of valgus stress to the knee in full extension 3. Pain in the groin region with hips flexed 80° to 90° and then medially (internally) rotated with adduction 4. Pain in the gluteal region with combined movements of hip flexion to 45° to 60°, abduction, and lateral (external) rotation

Correct Answer: 3 1. Symptoms of ulcerative colitis include mild to moderate anorexia and weight loss, abdominal pain, and skin rashes (p. 884). 2. Celiac disease is an intolerance for gluten. Initially, the condition is characterized by weight loss, abdominal bloating, weakness, and diarrhea. (p. 882) 3. The signs exhibited by the patient described in the stem are indicative of anorexia nervosa, which can occur frequently in this population (p. 93). 4. Symptoms of irritable bowel syndrome include abdominal pain and bloating. Although there may be some anorexia, the evidence of long-term nutritional deficits is not as evident as in anorexia nervosa. (p. 890)

A 14-year-old high school wrestler is participating in a conditioning program. Proximal muscle weakness, swelling of the hands and feet, and clubbing of the fingers are evident. The physical therapist should be MOST concerned with which of the following conditions? 1. Ulcerative colitis 2. Celiac disease 3. Anorexia nervosa 4. Irritable bowel syndrome

Correct Answer: 3 1. An annular disc tear is more common in an older population and more common in the lumbar region. 2. A compression fracture is more likely in older patients who have osteoporosis. 3. The stem describes the presentation of Scheuermann disease, which typically affects the T7-T10 region. 4. Spondylolisthesis may be present in the lumbar spine, but it is rare in the thoracic spine.

A 16-year-old patient reports the insidious onset of middle to lower thoracic pain. The pain is worse with prolonged standing or sitting. The patient's posture is characterized by excessive thoracic kyphosis and lumbar lordosis. Active rotation in sitting position is painful. Which of the following conditions is the MOST likely cause of the patient's pain? 1. Annular disc tear 2. Compression fracture 3. Scheuermann disease 4. Spondylolisthesis

Correct Answer: 3 1. The stem describes a patient who recently developed psoriasis (pp. 449-450). Bluish discoloration of the digits with cold exposure is associated with Raynaud disease (p. 255). 2. The stem describes a patient who recently developed psoriasis (pp. 449-450). The dermal effects associated with systemic lupus erythematosus include a major skin (p. 445). 3. The stem describes a patient who recently developed psoriasis, a systemic disease hallmarked by silver scaled papules and plaques in the scalp, elbows, knees, back, and buttocks. It is a systemic disease that can result in erosive arthritis, particularly in the DIP joints of the hands. (pp. 449-450) 4. The stem describes a patient who recently developed psoriasis, a systemic disease hallmarked by silver scaled papules and plaques in the scalp, elbows, knees, back, and buttocks. It is a systemic disease that can result in erosive arthritis, particularly in the DIP joints of the hands. Hip joints typically are not involved in psoriasis. (pp. 449-450)

A 43-year-old male patient reports the recent appearance of silver and scaly-appearing plaques on the scalp, elbows, and knees. If left unaddressed, which of the following complications is MOST likely to develop? 1. Bluish digits with cold exposure 2. Dermal reaction to sun exposure 3. Erosive arthritis in the DIP joints of the hands 4. Erosive arthritis in the hip joints

Correct Answer: 4 1. Graves disease is a hyperthyroid condition causing proximal muscle weakness and a generalized elevation of body metabolism. It has been linked to periarthritis and calcific tendinitis in the shoulder, and sometimes the wrist, but not the knee. It does not cause local joint inflammation. (Goodman, Differential Diagnosis, pp. 395, 417) 2. Complications of diabetes include atherosclerosis, delayed wound healing, and neuropathy, but not a sudden onset of monoarticular arthritis (Goodman, Differential Diagnosis, p. 403). 3. Clinical signs of sepsis include fever, tachycardia, and tachypnea (Goodman, Pathology). 4. The typical symptom of gout is acute monoarticular arthritis with redness and swelling. The knee is one of the commonly affected joints. The peak incidence is in the 40-50-year age group, and it predominantly affects men. (Goodman, Differential Diagnosis, p. 413)

A 45-year-old male patient reports knee pain with a sudden onset 2 days ago. The patient denies trauma or injury to the knee. The patient's knee is warm, red, and swollen. The patient is afebrile and has knee range of motion of 20° to 60°. Which of the following pathologies is the MOST likely cause of the knee pain? 1. Graves disease 2. Diabetes mellitus 3. Sepsis 4. Gout

Correct Answer: 4 1. Hip dysplasia occurs when the femoral head is subluxed or dislocated from the acetabulum. It is typically diagnosed in infancy and occurs more often in females. This patient is the wrong sex and is beyond the normal age range for this condition to cause an acute onset of pain. (Goodman, pp. 1178, 1181) 2. Femoroacetabular impingement (FAI) typically occurs in young to middle-aged active adults. The pain is typically activity related and intermittent at first. (Cheatham, pp. 114-115) 3. Slipped capital femoral epiphysis (SCFE) is an acute or chronic fracture of the proximal femoral physis. It typically occurs in males in early adolescence. It may occur acutely. This patient is beyond the normal age range for this condition to cause an acute onset of pain. (Cheatham, p. 230) 4. Osteonecrosis typically has a gradual onset of pain, occurs between the 3rd and 5th decades, and is more likely to occur in males than in females. History of corticosteroid use is a risk factor for osteonecrosis. (Goodman, pp. 1364-1365) Symptoms may be nonspecific and clinical findings vary. Pain is likely to be exacerbated with weight-bearing activities, however is often present even at rest. (Dutton, p. 252)

A 45-year-old male patient who has a history of corticosteroid use reports a recent onset of constant hip pain unrelated to movement. The patient MOST likely has which of the following conditions? 1. Hip dysplasia 2. Femoroacetabular impingement 3. Slipped capital femoral epiphysis 4. Osteonecrosis of the femoral head

Correct Answer: 3 1. Osteoarthritis often presents in the hands, but it is not often bilateral at the onset, nor present in multiple joints (Goodman, Pathology, pp. 1305-1306). Rheumatoid arthritis often presents with general fatigue, weakness, and bilateral symptomatic joints, most often presenting first in the hands and wrists (Goodman, Differential Diagnosis, pp. 448-449; Goodman, Pathology, pp. 1318-1321). 2. Reiter syndrome is a systemic disease that can cause pain in multiple joints; however, it is usually asymmetric, occurs after an infection, and presents over several weeks (Goodman, Differential Diagnosis, pp. 448-449). This is not consistent with the patient presentation in the scenario. Rheumatoid arthritis often presents with general fatigue, weakness, and bilateral symptomatic joints, most often presenting first in the hands and wrists (Goodman, Differential Diagnosis, pp. 438-441; Goodman, Pathology, pp. 1318-1321). 3. Rheumatoid arthritis often presents with general fatigue, weakness, and bilateral symptomatic joints, most often presenting first in the hands and wrists (Goodman, Differential Diagnosis, pp. 438-441; Goodman, Pathology, pp. 1318-1321). 4. While bilateral carpal tunnel syndrome is a possibility and would present with the wrist pain and perhaps tenderness of the carpal area, the metacarpophalangeal (MCP) joints would not be swollen or tender (Goodman, Pathology, p. 1670). Rheumatoid arthritis often presents with general fatigue, weakness, and bilateral symptomatic joints, most often presenting first in the hands and wrists (Goodman, Differential Diagnosis, pp. 438-441; Goodman, Pathology, pp. 1318-1321).

A 45-year-old patient reports general weakness and fatigue that developed over the past few months, along with increased pain bilaterally in the wrists and hands. The MCP and carpal joints are tender to touch, and the MCP joints appear slightly swollen. The patient MOST likely has which of the following conditions? 1. Osteoarthritis 2. Reiter syndrome 3. Rheumatoid arthritis 4. Carpal tunnel syndrome

Correct Answer: 2 1. Pain symptoms associated with bicipital tendinitis are usually in the anterior shoulder in the area of the biceps tendon. Pain occurs with full extension of the shoulder and elbow. (p. 650) 2. Factors associated with development of adhesive capsulitis include age older than 40 years, history of diabetes, and female gender. Stage II adhesive capsulitis, which is present for 3-9 months, is associated with decreased lateral (external) rotation and abduction and with pain that disrupts sleep. (pp. 650, 665, 667) 3. Glenohumeral instability is associated with "popping out" or "slipping" during overhead activities and is more common in athletic adolescents or young adults (p. 670). 4. Acromioclavicular joint sprain usually occurs as a result of a trauma, and pain associated with this condition is more localized to the joint itself (p. 650).

A 47-year-old female patient who has diabetes reports a 7-month history of right shoulder and upper arm pain of gradual onset. The patient reports waking up at night due to the pain and has decreased active range of motion in all ranges but especially in lateral (external) rotation and abduction. Which of the following conditions is MOST likely present? 1. Bicipital tendinitis 2. Adhesive capsulitis 3. Glenohumeral instability 4. Acromioclavicular joint sprain

Correct Answer: 2 1. In the presence of femoral retroversion that is asymmetrical, it may appear that medial (internal) rotation is limited, but that would not explain the limitation in abduction and may not explain the decreased stance time on the left (Tecklin). 2. Legg-Calvé-Perthes disease is associated with limited hip abduction and medial (internal) rotation and higher prevalence in boys than in girls. An age of 6 years is the most common age for the presentation of this condition. (Palisano) 3. Slipped capital femoral epiphysis includes groin pain, which is not reported by the patient described in the stem (Palisano). 4. Weakness of the gluteus medius would limit active, but not passive range, of motion (Palisano).

A 6-year-old boy has decreased stance time on the left lower extremity. There is no history of trauma and no pain in the extremity. Passive and active ranges of motion of both hip medial (internal) rotation and hip abduction are limited by 50%. Which of the following conditions is MOST likely present? 1. Femoral retroversion 2. Legg-Calvé-Perthes disease 3. Slipped capital femoral epiphysis 4. Gluteus medius muscle weakness

Correct Answer: 4 1. Hypotension is abnormally low arterial blood pressure or a fall in arterial blood pressure with an increase in exertion or heart rate. It results from many causes, including dehydration and lack of blood ejected from the left ventricle. (p. 66) 2. Hypertension, increased systolic blood pressure, results in increased myocardial work (p. 66). 3. Left ventricular failure results in backup of blood into the pulmonary system and decreased cardiac output. Clinical manifestations include dry cough or wheezing, tachycardia, light-headedness, pallor, or cyanosis. (p. 143t) 4. Right ventricular failure results in backup of blood into the systemic venous circulation, manifested by edema systemically, including jugular venous distention, ascites, and bilateral pedal edema (p. 143t).

A 62-year-old patient has ascites and bilateral pedal edema. The patient's pulse rhythm is regular. The patient's history is negative for any liver, kidney, or metabolic disease. Which of the following conditions is MOST likely present? 1. Hypotension 2. Hypertension 3. Left ventricular failure 4. Right ventricular failure

Correct Answer: 2 1. Typical symptoms of a lower extremity deep vein thrombosis are a dull ache or pain, swelling, warmth, and tenderness in the posterior lower leg (Dutton, pp. 268-269). 2. Pulmonary embolism is a common postsurgical condition with symptoms that include dyspnea, chest pain, cough, apprehension, and tachycardia. Individuals undergoing a total hip arthroplasty are at high risk of developing a pulmonary embolus. (Dutton, p. 56; Goodman, p. 850) 3. Recent surgery (especially total hip arthroplasty), obesity, and older age are risk factors for a deep vein thrombosis or pulmonary embolism. It would be prudent to consider a venous thromboembolus in this scenario because the patient has all of the risk factors. (Goodman, p. 648) 4. A local infection would present with swelling, warmth, and redness, not dyspnea, tachycardia, and light-headedness (Goodman, p. 1240).

A 74-year-old patient who is obese underwent a total hip arthroplasty 5 days ago. Currently, the patient exhibits dyspnea, tachycardia, and light-headedness. When a physical therapist squeezes the patient's calf muscle while positioning the ankle in dorsiflexion, no pain is reported and no swelling or warmth is seen. These findings are MOST indicative of which of the following conditions? 1. Deep vein thrombosis 2. Pulmonary embolism 3. Poor conditioning 4. Infection

Correct Answer: 1 1. Chorea is a type of dyskinesia that is often observed as a side effect of antiparkinsonian medication and that typically emerges with prolonged use of such medications. Chorea is characterized by involuntary, rapid, irregular, and jerky movements. 2. Dysmetria is defined as problems in judging the distance or range of movement and is associated with cerebellar dysfunction. 3. Dystonia is characterized by co-contraction of the agonist and antagonist muscles and is associated with basal ganglia dysfunction. Segmental dystonia involves two or more adjoining body regions dominated by sustained muscle contractions, causing twisting and repetitive movements and abnormal postures. 4. The presence of abnormal synergies is closely associated with lesions in the corticospinal centers that result in the emergence of mass movement patterns. Abnormal synergies are associated with motor cortex deficits.

A 77-year-old female patient who has a long history of taking antiparkinsonian medications exhibits random, rapid, and jerky movements. Which of the following terms BEST describes these movements? 1. Chorea 2. Dysmetria 3. Segmental dystonia 4. Abnormal synergies

Correct Answer: 4 1. Sustained limb posturing is characteristic of dystonia, not athetosis. 2. Tremor is characteristic of cerebellar involvement or Parkinson disease, not athetosis. 3. Rapid, jerky motions are characteristic of chorea, not athetosis. 4. Athetoid cerebral palsy is characterized by slow, involuntary, writhing, twisting, "wormlike" movements. Some muscles demonstrate tone that is too high, and others demonstrate tone that is too low.

A child who has athetoid cerebral palsy is MOST likely to exhibit which of the following characteristics? 1. Sustained limb posturing 2. Low frequency tremor 3. Rapid, jerky motions 4. Mixed muscle tone

Correct Answer: 3 1. Transient synovitis is most likely to occur in children younger than the patient described in the stem. Transient synovitis is associated with an active antalgic gait and with pain that is aggravated by medial (internal) rotation but also by abduction, unlike the pattern found with anterior labral tears. (p. 1580) 2. Trochanteric bursitis is more likely to be associated with lateral hip/thigh pain aggravated by lying on the involved side. This condition is more likely to occur in patients age 40-60 years, not in the demographic group of the patient described in the stem. Pain often is enhanced with passive hip adduction and resisted lateral (external) rotation, abduction, and extension. (pp. 945-947) 3. The clinical presentation of anterior acetabular labral tears most often includes pain on passive adduction, flexion, and medial (internal) rotation. The description of mechanism of injury and clinical presentation in the stem are most typical of this injury. (pp. 936-938) 4. Femoral head stress fractures present with a pain pattern similar to the pattern presented in the stem, but pain is increased with weight-bearing. Examination results are often negative except for an empty end-feel with hip rotation and a noncapsular pattern. (p. 952)

A collegiate athlete reports right anterior groin pain first encountered after a rotational injury. The patient is also experiencing painful clicking. Hip range of motion is normal, but pain is provoked with combined end-range hip flexion, adduction, and medial (internal) rotation. Radiographs of the hip and pelvis are normal. Which of the following diagnoses is MOST likely? 1. Transient synovitis 2. Trochanteric bursitis 3. Anterior acetabular labral tear 4. Femoral head stress fracture

Correct Answer: 3 1. A patient who has quadriceps weakness leans away from the involved side to avoid potential buckling of the involved knee. The patient may also demonstrate circumduction of the involved leg. 2. A patient who has a weak gluteus medius would have excessive lateral trunk lean during gait due to the decreased ability for the gluteus medius to stabilize the pelvis while in stance phase on the contralateral leg. 3. A patient who has weakness of the tibialis anterior muscle would have a steppage gait due to the decrease in ankle dorsiflexion. The patient would exhibit high steppage to allow clearing of the ankle and foot during gait and to avoid tripping over the foot. 4. Patients who have weakness in the gluteus maximus exhibit a posterior thrust of the trunk at heel strike (initial contact) to help obtain hip extension of the stance leg. They also exhibit an anterior tilt of the pelvis to help maintain posture.

A patient exhibits a steppage gait pattern. Which of the following muscles is MOST likely weak? 1. Quadriceps 2. Gluteus medius 3. Tibialis anterior 4. Gluteus maximus

Correct Answer: 1 1. Midline retraining in both sitting and standing positions with the use of visual cues or a visual aide is an appropriate intervention for a patient who exhibits pusher syndrome (ipsilateral pushing). 2. Weight-bearing should be encouraged on the involved lower extremity. The left lower extremity is involved, since the cerebrovascular accident affected the right side of the brain. 3. Fixed resistance on the uninvolved side should be used. The right lower extremity is uninvolved, since the cerebrovascular accident affected the right side of the brain. 4. Lowering the height of the assistive device will encourage weight-bearing on the uninvolved side.

A patient exhibits pusher syndrome (ipsilateral pushing) following a right cerebrovascular accident. Which of the following interventions is MOST appropriate? 1. Midline retraining in a sitting position 2. Weight-bearing on the right lower extremity 3. Providing fixed resistance on the left side 4. Raising the height of the assistive device

Correct Answer: 2 1. Culturally and linguistically appropriate services require professional translators with knowledge of the patient's language and knowledge of medical terms. The patient's grandson should not be assumed to be an appropriate translator due to the sensitive topics to be covered. 2. Culturally and linguistically appropriate services require professional translators with knowledge of the patient's language and knowledge of medical terms. 3. Culturally and linguistically appropriate services require professional translators with knowledge of the patient's language and knowledge of medical terms. Although the person is a member of the hospital staff, the person may not have adequate knowledge of the medical terms involved in the patient's care. 4. Use of gestures, pictures, and simple terms by the physical therapist may help avoid confidentiality issues but does not provide satisfactory communication between the therapist and the patient.

A non-English-speaking patient is accompanied to physical therapy by her young English-speaking grandson. The patient does not understand or speak enough English to fully participate in an initial examination. To provide the MOST appropriate services, the therapist should take which of the following actions? 1. Ask the grandson to translate and proceed with the examination. 2. Use a professional translator and proceed with the examination. 3. Ask a same-language-speaking member of the hospital's staff to translate and proceed with the examination. 4. Use gestures, pictures, and simple terms in order to proceed with the examination.

Correct Answer: 3 1. Since there is no remaining devitalized tissue, an enzymatic agent, which is used for debridement, would no longer be needed. More appropriate and less expensive dressings should be used instead. (p. 297) 2. Alginate dressings are highly absorbent, so they are more appropriate for a wound that is producing moderate to high amounts of exudate (pp. 309-310). 3. Hydrocolloids are indicated for wounds that have low to moderate amounts of drainage and that need protection from bacteria or other contaminants (pp. 312, 314). 4. Nonwoven gauze dressing would not be an appropriate choice because it would possibly adhere to the wound surface and disrupt healing (pp. 316-317).

A partial-thickness wound that has been treated for 10 days is currently debrided of all devitalized tissue, but granulation tissue is still not apparent. The wound is draining a minimal amount of serous fluid. Which of the following interventions would be MOST appropriate? 1. Enzymatic agent 2. Calcium alginate dressing 3. Hydrocolloid dressing 4. Nonwoven gauze dressing

Correct Answer: 3 1. Also known as thyrotoxicosis, thyroid hypersecretion is associated with an enlarged thyroid gland, also known as a goiter. Patients exhibit hypermetabolism and sympathetic overactivity. Patients report fatigue, tremor, heat intolerance, increased sweating with warm moist skin, weight loss, palpitation with tachycardia, diarrhea, and muscle weakness and atrophy. They do not display an abnormal vital sign response to exercise, nor do they have acetone breath. (pp. 484-486) 2. Giantism and acromegaly can result from excessive secretion of growth hormone. This can result from a pituitary tumor or from a hypothalamic abnormality that leads to increased growth hormone release (pp. 479-480). Pituitary hyperactivity does not result in an abnormal vital sign response to exercise and does not cause the acetone breath associated with ketoacidosis. 3. Patients who have diabetes potentially have a lack of insulin secretion or effectiveness, leading to disruption of glucose metabolism. Results of acute metabolic changes related to glucose metabolism include hyperglycemia (high blood glucose), electrolyte disturbances that are manifested by acidosis that triggers an increased respiration rate, irregular heart rate, and increased fatty acid metabolism resulting in acetone breath. (p. 522) 4. People who have hyposecretion of adrenal hormones have Addison disease. Addison disease is characterized by the inability to withstand food deprivation, hyperpigmentation, dehydration, and postural hypotension (p. 498). A patient who has adrenal insufficiency should not exhibit any of the signs or symptoms described in the stem.

A patient displays an irregular heart rhythm, increased respiratory rate, and acetone-like breath odor after performing 15 minutes of intense exercise. Which of the following conditions is MOST likely present? 1. Thyroid hypersecretion 2. Pituitary hypersecretion 3. Pancreatic hyposecretion 4. Adrenal hyposecretion

Correct Answer: 4 1. Although pain and swelling occur about the medial ankle in a patient who has an eversion ankle sprain, the patient would demonstrate pain with eversion stress. Also, the patient would relate this pain to a specific traumatic event (Dutton, pp. 1151-1154). The patient would also not necessarily have rearfoot pronation and forefoot abduction. 2. In retrocalcaneal bursitis, the pain is behind the ankle posterior to the talus, but there is no pain with resistive testing (Dutton, pp. 1165-1166). 3. Although pain occurs in the medial aspect of the ankle with tarsal tunnel syndrome, this condition is associated with paresthesias, which are not reported by the patient in the stem (Dutton, p. 1137). 4. The main function of the tibialis posterior is to plantar flex and invert the foot as well as support the medial arch. The tendon courses under the medial malleolus, causing pain and inflammation in this area when the tendon is dysfunctional. With a heel raise, the tendon becomes stressed due to its actions of plantar flexion and inversion. Finally, a finding of the "too many toes sign," which is the hallmark sign of this diagnosis, is due to forefoot abduction and hindfoot valgus. (Neumann)

A patient exhibits swelling and pain in the medial aspect of the ankle. During examination, the patient demonstrates rearfoot pronation in standing position and inability to perform a heel raise on the affected side. The patient demonstrates forefoot abduction when observed from behind. Which of the following conditions is MOST likely present? 1. Eversion ankle sprain 2. Retrocalcaneal bursitis 3. Tarsal tunnel syndrome 4. Posterior tibial tendon dysfunction

Correct Answer: 2 1. The hamate bone is not located within the anatomic snuffbox. 2. The anatomic snuffbox is the landmark for the scaphoid; a fall with an outstretched hand is a common mechanism of injury for a scaphoid fracture. 3. The tendons injured in De Quervain tenosynovitis are located within the anatomic snuffbox; however, they are not injured due to a fall on an outstretched hand. 4. The median nerve is not located within the anatomic snuffbox.

A patient falls with an outstretched hand for protection. Tenderness is present on palpation of the area indicated in the photograph. The patient MOST likely has which of the following conditions? 1. Hamate fracture 2. Scaphoid fracture 3. De Quervain tenosynovitis 4. Median nerve injury

Correct Answer: 2 1. Soft tissue trauma is extensive in a type II or III injury and will require 6 weeks to return to activity. 2. The patient depicted presents with a step-off deformity, which would be considered a type III disruption. Initially, the arm should be put in a sling. The patient will return to activity in about 6 weeks. The second degree of subluxation requires at least 6 weeks to return to activity, although the patient will have good structural strength at about 3 weeks. 3. Immobilization is imperative following a type III injury. 4. The joint will take 6 weeks to return to activity.

A patient fell 1 day ago, sustaining the injury shown in the photograph. What is the MOST appropriate intervention and expected number of weeks for the patient to return to activity? 1. Immobilization at night for 3 weeks; 3 weeks to return to activity 2. Immobilization for 3 weeks; 6 weeks to return to activity 3. Active assistive range of motion exercises; 6 weeks to return to activity 4. Active abduction and lateral (external) rotation exercises; 12 weeks to return to activity

Correct Answer: 1 1. Muscle setting isometric exercises are begun immediately to prevent adhesions of the tendon to the sheath or surrounding tissues and to promote alignment of healing tissue. 2. Weight-bearing is most likely restricted for as long as 6-8 weeks after a lower extremity repair. 3. Passive end-range stretching is not performed until about 8 weeks after surgery and can only be done initially within protected ranges. 4. Concentric exercises with gentle resistance are not performed until the repair has had several weeks to heal. Typically these exercises are not initiated until 8 weeks after the repair.

A patient had a biceps femoris tendon repair 3 days ago. After the initial postsurgical assessment, the physical therapist should FIRST initiate which of the following interventions? 1. Gentle isometric strengthening 2. Closed kinetic chain exercises 3. Passive end-range stretching 4. Concentric exercises with gentle resistance

Correct Answer: 1 1. Blood pressure should not be taken on the ipsilateral side. This is a precaution for patients who have a peripherally inserted central line, as well as for patients who have some other lines. (pp. 450, 458) 2. Although weight-bearing through the hand is a precaution for patients who have an arteriovenous graft (p. 454), it is not a precaution for patients who have a peripherally inserted central catheter inserted through the cephalic vein. 3. Although active upper extremity range of motion is a precaution for patients who have a central line and possibly for patients who have a pulmonary artery catheter, active range of motion is encouraged for patients who have a peripherally inserted line (pp. 450, 458). 4. Although maintaining elevation is a precaution for 24 hours following arteriovenous graft placement (p. 454), it is not a precaution for patients who have a peripherally inserted central catheter.

A patient had a central line peripherally inserted via the cephalic vein. Proper placement has been confirmed. Which of the following activities of the ipsilateral arm should be AVOIDED? 1. Blood pressure measurement 2. Weight-bearing through the hand 3. Active upper extremity range of motion 4. Positioning the hand below the level of the chest

Correct Answer: 3 1. A transfer is initially easier and safer to the strong side (Fairchild). Rotating or twisting the upper body toward the surgical side with the lower extremity fixed should be avoided after total hip arthroplasty (Cheatham). 2. The patient's left side is unaffected and able to bear weight without difficulty. The patient's ability to bear weight on the left would warrant a standing pivot transfer rather than use of a hydraulic lift. (Fairchild) 3. The femoral nerve is vulnerable to injury, which would result in quadriceps weakness. Quadriceps injury leads to knee buckling and possible injury during the transfer. Stabilizing the right knee will allow the transfer to take place with the physical therapist in control of the weak extremity. (Fairchild) 4. Although an abduction pillow would help with hip precautions [avoiding hip medial (internal) rotation and adduction] (Paz), the pillow is bulky and could potentially increase problems because it does not limit hip flexion beyond 90°, which is a potential cause of hip dislocation following a posterolateral approach total hip arthroplasty (Cheatham).

A patient had a right total hip arthroplasty yesterday and sustained a surgical injury to the right femoral nerve. To maximize safety of a bed-to-chair transfer, the physical therapist should: 1. have the patient transfer towards the right. 2. use a hydraulic lift. 3. stabilize the right knee. 4. pre-position with an abduction pillow.

Correct Answer: 4 1. Insulin shock would cause hunger, not a decline in appetite. Fever is also not a symptom of insulin shock. (p. 408) 2. Renal failure would not typically cause a fever (p. 365) 3. The most obvious signs of heart failure are the development of dyspnea and lower extremity edema. A fever might precipitate an exacerbation of heart failure, but a fever is not the result of heart failure. (pp. 238-239) 4. A patient who has the risk factors described in the stem, including diabetes and placement of a Foley catheter, is at risk for a urinary tract infection. Urinary tract infections can result in sepsis. The signs and symptoms of sepsis include fever and confusion. (pp. 364-365)

A patient has a Foley catheter, diabetes mellitus, a decline in appetite and fluid intake, confusion, and an oral temperature of 101°F (38.3°C). The signs and symptoms are MOST consistent with which of the following diagnoses? 1. Insulin shock 2. Renal failure 3. Heart failure 4. Sepsis

Correct Answer: 1 1. The valgus stress test with 20° of knee flexion isolates the medial collateral ligament. Laxity during the test shown would indicate a tear of this ligament. (p. 814) 2. The test for a lateral collateral ligament tear would put a force in the opposite direction, compared to the direction shown in the photograph. The photograph does not show the varus stress test of the knee. (p. 814) 3. If the test shown in the photograph was performed in full knee extension, the results could indicate disruption of multiple knee ligaments, but since it is performed in 20° of flexion, the result suggests a tear of the medial collateral ligament only (p. 815). 4. The test for the anterior cruciate ligament would be performed in a sagittal plane rather than in the frontal plane. Anterior cruciate ligament injury would be found by an anterior drawer or Lachman test. (p. 815)

A patient has a finding of extreme laxity during the test shown in the photograph. Which of the following ligaments is MOST likely injured? 1. Medial collateral 2. Lateral collateral 3. Posterior cruciate 4. Anterior cruciate

Correct Answer: 4 1. Foam dressings are highly absorbent and are appropriate for a granular wound that is draining (p. 191). 2. Collagen dressings are highly absorbent and are appropriate for a granular wound that is draining (p. 193). 3. Calcium alginate dressings are highly absorbent and are appropriate for a granular wound that is draining (p. 192). 4. Hydrogel dressings provide hydration to dry wound beds and are appropriate for a granular wound that is not draining (p. 190).

A patient has a granular ulcer that is non-draining. Which of the following types of dressing would be MOST appropriate for the patient? 1. Foam 2. Collagen 3. Calcium alginate 4. Hydrogel

Correct Answer: 2 1. Cryotherapy would restrict blood and lymph flow due to the physiological reaction to cold. This is contrary to the effect the therapist is attempting to achieve to relieve lymphedema. (Zuther, p. 505) 2. Manual lymphatic drainage and compression with short-stretch dressings are the cornerstones of lymphedema therapy (Zuther, p. 247). 3. Percussive massage, also known at tapotement, is used to stimulate the skin, subcutaneous tissue, and muscle tissue (Andrade). It is applied with considerable pressure and should not be confused with techniques of manual lymph drainage, which are very gentle techniques designed to have an effect on fluid components and lymphatic structures located in the superficial tissues (Zuther, pp. 247-248). 4. Negative pressure therapy is more appropriate for wound treatments (Hamm).

A patient has a history of breast cancer, lymph node dissection surgery, and lymphedema. Which of the following interventions is MOST appropriate? 1. Cryotherapy 2. Compression 3. Percussive massage 4. Negative pressure therapy

Correct Answer: 3 1. With left brain injuries, common impairments include speech and language problems, difficulty planning and sequencing movements, and a cautious behavioral style. Spatial impairments are more likely to be associated with right brain injuries. 2. With left brain injuries, common impairments include speech and language problems, difficulty planning and sequencing movements, and a cautious behavioral style. Spatial impairements, difficulty sustaining movements, and an impulsive behavioral style are more likely to be associated with right brain injuries. 3. Certain sensory, motor, and behavioral impairments are associated with hemispheric brain injuries. With left brain injuries, common impairments include speech and language problems, difficulty planning and sequencing movements, and a cautious behavioral style. 4. With left brain injuries, common impairments include speech and language problems, difficulty planning and sequencing movements, and a cautious behavioral style. Difficulty sustaining movements and impulsive behavior are more likely to be associated with right brain injuries.

A patient has a left brain injury resulting from a cerebrovascular accident. Which of the following impairments are MOST likely to be observed? 1. Spatial impairments, difficulty planning movements, and slow, cautious behavior 2. Spatial impairments, difficulty sustaining movements, and quick, impulsive behavior 3. Speech impairments, difficulty planning movements, and slow, cautious behavior 4. Speech impairments, difficulty sustaining movements, and quick, impulsive behavior

Correct Answer: 1 1. A lesion of the right middle cerebral artery, which affects the right parietal lobe, typically produces impairment of spatial perception (p. 600). 2. Ataxia involving the limbs and gait occurs with damage to the cerebellum, involving the vertebrobasilar arteries (p. 603). 3. Visual agnosia occurs with damage to the left occipital lobe from a lesion of the posterior cerebral artery (p. 601). 4. A memory defect is a characteristic of damage to the inferomedial area of the temporal lobe either bilaterally or only on the dominant side of the brain (usually the left side). Damage to the posterior cerebral artery is implicated. (p. 601)

A patient has a lesion in the right middle cerebral artery. During examination, a physical therapist should expect to find: 1. impaired spatial perception. 2. ataxia of limbs and gait. 3. visual agnosia. 4. short-term memory loss.

Correct Answer: 2 1. A Stage 1 pressure injury is characterized by intact skin with nonblanchable redness of a localized area, usually over a bony prominence. A pink, shiny, shallow wound without slough on the heel associated with bed rest is consistent with a Stage 2 pressure injury. 2. A pink, shiny, shallow wound without slough on the heel associated with bed rest is consistent with a Stage 2 pressure injury. 3. A Stage 3 pressure injury is characterized by full-thickness skin loss. Subcutaneous fat may be visible, but bone, tendon, and muscle are not exposed. A pink, shiny, shallow wound without slough on the heel associated with bed rest is consistent with a Stage 2 pressure injury. 4. A Stage 4 pressure injury is characterized by full-thickness skin loss with exposed bone, tendon, or muscle. A pink, shiny, shallow wound without slough on the heel associated with bed rest is consistent with a Stage 2 pressure injury.

A patient has a pink, shiny, shallow wound without slough on the heel after 3 weeks of bed rest following a motor vehicle accident. Which of the following classifications BEST represents the wound? 1. Stage 1 2. Stage 2 3. Stage 3 4. Stage 4

Correct Answer: 4 1. Anterior shoulder pain and a recent fall while bicycling are consistent with the presence of acromioclavicular joint dysfunction. The test in the photograph is the Hawkins-Kennedy test, which is used to assess for shoulder impingement, not acromioclavicular joint dysfunction. Tests for acromioclavicular joint dysfunction include the O'Brien sign, Paxinos sign, and acromioclavicular joint palpation. 2. Shoulder instability and clicking with throwing activities are consistent with the presence of a labral tear. Additional special tests to assess for labral tears including the crank test, compression rotation test, Speed test, active compression test, Yergason test, and anterior slide test. The Hawkins-Kennedy test, which is shown in the photograph, is not used to assess for labral tears. 3. Reports of diffuse upper extremity pain and numbness into the lateral upper extremity are consistent with neurological involvement, suggesting cervical radiculopathy or thoracic outlet syndrome. Neural signs of tingling/burning are not expected with subacromial impingement. The Hawkins-Kennedy test, which is shown in the photograph, would not be used to assess for thoracic outlet syndrome or cervical myelopathy. 4. The photograph shows the Hawkins-Kennedy test for subacromial impingement. Patients who have subacromial impingement often describe a painful arc and pain with overhead activities.

A patient has a positive finding on the test shown in the photograph. The patient will MOST likely make which of the following subjective reports? 1. Anterior shoulder pain and a recent fall while bicycling 2. Shoulder instability and clicking with throwing activities 3. Diffuse upper extremity pain and numbness into the lateral upper extremity 4. Lateral shoulder pain and pain with overhead activities

Correct Answer: 3 1. Although rare, a tumor could cause symptoms of radicular pain. However, plain radiographs are not considered sensitive to early changes associated with tumors, infection, soft tissue structures, and some fractures. (Dutton, p. 345) 2. Disc herniation is a common cause of cervical radiculopathy. However, plain radiographs are not considered sensitive to early changes associated with tumors, infection, soft tissue structures, and some fractures. (Dutton, pp. 1312-1313) 3. Bone spurs are a common cause of lateral stenosis, which can cause radicular pain. Osteophytes is the correct answer because radiographs provide an excellent view of cortical bone and are more sensitive than magnetic resonance imaging in detecting calcification. (Dutton, pp. 345, 1317) 4. Although rare, a fracture could give some signs of cervical radiculopathy. However, plain radiographs are not considered sensitive to early and small changes associated with acute stress fractures and can result in false-negative findings. (Magee)

A patient has a recent onset of intermittent signs of cervical radiculopathy. Which of the following conditions is MOST likely to be detected by plain radiographs as the structural cause of the symptoms? 1. Tumor 2. Disc herniation 3. Osteophytes 4. Acute stress fractures

Correct Answer: 2 1. The biceps brachii is innervated by the musculocutaneous nerve and would be nonfunctional (p. 170). The supinator would not influence elbow flexion (p. 174). 2. If the biceps brachii and the brachialis are paralyzed, weak elbow flexion is achieved through function of the brachioradialis, which is innervated by the radial nerve (p. 170), with assistance from the pronator teres, which is innervated by the median nerve (p. 173). 3. The action of the extensor digitorum is primarily at the wrist and hand (p. 204). The anconeus would assist with extension at the elbow (p. 172). 4. The brachialis is innervated by the musculocutaneous nerve and would be paralyzed (p. 169). The extensor carpi radialis longus does not have a primary action at the elbow (p. 187).

A patient has a severed musculocutaneous nerve. Which of the following muscles can the patient use to substitute for the resultant loss in upper extremity function? 1. Biceps brachii and supinator 2. Brachioradialis and pronator teres 3. Extensor digitorum and anconeus 4. Brachialis and extensor carpi radialis longus

Correct Answer: 3 1. Mixed red-white coloring is evident with deep partial-thickness burns because the dermis is almost completely destroyed. 2. Marked edema is present with deep partial-thickness burns because of broken blisters and leakage of plasma fluid. Capillary destruction is marked. 3. Intact blisters are the most common sign of superficial partial-thickness burns. Damage is through the epidermis and into the papillary layer of the dermis. 4. Eschar is evident with full-thickness burns. It is a hard, devitalized tissue consisting of coagulated plasma and necrotic cells. Full-thickness burns destroy all of the epidermal and dermal layers and possibly the subcutaneous fat layer.

A patient has a superficial partial-thickness burn. Which of the following signs would MOST likely be observed in the burned area? 1. Mixed red-white coloring 2. Marked edema 3. Intact blisters 4. Eschar

Correct Answer: 4 1. Left thoracic rotation would create a rib hump on the left. In the example described in the stem, the rib hump is on the right. A structural scoliosis creates a rib hump that is present when the patient is in standing position and in full forward flexion. The rib hump in the example is not present with full forward flexion. 2. Although it is correct that this scoliosis would involve right rotation and left side bending, a structural scoliosis creates a rib hump that is present when the patient is in standing position and in full forward flexion. The rib hump in the example is not present with full forward flexion. 3. The patient exhibits a functional scoliosis since the curve disappears upon forward bending. However, left thoracic rotation and right side bending would be seen as a rib hump on the left, not the right. 4. A right thoracic rib hump is associated with a right thoracic rotoscoliosis. Scoliosis is an abnormal lateral curve that can be structural or functional. Structural scoliosis is a fixed deformity that persists during forward bending. Functional scoliosis is a changeable adaptation that is not present when fully forward flexed. The abnormal lateral curve is associated with rotation to the convex side of the curve. The rotation causes the transverse processes of the vertebrae to move posteriorly on the ipsilateral side, and the attached ribs follow to create a rib hump. In this example, the rib hump is on the right, so the rotation is to the right. The rotational component of scoliosis most frequently occurs to the convex side of the curve, so this example most likely involves left side bending.

A patient has a thoracic right rib hump that is present when the patient is standing but disappears upon forward bending. Which of the following conditions is MOST likely present? 1. Structural scoliosis involving left thoracic rotation and right side bending 2. Structural scoliosis involving right thoracic rotation and left side bending 3. Functional scoliosis involving left thoracic rotation and right side bending 4. Functional scoliosis involving right thoracic rotation and left side bending

Correct Answer: 3 1. Increased stance phase would be associated with increased time weight-bearing on the sole of the shoe (p. 231). 2. Increased/excessive toe off (preswing) would be associated with increased time weight-bearing on the front portion of the sole of the shoe (p. 231). 3. Decreased dorsiflexion would cause increased wear to the anterior portion of the toe box secondary to poor clearance and toe drag (p. 233). 4. Circumduction would not cause scuffing and wear to the outside of the toe box because the limb is lifted off the floor to clear the foot/toe box during circumduction (p. 233).

A patient has diabetes with peripheral neuropathy. The patient's shoe on the involved side shows scuffing and wear on the outside of the shoe over the toe box. Which of the following gait deviations is MOST likely contributing to the wear on the shoe? 1. Increased stance phase 2. Excessive toe off (preswing) 3. Decreased dorsiflexion 4. Circumduction

Correct Answer: 3 1. With arterial aneurysm in the lower extremity, the affected artery is dilated (p. 632) and there is decreased blood flow and ischemia in the limbs. In this case, the ankle-brachial index should be less than 1.0. 2. Arterial thrombosis is an occlusive disease of the arteries. With occlusive diseases, the blood flow to the lower extremity decreases. Decreased blood flow to the lower extremities will result in an ankle-brachial index of less than 1.0. (p. 638) 3. Ankle-brachial index is a ratio of the systolic blood pressure at the ankle and the brachial systolic pressure. The normal value of the ankle-brachial index is 1.0, indicating similar blood flow in the ankle and brachial arteries. An ankle-brachial index greater than 1.1 relates to arterial calcification in the leg. With arterial calcification, the artery cannot be fully compressed for valid measurement of arterial pressure at the ankle. An ankle-brachial index greater than 1.1 is mostly found in patients who have diabetes. (p. 645) 4. The normal value of the ankle-brachial index is 1.0. With severe arterial occlusion, the ankle-brachial index will be less than 1.0. An ankle-brachial index of 1.1 or higher is not an indication of arterial occlusion. (pp. 641, 645)

A patient has an ankle-brachial index (ABI) of 1.5. Which of the following conditions affecting the lower extremity should a physical therapist suspect? 1. Arterial aneurysm 2. Arterial thrombosis 3. Arterial calcification 4. Arterial occlusive disease

Correct Answer: 2 1. Arterial wounds are typically on the dorsum of the foot, lateral leg, or toes and have minimal edema initially. Typically edema is only present if the limb is held in a dependent position. There is minimal exudate but severe pain. These wounds can be quite deep, involving even tendon and bone. These characteristics are not present in venous wounds. (p. 383) 2. The aching pain, especially worsened with the limb in the dependent position, is a classic sign for venous wounds. The dependent position causes increased pressure in the venous system. (p. 343) 3. Neuropathic ulcers are typically located in the forefoot area, specifically on the metatarsal heads, toes, or over an area of increased weight-bearing that may be present because of a foot deformity. Neuropathic ulcers are typically not found on the leg. (pp. 399, 402) 4. A pressure injury is typically present over a bony prominence, such as the lateral malleolus or calcaneus. Pressure injuries have taut, shiny, hardened skin around the wound, with edema limited to that area. Venous wounds have marked edema. (p. 310)

A patient has an irregularly shaped, open wound on the medial leg that is 5 mm deep and 25 mm wide. Moderately severe edema is observed. The patient reports aching pain that worsens when the leg is in a dependent position and decreases when the leg is elevated. The patient MOST likely has which of the following conditions? 1. Arterial wound 2. Venous wound 3. Neuropathic ulcer 4. Pressure injury

Correct Answer: 4 1. The mechanism of injury described would cause a superior labrum anterior to posterior (SLAP) lesion, which is indicated by the positive result on the clunk test (Magee, pp. 321-322). A contrast arthrography radiograph would demonstrate adhesive capsulitis, not a labral lesion. A labral lesion would be best confirmed through magnetic resonance arthrography. (McKinnis, pp. 20-21, 546) 2. The mechanism of injury described is one of anterior dislocation causing a labral tear with a superior labrum anterior to posterior (SLAP) lesion, which is indicated by the positive result on the clunk test (Magee, pp. 321-322). An anterior-posterior bilateral radiograph with weights is utilized to diagnose acromioclavicular injuries (McKinnis, p. 546). 3. The mechanism of injury described is one of anterior dislocation causing a labral tear with a superior labrum anterior to posterior (SLAP) lesion, which is indicated by the positive result on the clunk test (Magee, pp. 321-322). Magnetic resonance arthrogram is the best choice for imaging labral tears and is preferred over a computed tomography arthrogram (McKinnis, pp. 529, 546). 4. Magnetic resonance arthrogram is correct because the mechanism of injury described is one of anterior dislocation causing a labral tear with a superior labrum anterior to posterior (SLAP) lesion, which is indicated by the positive result on the clunk test (Magee, pp. 321-322). Magnetic resonance arthrogram is the best choice for imaging labral tears (McKinnis, p. 529).

A patient has anterior shoulder pain and a positive result on the shoulder clunk test after performing repetitive overhead throwing. Results of which of the following imaging studies would BEST confirm a diagnosis? 1. Contrast arthrography radiograph 2. Anterior-posterior bilateral radiograph with weights 3. Computed tomography arthrogram 4. Magnetic resonance arthrogram

Correct Answer: 3 1. Knowledge of results (terminal feedback about the outcome of the movement in terms of the movement's goal) has been shown to be important for learning motor tasks. Feedback for very simple movements has been shown to be more beneficial if given every 15 repetitions as a summary. For more complex tasks, such as hitting a ball with a bat, the most effective summary length for learning is every five repetitions. 2. Knowledge of results (terminal feedback about the outcome of the movement in terms of the movement's goal) has been shown to be important for learning motor tasks. Feedback for very simple movements has been shown to be more beneficial if given every 15 repetitions as a summary. For more complex tasks, such as hitting a ball with a bat, the most effective summary length for learning is every five repetitions. 3. Knowledge of results (terminal feedback about the outcome of the movement in terms of the movement's goal) has been shown to be important for learning motor tasks. Feedback for very simple movements has been shown to be more beneficial if given every 15 repetitions as a summary. For more complex tasks, such as hitting a ball with a bat, the most effective summary length for learning is every five repetitions. 4. Knowledge of results (terminal feedback about the outcome of the movement in terms of the movement's goal) has been shown to be important for learning motor tasks. Feedback for very simple movements has been shown to be more beneficial if given every 15 repetitions as a summary. For more complex tasks, such as hitting a ball with a bat, the most effective summary length for learning is every five repetitions.

A patient has begun practicing 50 repetitions of hitting a ball tossed by the physical therapist. Which of the following options BEST reflects the feedback that should be given regarding the number of times the patient used the correct technique for hitting the ball? 1. No feedback is required. 2. Feedback should be given after every repetition. 3. Feedback should be given after every five repetitions. 4. Feedback should be given after the 50 repetitions.

Correct Answer: 1 1. Agonist reversals include resisted concentric, isometric, and eccentric contraction tasks. Therefore, a patient who has decreased eccentric control would benefit from this technique. (pp. 275, 277) 2. Dynamic reversals only utilize concentric contractions. This would not be an appropriate choice for a patient who has impaired eccentric control. (p. 275) 3. Rhythmic stabilization only utilizes isometric contractions. This would not be an appropriate choice for a patient who has impaired eccentric control. (p. 273) 4. Rhythmic initiation only utilizes concentric contractions. This would not be an appropriate choice for a patient who has impaired eccentric control. (p. 264)

A patient has decreased eccentric control of the elbow flexors. Which of the following proprioceptive neuromuscular facilitation (PNF) techniques is MOST appropriate to perform? 1. Agonist reversals (combination of isotonics) 2. Dynamic reversals (isotonic reversals) 3. Rhythmic stabilization 4. Rhythmic initiation

Correct Answer: 1 1. Vertebral (basilar) arteries supply the brainstem and cerebellum. Lesions of these arteries usually manifest as unilateral or bilateral weakness of extremities and loss of vibratory sense, two-point discrimination, and position sense. Diplopia, homonymous hemianopsia, dysphagia, dysarthria, nausea, and confusion may also occur. 2. The anterior cerebral artery supplies the superior surfaces of frontal and parietal lobes and the medial surfaces of the cerebral hemispheres, which control the motor and somesthetic cortex serving the legs. The frontal lobe controls the personality; since personality changes are not mentioned, this artery is not likely to be affected. Also, lesions of this artery are most likely to produce hemiparesis or hemiplegia, not bilateral weakness. 3. The middle cerebral artery supplies the frontal lobe, parietal lobe, and cortical surfaces of the temporal lobe and, therefore, affects higher cerebral processes of communication, language interpretation, and interpretation of space, sensation, form, and voluntary movement. Lesions of this artery are most likely to manifest as alterations in communication, cognition, mobility, and sensation. Contralateral hemianopsia and hemiplegia (greater in the face and arm rather than leg) is also likely to be observed. Also, lesions of this artery are most likely to produce hemiparesis or hemiplegia, not bilateral weakness. 4. The posterior cerebral artery supplies the medial and inferior temporal lobes, medial occipital lobe, thalamus, posterior hypothalamus, and visual receptive area. Lesions of this artery are most likely to manifest as contralateral hemiplegia (greater in the face and arm than in the leg), not bilateral weakness, ataxia/tremor, homonymous hemianopsia, cortical blindness, receptive aphasia, and memory deficits. Since ataxia and tremors are not mentioned as the presenting symptoms, this artery is not likely to be affected.

A patient has diplopia, dysphagia, and bilateral weakness of the lower extremities. The patient also has loss of vibratory sense, two-point discrimination, and position sense. There are no signs of personality changes or aphasia. Which of the following arteries is MOST likely affected? 1. Basilar 2. Anterior cerebral 3. Middle cerebral 4. Posterior cerebral

Correct Answer: 2 1. Weakness in the gastrocnemius-soleus muscle results in excessive knee flexion during stance phase (Dutton, pp. 977-978). 2. Excessive ankle eversion during stance is most frequently associated with marked weakness of inverters such as the tibialis anterior and tibialis posterior (Dutton, p. 309), which may result from a malalignment such as a forefoot or rearfoot varus/valgus (Magee). Other causes of excessive eversion include plantar flexion contracture, fibular (peroneal) hypertonicity, and valgus deformity (Dutton, p. 314). A manual muscle test of the tibialis anterior and tibialis posterior muscles would best determine if these muscles are weak and in need of strengthening (Dutton, pp. 1136-1137). 3. The modified Ashworth test is used to examine spasticity (Umphred). However, excessive activation of the tibialis posterior muscle during walking (as would occur with spasticity) results in excessive inversion. The case describes excessive foot eversion. 4. The modified Ashworth test is used to examine spasticity (Umphred). Excessive activation of the tibialis anterior muscle during walking (as would occur with spasticity) results in excessive inversion. The case describes excessive foot eversion.

A patient has excessive ankle eversion when walking. Which of the following examination measures is MOST likely to determine the cause of the patient's gait deviation? 1. Manual muscle test of the gastrocnemius and soleus 2. Manual muscle test of the tibialis anterior and tibialis posterior 3. Modified Ashworth test of the tibialis posterior and flexor digitorum 4. Modified Ashworth test of the tibialis anterior and extensor digitorum

Correct Answer: 4 1. Asthma is associated with sputum that is predominantly eosinophilic, and bronchodilators improve spirometry scores of patients who have asthma (Hillegass, pp. 207-209). 2. Pneumococcal pneumonia is associated with sputum that is most often pinkish, blood-flecked, or rusty and will show evidence of bacteria when cultured. Treatment is centered on antibiotics. Oxygen can be administered, but bronchodilators are not a treatment of choice. (Hillegass, pp. 142-143) 3. Chronic bronchitis is associated with sputum that is predominantly neutrophilic, and bronchodilators improve the spirometry scores of patients who have chronic bronchitis (Goodman). 4. Emphysema has the features of higher than normal residual volume (because of destroyed alveolar walls and enlarged air spaces), absent or mucoid sputum (as opposed to sputum with a lot of neutrophils), and spirometry measures that are unimproved with bronchodilators (unlike asthma, which improves with bronchodilators) (Hillegass, pp. 192-193, 197).

A patient has higher than normal residual volume, absent or mucoid sputum, and spirometry measures that are unimproved with bronchodilators. The patient MOST likely has which of the following conditions? 1. Asthma 2. Pneumococcal pneumonia 3. Chronic bronchitis 4. Emphysema

Correct Answer: 1 1. Jaundice, darkened urine, and ascites are all clinical signs of liver disease. Asterixis, or liver flap, is also likely to be present as a result of ammonia imbalance, which causes this neurologic symptom. (Goodman, p. 341) 2. Pronator drift is more likely to be observed in the presence of an upper motor neuron disorder. Upper motor neuron signs/symptoms are associated with central nervous system conditions, such as cerebrovascular accident, spinal cord injury, and multiple sclerosis. (Fruth, pp. 397, 400) 3. A positive result on the Hoffman test, or Hoffman sign, is associated with corticospinal tract disorders. A positive result may be found contralateral to the area of a brain lesion or bilaterally in the presence of injury or compression of the spinal cord. (Fruth, p. 402) 4. Jaundice, ascites, and darkened urine are common clinical signs of liver disease. Rebound tenderness is most likely to be associated with appendicitis and peritonitis. (Goodman, pp. 319-320)

A patient has jaundice, dark urine, and ascites. Which of the following findings is MOST likely to be present during the physical therapy examination? 1. Asterixis 2. Pronator drift 3. Hoffman 4. Rebound tenderness

Correct Answer: 3 1. Common fibular (peroneal) nerve entrapment would result in pain in the lateral surface of the knee, leg, and foot (p. 100). 2. Deltoid ligament strain is associated with pain with palpation of the ligament area medially and would not be associated with weakness of the toe flexors (p. 1087). 3. Pain only with passive ankle eversion, a pronated foot, valgus deformity, and weak toe flexion strength are associated with tarsal tunnel syndrome (p. 1137). 4. Although many of the signs and symptoms described in the stem are consistent with tibialis posterior tendinitis, the weakness of the toe flexors would not be associated with this condition (p. 1136).

A patient has medial ankle pain, a pronated foot with a calcaneal valgus deformity, pain with passive ankle eversion, and weakness of the great toe flexors. The patient MOST likely has which of the following conditions? 1. Common fibular (peroneal) nerve injury 2. Deltoid ligament strain 3. Tarsal tunnel syndrome 4. Tibialis posterior tendinitis

Correct Answer: 1 1. Stretching is a stimulus in the early healing stages for the proper alignment of healing collagen. Gentle stretching is performed to provide a stimulus for fiber orientation without disruption of the immature collagen. (p. 522) 2. Ultrasound would not be appropriate for a patient who has diminished sensation (p. 381). 3. Cross-friction techniques are commonly performed for chronic conditions, not acute or subacute conditions (p. 422). 4. Use of a hot pack would not be appropriate for a patient who has diminished sensation (p. 381).

A patient who has decreased tactile sensation due to diabetes mellitus is being treated for a mild Achilles tendon strain. Which of the following interventions is MOST appropriate to restore normal alignment of the healing collagen? 1. Gastrocnemius stretching 2. Ultrasound to the gastrocnemius 3. Cross-friction massage at the muscle belly 4. Hot pack to the gastrocnemius

Correct Answer: 4 1. Central cord syndrome is caused by hyperextension injury (trauma) with bleeding into the central spinal cord (Umphred). 2. A transverse ligament tear would present with a history of trauma, heaviness of the head, lump in the throat, nausea, headache, and dizziness, not the signs and symptoms presented in the stem (Dutton, pp. 1222-1223). 3. Cervical disc herniation would present with signs and symptoms specifically limited to local findings for the level of involvement, for example, dermatome (anterolateral shoulder/arm), myotome (deltoid/biceps), and deep tendon reflex (biceps) signs associated with the right cervical spine C5 nerve root. (Dutton, pp. 1312-1314) 4. All of the signs and symptoms in the stem fit the clinical presentation of cervical myelopathy (Goodman).

A patient has nontraumatic neck and shoulder pain, decreased hand dexterity, paresthesia in the right upper extremity, hyperreflexia, and urinary retention with overflow incontinence. The patient MOST likely has which of the following conditions? 1. Central cord syndrome 2. Cervical transverse ligament tear 3. Cervical disc herniation 4. Cervical myelopathy

Correct Answer: 2 1. The photograph depicts the L5 dermatome (Moore; Magee, p. 942). Although the knee flexors are innervated by L5-S2, knee extensors are innervated by L3-L4 (Moore; Magee, p. 587). 2. The photograph depicts the L5 dermatome (Moore; Magee, p. 942). Muscles innervated by L5 include the extensor hallucis longus (great toe extension) and fibularis (peroneal) muscles (foot eversion) (Moore). 3. The photograph depicts the L5 dermatome (Moore; Magee, p. 942). Knee extensors (quadriceps) are innervated by L3-L4, while ankle plantar flexors (gastrocnemius/soleus) are innervated by S1-S2 (Moore; Magee, p. 587). 4. The photograph depicts the L5 dermatome (Moore; Magee, p. 942). Although the muscles involved in foot inversion are innervated by L4-L5-S1, plantar flexors are innervated by S1-S2 (Moore; Magee, p. 587).

A patient has pain and sensory changes in the distribution shown in the photograph. The patient is MOST likely to exhibit impaired strength in which of the following pairs of motions? 1. Knee extension and knee flexion 2. Great toe extension and foot eversion 3. Knee extension and ankle plantar flexion 4. Foot inversion and ankle plantar flexion

Correct Answer: 2 1. To control edema, a compression wrap should be used with more pressure applied distally than proximally. 2. When applying compression wraps to control edema, a spiral wrap is used with more pressure applied distally than proximally. 3. In no case should a wrap be applied with the proximal pressure greater than the distal pressure. 4. When applying a compression wrap for joint support, the wrap is applied with even pressure distally to proximally. This patient needs edema control, for which more pressure distally than proximally is used.

A patient has sustained a moderate ankle sprain with significant swelling greater than 1 inch (2.5 cm) throughout the ankle and into the foot. Which of the following wrapping techniques is MOST appropriate to control the edema? 1. Figure-8 compression wrap with consistent pressure on the limb distally and proximally 2. Spiral compression wrap with more pressure on the limb distally than proximally 3. Figure-8 compression wrap with more pressure on the limb proximally than distally 4. Spiral compression wrap with consistent pressure on the limb distally and proximally

Correct Answer: 1 1. The photograph depicts a hammer toe (p. 1003). A high toe box would allow space for the hammer toe and would decrease irritation (pp. 1289, 1292). 2. A metatarsal pad transfers stress from the metatarsal heads to the metatarsal shafts, reducing plantar pressure (p. 1290). The photograph shows pressure on the dorsum of the hammer toe, which would not be relieved by a metatarsal pad. 3. Rocker bars affix to the sole of the shoe proximal to the metatarsal heads to reduce the distance the patient travels during stance phase and to shift the load from the metatarsophalangeal joints to the metatarsal shaft (pp. 1291-1292). The photograph shows pressure on the dorsum of the hammer toe, which would not be relieved by a rocker bar. 4. A metatarsal bar is placed posterior to the metatarsal heads on the sole of a shoe to assist in transferring stress from the metatarsophalangeal joints to the metatarsal shafts during late stance (p. 1292). The photograph shows pressure on the dorsum of the hammer toe, which would not be relieved by a metatarsal bar.

A patient has the deformity shown in the photograph. Which of the following shoe modifications is MOST likely to be beneficial for this patient? 1. High toe box 2. Metatarsal pad 3. Rocker bar 4. Metatarsal bar

Correct Answer: 3 1. Serous drainage is a clear, watery drainage. 2. Chylous drainage is milky, white drainage that occurs after abdominal surgeries where there is trauma to the cisterna chyli or adjacent lymphatic trunks. 3. Sanguineous drainage is described as thin, bloody drainage from a surgical wound or acute traumatic wound. 4. Purulent drainage is thick, often odiferous drainage that contains large amounts of bacteria.

A patient has thin, bloody drainage from an abdominal surgical wound. This finding should be classified as which of the following types of drainage? 1. Serous 2. Chylous 3. Sanguinous 4. Purulent

Correct Answer: 2 1. When mobilizing a patient who has a temporary pacemaker, the therapist should be aware of the location of the pacemaker and the wires and should monitor the patient continuously during mobilization. Although mobilization is not contraindicated in the presence of a temporary pacemaker, restrictions may be associated with the underlying condition, and the therapist should check with the nurse or physician regarding these types of restrictions. (Hillegass, p. 438) 2. When mobilizing patients who have an intra-aortic balloon pump, no hip flexion is allowed in the leg where the catheter is inserted, therefore, sitting at the edge of the bed or moving from sitting to standing position would be restricted. Patients are restricted to strict bed rest but can participate in therapeutic activities. Out-of-bed activities are contraindicated until the intra-aortic balloon pump is removed. (Hillegass, p. 440) 3. The presence of a chest tube is not a contraindication to mobilization (Hillegass, p. 437). 4. Mechanical ventilation is typically not a contraindication for physical therapy (Paz). Although not a weaning mode of ventilation, assist/control mode allows the patient to generate as many breaths as needed. Patients usually tolerate increased demands during physical therapy if medically stable (Hillegass, p. 435). After cardiovascular surgery, the individual will still have good respiratory drive.

A patient in intensive care is recovering from cardiovascular surgery. Which of the following medical devices would require the MOST restrictions for implementing an upright mobility program beginning with sitting and transferring from the bed to a chair? 1. Temporary pacemaker with an external pacing box 2. Intraaortic balloon pump via femoral sheath access 3. Chest tubes inserted bilaterally at the sixth intercostal space 4. Assist control mode of mechanical ventilation

Correct Answer: 1 1. The Dynamic Gait Index was designed to examine postural control during gait. It can help identify patients who have vestibular dysfunction and examines fall risk by means of a cutoff score. (Umphred) 2. Observational Gait Analysis is a functional measure used for the examination of movement patterns during gait. It is used to identify gait deficits. The therapist examines the motion of each major joint during phases of gait. It does not involve scoring of fall risk, nor does it specifically examine postural control. (Shumway-Cook, pp. 427-428) 3. The Wolf Motor Function Test assesses upper extremity function of patients post stroke. It does not include an examination of gait. (Shumway-Cook, p. 553) 4. The Segmental Assessment of Trunk Control examines balance and postural control only in the seated position (Shumway-Cook, pp. 273-275).

A patient is admitted to a local hospital after experiencing falls and severe oscillopsia for the past 2 days. To examine the patient's postural control during gait, which of the following measures is BEST to perform? 1. Dynamic Gait Index 2. Observational Gait Analysis 3. Wolf Motor Function Test 4. Segmental Assessment of Trunk Control

Correct Answer: 3 1. Changes in a patient's skin and nail beds may be the first sign of an inflammatory, infectious, or immunological disorder. Significant changes warrant further questioning and a report to the patient's physician. It would not be appropriate to ignore the findings. (p. 163) 2. Changes in a patient's skin and nail beds might be a sign of systemic disease. It would not be necessary to stop physical therapy, but a referral to the patient's physician would be appropriate. (p. 163) 3. Any changes in the integumentary system maybe the precursor to infection, inflammation, and systemic disease. Since the patient declines any follow-up, changes should be reported to the physician. (pp. 163, 169) 4. It would not be necessary to alarm the patient by a referral to an emergency department. Questioning the patient about changes is appropriate, as is informing the physician. (p. 169)

A patient is being reevaluated after participating in 4 weeks of physical therapy. During the reevaluation, the physical therapist notices significant changes in the patient's skin and nail beds. The patient reports noticing the changes but does not consider them important enough to visit the physician. Which of the following actions would be MOST appropriate for the therapist? 1. Continue with the current treatment plan without taking any additional steps. 2. Put physical therapy on hold until the patient's skin and nail bed changes resolve. 3. Report the changes in the patient's skin and nail beds to the physician. 4. Refer the patient to an emergency department or urgent care center.

Correct Answer: 2 1. Although increasing the resistance may improve strength in the short term, in the absence of increasing the speed at which the exercise is performed, power may actually be reduced. Increasing the repetitions per set beyond 15 would be more likely to improve endurance. The combination would not be as likely to increase power. 2. Power requires a combination of strength and speed. Strength is improved by training at a greater percentage of the one-repetition maximum, at least 60% for untrained individuals and 80% to 100% for an advanced training program. Speed is an essential component of power. Strength training in the absence of increasing speed may actually reduce power. In the example described in the stem, the individual is training below 60%, so increasing the weight at which the exercise is performed, in combination with increasing the speed at which the exercise is performed, would be most likely to increase power. 3. Decreasing the sets, while increasing the number of repetitions per set, may keep the training volume the same. However, without an increase in resistance and an increase in speed, this modification would be unlikely to generate an increase in power. 4. Although increasing the resistance may lead to an increase in strength, reducing the speed at which the exercise is performed would not be likely to improve power.

A patient is performing three sets of 15 repetitions at 30% of the one-repetition maximum. To improve power, which of the following modifications would be BEST? 1. Increase the resistance and the number of repetitions per set. 2. Increase the resistance and the speed at which the exercise is performed. 3. Decrease the number of sets and increase the number of repetitions per set. 4. Increase the resistance and decrease the speed at which the exercise is performed.

Correct Answer: 4 1. Motor learning principles suggest that psychomotor skills are best learned when practice conditions allow errors to occur, when performers are encouraged to engage in active sensory encoding and retrieval processes, and when knowledge of results is used minimally. The conditions within this choice do not create variations within the task. 2. Motor learning principles suggest that psychomotor skills are best learned when practice conditions allow errors to occur, when performers are encouraged to engage in active sensory encoding and retrieval processes, and when knowledge of results is used minimally. The conditions within this choice do not vary the task and provide too much feedback. 3. Motor learning principles suggest that psychomotor skills are best learned when practice conditions allow errors to occur, when performers are encouraged to engage in active sensory encoding and retrieval processes, and when knowledge of results is used minimally. The conditions in this choice do not create variations within the task and provide too much feedback. 4. Motor learning principles suggest that psychomotor skills are best learned when practice conditions allow errors to occur, when performers are encouraged to engage in active sensory encoding and retrieval processes and when knowledge of results is used minimally. Such practice typically involves varying the task, varying the environment in which the task occurs, and providing minimal feedback of results.

A patient is practicing moving from seated to standing position. Which of the following transfers to standing position would BEST facilitate motor learning of the task? 1. From a single chair at a self-selected speed, with minimal feedback of results 2. From a single chair at a variety of speeds, with maximum feedback of results 3. From a variety of chairs at a single speed, with maximum feedback of results 4. From a variety of chairs at a variety of speeds, with minimal feedback of results

Correct Answer: 3 1. Use of a quad cane in the right hand is not the best choice for many reasons. The cane is on the wrong side for a reciprocal gait pattern. The cane on the affected side would result in increased, rather than decreased, weight-bearing on the right. (pp. 295-296, 381) 2. The quad cane would not offer enough support to relieve weight-bearing to partial weight-bearing without the postural deviations shown in the photograph. It would be difficult to achieve a normal gait pattern and achieve weight-bearing at the same time. (p. 381) 3. A quad cane may not offer enough support to relieve weight-bearing to partial weight-bearing. Given the patient's age, crutches appear to be a better choice. (pp. 294, 308-309) 4. Use of a standard walker is an alternative, but given the age and apparent ability of the patient, a walker would provide more support than the patient requires. A physical therapist should select the least restrictive device. (pp. 292-294)

A patient is referred for partial weight bearing (25%) on the right and arrives in the clinic walking with a quad cane in the left hand. The physical therapist should instruct the patient in: 1. proper use of a quad cane in the right hand. 2. proper use of a quad cane in the left hand. 3. use of crutches instead of a quad cane. 4. use of a standard walker instead of a quad cane.

Correct Answer: 1 1. Marked ulnar drift is a hallmark sign of rheumatoid arthritis (O'Sullivan, p. 1000). Because cervical spine ligaments can be affected in this population, cervical stabilization exercises in neutral are appropriate for managing neck pain in patients who have rheumatoid arthritis (Brody). 2. Cervical spine ligaments can be affected in patients who have rheumatoid arthritis, placing the patient at high risk for subluxation and significant complications from cervical mobilization (O'Sullivan, p. 998). 3. Cervical traction is contraindicated in patients who have rheumatoid arthritis (Belanger, p. 334). 4. Although thermal agents can provide some local pain relief, application of heating modalities to an area with potential ligamentous laxity or acute inflammation is not recommended for patients who have rheumatoid arthritis (Belanger, p. 82; O'Sullivan, p. 1024).

A patient is referred to a physical therapist for treatment of chronic neck pain. During the examination, the therapist notices that the patient has a marked ulnar drift in both hands at the MCP joints. Which of the following treatments should the therapist perform? 1. Cervical stabilization exercises with the patient in good postural alignment 2. Gentle cervical mobilization for 8 minutes 3. Intermittent cervical traction at 8% of the patient's body weight for 10 minutes 4. Moist hot pack on the patient's cervical spine with the patient in sitting position for 15 minutes

Correct Answer: 4 1. The purpose of postural drainage is to drain secretions for airway clearance. Postural drainage for the lung bases would require prone positioning, which is not likely to be tolerated by a patient who just had abdominal surgery. Splinted coughing is more appropriate for airway clearance for patients who have undergone surgery. (pp. 541, 545) 2. The oxygen saturation is within normal limits, so supplemental oxygen appears unnecessary (p. 420). 3. Deep coughing may be too painful 1 day after surgery. Splinted coughing is more appropriate for airway clearance for patients who have undergone surgery. (pp. 544-545) 4. Atelectasis is present in up to 95% of patient who undergo abdominal surgery (p. 180). Deep breathing (diaphragmatic breathing) is used to resolve atelectasis and increase oxygenation (pp. 550-551, 553).

A patient is referred to physical therapy 1 day after having a colon resection with a colostomy. Breath sounds are decreased at lung bases, and arterial oxygen saturation is 91% on room air. Which of the following interventions is BEST for the patient? 1. Postural drainage 2. Supplemental oxygen 3. Deep coughing 4. Deep-breathing exercises

Correct Answer: 4 1. A backward trunk lean, not forward, is associated with compensations to advance the lower extremity (pp. 1008-1009). 2. Walking in a forward leaning posture does not require disassociation of trunk movements from the pelvis or lower extremities (p. 1007). 3. Hip extensors are less in demand when leaning backward, not forward. With weak hip extensors, the trunk would be displaced posteriorly relative to the lower extremities. (pp. 1008-1009) 4. The patient leans forward to improve stability in the forward progression in gait. In addition, the head is flexed and held with a downward gaze because the patient is relying on visual input due to decreased sensory input. (pp. 1006-1007)

A patient maintains the posture shown in the photograph throughout the gait cycle and reports a sense of numbness in both legs when walking. The MOST probable explanation for this posture is that the patient is: 1. using the trunk position to advance the lower extremity. 2. unable to disassociate trunk movements from pelvic or limb movement. 3. attempting to decrease the demand on the hip extensors. 4. substituting visual input for impaired proprioception in the lower extremities.

Correct Answer: 2 1. Weak dorsiflexors are typically associated with the gait deviation described in the stem (O'Sullivan). Strengthening the plantaris, a plantar flexor, would not correct this gait deviation. 2. Weak dorsiflexors are typically associated with the gait deviation described in the stem. Tibialis anterior strengthening will improve ankle dorsiflexion and, therefore, improve this gait pattern. (O'Sullivan) 3. Limited dorsiflexion range of motion is typically associated with the gait deviation described in the stem (O'Sullivan). A talocrural posterior-anterior glide mobilization is intended to improve ankle plantar flexion and would not help to improve this gait pattern (Dutton, p. 1126). 4. Tibiofemoral posterior glide mobilization is intended to improve knee extension range of motion and would not help to improve the gait pattern described in the stem (Dutton, p. 1062).

A patient makes heel strike (initial contact) with the midfoot while walking. Which of the following gait interventions would MOST effectively normalize the patient's gait pattern? 1. Strengthening of the plantaris muscle 2. Strengthening of the tibialis anterior muscle 3. Posterior-anterior mobilization of the talocrural joint 4. Posterior-anterior mobilization of the tibiofemoral joint

Correct Answer: 2 1. The physician should be contacted immediately because the new symptoms could indicate a compression of the cauda equina. 2. Withholding physical therapy, referring the patient to an emergency department, and contacting the referring physician is the best course of action. The patient's new symptoms of night pain, urinary incontinence, and abdominal pain may indicate the presence of cauda equina syndrome and require medical assessment. 3. The physician should be informed of the patient's new symptoms, which could indicate the presence of cauda equina syndrome and require medical assessment. 4. The patient's symptoms may indicate cauda equina syndrome. The physical therapist should contact the physician immediately.

A patient receiving physical therapy due to a history of lumbar and thoracic pain reports a new onset of night pain, urinary incontinence, and severe abdominal pain after a recent fall from a roof. Which of the following courses of action is BEST for the physical therapist? 1. Document the patient's subjective report, and withhold therapy until the following week. 2. Withhold physical therapy, refer the patient to an emergency department, and contact the referring physician. 3. Document the patient's subjective report, palpate the painful area, and perform gentle lumbar oscillations. 4. Provide gentle lumbar mobilization and traction techniques, and send an email to the physician describing the patient's symptoms.

Correct Answer: 2 1. Trendelenburg gait is attributed to weakness in the gluteus medius muscle or L5 nerve root involvement (Magee, pp. 585, 1009). 2. The L4 nerve root is the main segmental innervation to the tibialis anterior. The L4 nerve root is also the myotome for ankle dorsiflexion. Impingement of the L4 nerve root would result in foot slap. (Magee, p. 585; O'Sullivan, p. 239) 3. Backward trunk lean reduces demands on a weakened stance limb gluteus maximus (O'Sullivan, p. 242). The gluteus maximus is innervated by the inferior gluteal nerve (L5-S2) (Magee, pp. 585, 1008-1009). 4. Causes of toe walking include a tight Achilles tendon, clubfoot, cerebral palsy, or limb length discrepancies (Magee, p. 1009; O'Sullivan, p. 239). It is not associated with L4 impingement.

A patient with low back pain has L4 nerve root impingement. The patient will MOST likely demonstrate which of the following gait deviations? 1. Trendelenburg gait 2. Foot slap 3. Posterior thrust of the trunk at heel strike (initial contact) 4. Toe walking

Correct Answer: 3 1. Dependent edema is one of the early signs of right-sided heart failure. The edema is usually symmetric and occurs in the feet and ankles (Goodman, Differential Diagnosis; Goodman, Pathology, pp. 591-592). 2. Edema associated with chronic venous insufficiency usually presents in a gaiter distribution creating the appearance of an inverted bottle at the calf. Acute onset of swelling at the dorsum of the foot, as described in the stem, is not typical of chronic venous changes. (Goodman, Pathology, pp. 655-656) 3. Lymphedema is usually unilateral with typical presentation distally on the extremity (dorsum of the foot or hand) (Goodman, Differential Diagnosis; Goodman, Pathology, pp. 680-682). 4. Lipedema is a symmetrical swelling of both legs, extending from hips to ankles. Lipedema onset is primarily proximal to distal. (Goodman, Pathology, pp. 702-704).

A patient reports an insidious onset of swelling of 1 month's duration on the dorsum of the left foot. Which of the following conditions is the MOST likely cause? 1. Heart failure 2. Chronic venous insufficiency 3. Lymphedema 4. Lipedema

Correct Answer: 3 1. Leaving this message would violate Health Insurance Portability and Accountability Act (HIPAA) regulations. Messages should be limited to clinic name and call-back number without any information specific to the patient's therapy program. (p. 363) 2. The physical therapist must give the patient advanced notice of the intent to discharge (p. 248). 3. Termination of the provider-patient relationship is justified when the patient makes a knowing voluntary election to end the relationship. This is the only option that seeks to determine the patient's wishes. (p. 467) 4. The cancellations may have nothing to do with nonadherence. The therapist should notify the physician about nonadherence or when the need to terminate the patient-therapist relationship has been confirmed with the patient. (p. 467)

A patient reports decreased levels of low back pain after receiving physical therapy interventions over a 3-week period. The patient then cancels the last three scheduled appointments. Which of the following actions is MOST appropriate for the physical therapist to take? 1. Leave a message on the patient's answering machine discussing the patient's noncompliance. 2. Document that the patient has discharged self from physical therapy. 3. Talk with the patient to find out the reason for the cancellations. 4. Speak to the referring physician concerning the patient's nonadherence to the plan of care.

Correct Answer: 3 1. Anterior and middle deltoid muscles are not the main muscles involved in pulling the hatch door shut. These muscles would be used in upward movements. 2. Levator scapulae and serratus anterior muscles are not the main muscles involved in pulling the hatch door shut. These muscles would be used in upward movements. 3. The person is pulling the car rear hatch door shut. Forceful downward rotation of the upper limb is necessary to perform the pictured activity. The latissimus dorsi primarily performs this task, with medial (internal) rotation, adduction, and extension of the shoulder. The rhomboids adduct and elevate the scapula, rotating it so the glenoid faces downward. 4. The upper and lower trapezius muscles are not the main muscles involved in pulling the hatch door shut. These muscles would be used in upward movements.

A patient reports difficulty, but no pain, when performing the task in the photograph. Which of the following muscles MOST likely require strengthening in the patient? 1. Anterior and middle deltoid 2. Levator scapulae and serratus anterior 3. Rhomboids and latissimus dorsi 4. Upper and lower trapezius

Correct Answer: 4 1. Restricting fluid intake would result in dehydration, which is not a viable treatment option for incontinence. 2. The time intervals between voiding should be increased to suppress urges. 3. The detrusor muscle should be relaxed or inhibited in patients who have urge incontinence. 4. The patient is describing signs of urge incontinence with possible detrusor contractions. Relaxation training is helpful to decrease bladder contractions.

A patient reports incontinence and a sensation of urgency to urinate with little output. Which of the following interventions is BEST to include in the therapeutic program? 1. Restriction of fluid intake 2. Scheduling an increased frequency of voiding 3. Detrusor contraction exercises 4. Relaxation training

Correct Answer: 1 1. Entrapment of the ulnar nerve as it courses through the hook of the hamate and the pisiform can lead to paresthesia along the ulnar side of the hand into the volar (palmar) aspect in the little finger (5th digit) and medial half of the ring finger (4th digit). It will also lead to weakness of the hypothenar muscles, making it difficult to perform gripping activities, such as opening jars or turning doorknobs. Common etiologies include repetitive gripping as occurs with knitting, tying knots, or using pliers and staplers. (p. 409) 2. Carpal tunnel syndrome is a disorder affecting the median nerve as it courses through the wrist (p. 405). It is characterized by numbness and tingling in the median nerve distribution of the hand in the volar (palmar) aspect of the lateral three and a half digits (p. 388). It is also marked by atrophy of the first two lumbricals and the thenar muscles. Symptoms tend to get worse at night (p. 406). 3. Thoracic outlet syndrome is characterized by symptoms related to entrapment of the neurovascular bundle in the neck and shoulder region and will have more widespread symptoms that are not just localized to the hand (p. 402). 4. Complex regional pain syndrome typically occurs after trauma or injury to the affected extremity and is characterized by hyperesthesia and pain that is out of proportion to the injury. It is also characterized by edema, vasomotor instability, trophic changes, and, in chronic cases, increased fibrosis and synovial proliferation leading to joint restrictions. (p. 410)

A patient reports numbness and tingling in the medial aspect of the hand. The patient reports having used a stapler on 100 packets of paper last week for a presentation. Which of the following syndromes is MOST likely present? 1. Ulnar tunnel 2. Carpal tunnel 3. Thoracic outlet 4. Complex regional pain

Correct Answer: 1 1. The pes anserine is medial and just distal to knee joint line. The semitendinosus and sartorius attach here, would be stretched with extension and valgus, and are involved in knee flexion. (p. 858) 2. The patellar tendon is located on the anterior knee and would not be painful with passive knee extension (p. 857). 3. Medial meniscus injury would be painful with palpation on the joint line (p. 858). 4. The popliteus tendon is located in the posterior knee (p. 858).

A patient reports pain in response to palpation of the anteromedial knee below the joint line. In addition, the patient reports pain with active knee flexion, passive knee extension, and valgus stress. Which of the following structures is the MOST likely source of the pain? 1. Pes anserine 2. Patellar tendon 3. Medial meniscus 4. Popliteus tendon

A patient reports pain in the lateral lower leg with application of a stretch as shown in the photograph. Which of the following muscles should be tested for pain with contraction? 1. Gastrocnemius and soleus 2. Tibialis anterior and extensor hallucis longus 3. Extensor digitorum longus and extensor digitorum brevis 4. Fibularis (peroneus) longus and fibularis (peroneus) brevis Correct Answer: 4 1. The gastrocnemius and soleus are found in the posterior aspect of the leg and perform plantar flexion. A stretch would be in the direction of dorsiflexion, and muscle contraction would cause pain with plantar flexion. 2. The tibialis anterior is in the anterior compartment of the lower leg. The tibialis anterior performs dorsiflexion and inversion and is stretched with plantar flexion and eversion. Pain with muscle contraction would occur with dorsiflexion and inversion. The function of the extensor hallucis longus is extension of the metatarsophalangeal and interphalangeal joints of the great toe; it also has accessory function in ankle dorsiflexion and inversion of the foot. Pain with muscle contraction would occur with dorsiflexion and inversion. 3. The extensor digitorum longus is in the anterior compartment of the lower leg. Contraction in the direction of plantar flexion with toe flexion would cause pain associated with this muscle. The functions of the extensor digitorum brevis are extension of the metatarsophalangeal joint of the great toe and the second through fourth toes; it assists with extension of the interphalangeal joints of the second through fourth toes. The stretch shown in the photograph would not provoke pain in this muscle. 4. Lateral lower leg pain is in the area of the fibularis (peroneal) musculature. These muscles are stretched when moved into dorsiflexion and inversion, which would provoke pain. The fibularis (peroneal) muscles perform plantar flexion and eversion; which, with contraction, would be painful.

A patient reports pain in the lateral lower leg with application of a stretch as shown in the photograph. Which of the following muscles should be tested for pain with contraction? 1. Gastrocnemius and soleus 2. Tibialis anterior and extensor hallucis longus 3. Extensor digitorum longus and extensor digitorum brevis 4. Fibularis (peroneus) longus and fibularis (peroneus) brevis

Correct Answer: 4 1. The apparent diagnosis is complex regional pain syndrome type I, or reflex sympathetic dystrophy (p. 410). Patients who have complex regional pain syndrome often have hypersensitivity to heat and/or cold and, therefore, would not likely tolerate use of thermal modalities such as continuous ultrasound (p. 412). 2. The apparent diagnosis is complex regional pain syndrome type I, or reflex sympathetic dystrophy (p. 410). Non-weight-bearing gait would promote more swelling due to disuse of the muscle pump (p. 412). 3. The apparent diagnosis is complex regional pain syndrome type I, or reflex sympathetic dystrophy (p. 410). Retrograde massage is recommended in complex regional pain syndrome; deep friction massage may increase edema (p. 412). 4. The apparent diagnosis is complex regional pain syndrome type I, or reflex sympathetic dystrophy (p. 410). Range of motion and therapeutic exercises (including weight-bearing exercises) may encourage a decrease in sympathetic tone, enhance circulation, help to maintain range of motion and strength, and increase functional use of the extremity (p. 412).

A patient reports significant hypersensitivity to light touch in the left foot and ankle region following minor trauma. A physical therapist notes warmth, swelling, and redness over the entire foot and ankle. Which of the following interventions is MOST appropriate? 1. Continuous ultrasound 2. Gait training, non-weight-bearing on the left 3. Deep friction massage 4. Progressive weight-bearing activities

Correct Answer: 4 1. The brachialis as an elbow flexor is shortened. The pollicis longus attaches below the elbow and would not be restricted. 2. The brachioradialis flexes the elbow. The cast position shortened this muscle. The triceps would not be short. 3. The biceps brachii flexes the elbow and supinates the forearm. Although the biceps brachii would be shortened, the triceps would not be short. 4. The biceps brachii and brachioradialis flex the elbow and supinate the forearm. The cast position would shorten both muscles.

A patient sustained a nondisplaced midshaft radial and ulnar fracture 12 weeks ago. The patient was casted in mid-range elbow flexion with the forearm in a neutral position. Which of the following muscle pairs would MOST likely demonstrate contractile tissue shortening following the cast removal? 1. Brachialis and flexor pollicis longus 2. Brachioradialis and triceps 3. Biceps brachii and triceps 4. Biceps brachii and brachioradialis

Correct Answer: 2 1. Placing the patient in prone position would take a good deal of planning, would be very staff dependent, and may position the C2-C4 fusion in an undesired position. Cervical fusion requires that the patient avoid bending and twisting the neck, which may occur when placing the patient in prone position. 2. Sidelying is the best position to relieve pressure to the affected areas and will afford the patient a safe position that will not affect the cervical spine fusion or ventilator management. 3. Sitting position is not acceptable because areas of the pelvis and trunk would still be affected by pressure. 4. Supine position causes increased pressure on the sacrum.

A patient underwent a C2-C4 fusion procedure and is ventilator dependent. In what position should the physical therapist position the patient to avoid skin breakdown of the sacrum, ischial tuberosity, scapula, posterior calcaneus, and occipital tuberosity? 1. Prone 2. Sidelying 3. Sitting 4. Supine

Correct Answer: 1 1. Excessive foot pronation during midstance to toe off is the result of a compensated rearfoot (or forefoot) varus deformity. 2. A compensated forefoot valgus deformity would result in excessive foot supination. 3. Uncompensated lateral (external) rotation of the tibia would result in excessive foot supination. 4. Uncompensated pes cavus would result in excessive foot supination.

A patient walks with excessive foot pronation during midstance through toe off (preswing). What is the MOST likely cause of the patient's gait deviation? 1. Compensated rearfoot varus deformity 2. Compensated forefoot valgus deformity 3. Uncompensated lateral (external) rotation of the tibia 4. Uncompensated pes cavus

Correct Answer: 2 1. An increase in the secretion of adrenocorticotropic hormone causes Cushing disease, associated with hypertension, mental changes, weight gain, and increased hair growth, not the symptoms listed (p. 500). 2. A hyperactive thyroid will elevate the body's metabolism, causing an elevated heart rate, fatigue, weight loss, heat intolerance, and muscle atrophy, among other symptoms (p. 484). 3. An increase in insulin will cause a decrease in blood glucose levels, resulting in a hypoglycemic reaction, which includes pallor, increased perspiration, tachycardia, weakness, shakiness, and blurred vision. Fatigue and weight loss are not symptoms of hyperglycemia. (p. 510) 4. Elevation of epinephrine can cause an increase in blood pressure, tachycardia, and hyperglycemia. It causes an increase in the sympathetic response ("fight or flight"). Heat intolerance, fatigue, and weight loss are not consequences of an increase in epinephrine. (p. 473)

A patient who exhibits heat intolerance, tachycardia, fatigue, and weight loss is MOST likely to have an elevated level of which of the following hormones? 1. Adrenocorticotropic hormone 2. Thyroid hormone 3. Insulin 4. Epinephrine

Correct Answer: 2 1. The patient has signs and symptoms consistent with a diagnosis of a lateral ankle (inversion) sprain. A grade 2 lateral ankle sprain is characterized by localized swelling and more diffuse lateral tenderness (Dutton, p. 1151). Gastrocnemius stretching with a strap while in long-sitting position is indicated in the subacute phase of rehabilitation (4-14 days). This exercise stretches the ankle out of the neutral position, which is a goal for the subacute stage (Dutton, p. 1154; Kisner). 2. The patient has signs and symptoms consistent with a diagnosis of a lateral ankle (inversion) sprain. A grade 2 lateral ankle sprain is characterized by localized swelling and more diffuse lateral tenderness (Dutton, p. 1151). Gentle posterior talocrural joint mobilizations should be performed in the acute stage of healing to maintain mobility and inhibit pain. 3. The patient has signs and symptoms consistent with a diagnosis of a lateral ankle (inversion) sprain. A grade 2 lateral ankle sprain is characterized by localized swelling and more diffuse lateral tenderness (Dutton, p. 1151). Stationary cycling is appropriate to address cardiovascular endurance and provide controlled ankle range of motion in the subacute phase of rehabilitation (4-14 days) (Dutton, p. 1154). 4. The patient has signs and symptoms consistent with a diagnosis of a lateral ankle (inversion) sprain. A grade 2 lateral ankle sprain is characterized by localized swelling and more diffuse lateral tenderness (Dutton, p. 1151). Active range of motion exercises, including ankle pumping and toe curls, are indicated only if performed within a pain-free range (Dutton, p. 1154).

A patient who fell while running 3 days ago reports diffuse lateral ankle pain with active movement. The patient exhibits localized swelling distal and anterior to the lateral malleolus. Minimal laxity is noted with an anterior drawer test. Which of the following interventions would be MOST appropriate for the patient at this time? 1. Gastrocnemius-soleus stretching 2. Posterior talocrural joint mobilizations 3. Stationary cycling for up to 30 minutes 4. Active range of motion to end-range

Correct Answer: 3 1. Active range of motion exercises should not be started until 6 to 8 weeks after surgery for a large rotator cuff tear. 2. Supine, active-assisted range of motion exercises of the involved shoulder are appropriate. Seated, active-assisted range of motion exercises would put too much stress on the patient's shoulder because a 5-centimeter tear is considered massive and requires a more conservative approach. 3. Passive range of motion exercises are initiated immediately in rehabilitation. A 5-centimeter tear is considered a massive rotator cuff tear that requires a conservative approach to rehabilitation, but passive range of motion exercises should occur immediately. 4. Isotonic strengthening exercises should not be started until 6 to 8 weeks after surgery for a large rotator cuff tear.

A patient who had a 1.96-inch (5-cm) rotator cuff tear underwent surgical repair 1 week ago. Which of the following interventions is MOST appropriate at this time? 1. Active range of motion 2. Active-assisted range of motion against gravity 3. Passive range of motion 4. Isotonic strengthening

Correct Answer: 2 1. The flexion synergy of the affected upper extremity results in scapular retraction/elevation or hyperextension. The scapula should be mobilized in protraction (not retraction) to preserve the glenohumeral rhythm that prevents soft tissue impingement in the subacromial space during overhead movements of the arm. 2. Flexion synergy of the upper extremity includes scapular retraction/elevation or hyperextension. In the upper extremity, correct passive range of motion techniques require careful attention to lateral (external) rotation and distraction of the humerus, especially as ranges approach 90° of flexion or more. The scapula should be mobilized on the thoracic wall with an emphasis on upward rotation and protraction to prevent soft tissue impingement in the subacromial space during overhead movements of the arm. 3. The scapula should be mobilized on the thoracic wall with an emphasis on upward rotation (not downward rotation) and protraction (not retraction) to prevent soft tissue impingement in the subacromial space during overhead movements of the arm. 4. The scapula should be mobilized in upward rotation (not downward rotation) to preserve the glenohumeral rhythm that prevents soft tissue impingement in the subacromial space during overhead movements of the arm.

A patient who had a cerebrovascular accident exhibits a flexion synergy of the left upper extremity. To promote good upper extremity movement, a physical therapist should mobilize the patient's scapula toward which of the following directions? 1. Upward rotation and retraction 2. Upward rotation and protraction 3. Downward rotation and retraction 4. Downward rotation and protraction

Correct Answer: 3 1. The patient is struggling to show improvements, and the therapist's assessment is that the patient will have deficits that will prevent the patient from becoming more independent with mobility. Continuing treatment for 2 more weeks is not a good use of resources. 2. Recommending transfer back to the hospital is inappropriate because the patient does not show any significant decline warranting a hospital admission. The patient is struggling to show improvements, and the therapist's assessment is that the patient will have deficits that will prevent the patient from becoming more independent with mobility. 3. A revision in the plan of care is indicated if the patient progresses more slowly than expected. Each modification must be evaluated in terms of overall effect on the plan of care. Family and caregiver education/training is a component of effective discharge planning. The caregiver should understand the proper use of any relevant assistive equipment and appropriate transfer and guarding techniques and should use correct body mechanics. 4. The family/caregiver and patient may be at risk for possible injury if not instructed in the proper body mechanics and transfer techniques.

A patient who had a cerebrovascular accident has not progressed with mobility during the past 2 weeks of treatment in a skilled nursing facility. The physical therapist's prognosis is that the patient has residual deficits that will prevent the patient from becoming more independent. The family wants to take the patient home. Which of the following treatment plans is MOST appropriate for the patient? 1. Continue treatment for 2 weeks and then reassess to determine if the additional intervention has resulted in further improvement. 2. Recommend transferring the patient back to the hospital for reassessment for possible extension of the cerebrovascular accident. 3. Change the focus of the treatment to family or caregiver training in assisting the patient to ensure a safe discharge. 4. Discharge the patient to home at the current level of function and have the patient's family monitor the patient for further improvement.

Correct Answer: 1 1. Limiting the knee flexion to 90° for the first 4 to 6 weeks is recommended to limit the shear stress to the healing meniscus. Flexion beyond 90° introduces shear to the meniscus and joint, damaging the repaired and healing meniscus. 2. Full knee flexion is not appropriate because this results in shear forces to the meniscal repair beyond 90°. Flexion beyond 90° introduces shear to the meniscus and joint, which can damage the repaired and healing meniscus. 3. Sit-to-stand transfers from a low chair would cause knee flexion to go beyond 90°. Flexion beyond 90° introduces shear to the meniscus and joint, which can damage the repaired and healing meniscus. 4. Gluteus bridges with knee flexion at 100° is not appropriate because this places shear forces to the meniscal repair beyond 90°. Flexion beyond 90° introduces shear to the meniscus and joint, which can damage the repaired and healing meniscus.

A patient who had a medial meniscus repair 1 week ago has limited knee flexion. Which of the following interventions is MOST appropriate for the patient at this time? 1. Limit knee flexion to 90°. 2. Perform full knee flexion as tolerated. 3. Perform sit-to-stand transfers from a low chair. 4. Perform gluteus bridges with knee flexion at 100°.

Correct Answer: 3 1. Restriction from all sports is not necessary. The patient may participate in low-intensity sports. 2. Full activity without restrictions for a patient who has pulmonary hypertension may put the patient at risk for developing cyanosis, heart failure, or pulmonary hemorrhage. 3. Although exercise tolerance for this patient is likely to be normal or only mildly impaired, intense exertion accompanied by pulmonary hypertension may result in cyanosis, heart failure, or pulmonary hemorrhage. The patient may participate in low-intensity sports. 4. The patient should be encouraged to participate in low-intensity sports. Limiting the patient to only basic activities of daily living is too restrictive.

A patient who had an atrial septal defect repair continues to have mild pulmonary hypertension. Which of the following activity-level recommendations is MOST appropriate? 1. Participation in all sports is restricted. 2. Participation in sports is not restricted. 3. Participation is limited to low-intensity sports. 4. Participation is limited to basic activities of daily living.

Correct Answer: 4 1. Although unilateral pitting edema is a factor contributing to the Wells Clinical Decision Rule for a deep vein thrombosis, presence of a Baker cyst and unilateral varicose veins are not. 2. Varicose veins, bilaterally equal pitting edema, and a painful calf raise are not factors considered in the Wells Clinical Decision Rule for a deep vein thrombosis. 3. None of the listed clinical presentations are factors considered in the Wells Clinical Decision Rule for a deep vein thrombosis. 4. The Wells Clinical Decision Rule for a deep vein thrombosis is a valid and reliable tool in predicting the risk of lower extremity deep vein thrombosis in the orthopedic population. Cancer within 6 months, unilateral swelling greater than 3 cm, and pitting edema are three of the factors in the Wells Clinical Decision Rule, which puts the probability of developing a deep vein thrombosis greater than 75%.

A patient who had knee arthroscopic surgery 2 weeks ago reports a new onset of calf pain 2 days after an intense exercise session. Which of the following findings should MOST raise suspicion of deep vein thrombosis? 1. Baker cyst, unilateral pitting edema, unilateral varicose veins 2. Bilateral varicose veins, bilaterally equal pitting edema, painful calf raise 3. Knee swelling of 1.2 inches (3 cm) compared to the previous session, painful calf raise, relief with recumbency 4. Calf swelling of 1.4 inches (3.5 cm) compared to the contralateral leg, pitting edema, recent history of cancer

Correct Answer: 2 1. Air travel is not contraindicated for individuals who have Stage 0 lymphedema; however, compression garments should be worn during flight to avoid potential swelling due to the decreased air pressure. 2. Individuals who have lymphedema in any stage (Stage 0 or risk for other stages) should protect the skin and avoid any activities that could pierce the skin tissue. 3. Aquatic activities, which include swimming, are recommended. Swimming would not cause skin breakdown and is not contraindicated for patients who have Stage 0 lymphedema. 4. Stationary cycling need not be avoided, although on-road cycling, which could result in falls and skin damage, should be avoided. Lower extremity exercise is recommended, and stationary cycling is a safe lower extremity activity for individuals who have Stage 0 lymphedema.

A patient who has Stage 0 lymphedema should be educated to AVOID which of the following activities? 1. Prolonged air travel 2. Running barefoot 3. Aquatic activities 4. Stationary cycling

Correct Answer: 3 1. A patient who has a C6 spinal cord injury retains full use of the diaphragm but lacks innervation to abdominal and intercostal musculature. The patient will display outward, not inward, motion of the abdomen and inward motion of the upper chest. 2. A patient who has a C6 spinal cord injury retains full use of the diaphragm but lacks innervation to abdominal and intercostal musculature. The patient will display outward, not inward, motion of the abdomen and inward, not outward, motion of the upper chest. 3. A patient who has a C6 spinal cord injury retains full use of the diaphragm but lacks innervation to abdominal and intercostal musculature. The patient will display outward motion of the abdomen and inward motion of the upper chest. The outward motion of the abdominal area is caused by the diaphragm contracting and pushing abdominal contents forward and outward, and the inward motion of the upper chest is due to the lack of structural support from paralyzed thoracic musculature. 4. A patient who has a C6 spinal cord injury retains full use of the diaphragm but lacks innervation to abdominal and intercostal musculature. The patient will display outward motion of the abdomen and inward, not outward, motion of the upper chest.

A patient who has a C6 spinal cord injury (ASIA Impairment Scale A) is MOST likely to exhibit which of the following movement patterns during inhalation? 1. Inward motion of the abdomen and inward motion of the upper chest 2. Inward motion of the abdomen and outward motion of the upper chest 3. Outward motion of the abdomen and inward motion of the upper chest 4. Outward motion of the abdomen and outward motion of the upper chest

Correct Answer: 3 1. Strengthening the neck extensors is not recommended. Stretching of the posterior cervical extensors is recommended for patients who have cervicogenic headaches, which is suggested by the patient's posture, pain distribution, and cervical joint hypomobility. (Dutton, pp. 1232-1233) 2. Strengthening of the upper trapezius is not appropriate because tightness of this muscle is associated with cervicogenic headache (Olson, p. 324). 3. The presence of cervicogenic headaches is suggested by the patient's posture, pain distribution, and cervical joint hypomobility. Decreased strength in the deep cervical cranioflexors is associated with this condition; therefore, strengthening of the deep neck flexors would be appropriate. (Olson, p. 325; Dutton, pp. 1232-1233) 4. Decreased strength in the deep cervical cranioflexors is associated with cervicogenic headache. The sternocleidomastoid muscle is a superficial neck flexor. (Olson, p. 325; Dutton, pp. 1232-1233, 1262)

A patient who has a forward head posture reports right-sided headaches and neck pain. Assessment reveals stiffness of the right occipitoatlantal joint segment. The patient's goal is to decrease headache intensity and to improve cervical function. Which of the following muscles would be MOST appropriate to strengthen? 1. Neck extensors 2. Upper trapezius 3. Deep neck flexors 4. Sternocleidomastoid

Correct Answer: 3 1. Wall sits are performed in an upright position and would not exacerbate a hiatal hernia. 2. An overhead press is typically performed in seated, semireclined, or standing position and would not exacerbate a hiatal hernia. 3. Individuals who have a hiatal hernia should avoid supine position and avoid the Valsalva maneuver. Bilateral leg lifts must be performed in supine position and require strong contractions of the stomach muscles, encouraging the Valsalva maneuver, which would worsen the hiatal hernia. 4. Hamstring stretching can be modified to be done in a position other than supine to avoid exacerbating a hiatal hernia.

A patient who has a hiatal hernia is receiving physical therapy. Which of the following exercises would MOST likely worsen the symptoms related to the hernia? 1. Wall sits 2. Overhead press 3. Bilateral leg lifts 4. Hamstring stretch

Correct Answer: 1 1. Patients who have a history of long-term nonsteroidal antiinflammatory drug use should be monitored for signs and symptoms of bleeding. Pain occurring within 1-3 hours of eating is typical in duodenal ulcers. The pain occasionally radiates to the mid thoracic back and right upper quadrant, including the right shoulder. Right shoulder pain alone may occur as a result of blood within the peritoneal cavity. Melena (dark, tarry stools) and coffee-ground vomitus are indicative of bleeding. Referral to a physician is warranted. (pp. 877-878) 2. Although gastritis is a common adverse effect of nonsteroidal antiinflammatory drugs and is associated with epigastric pain and loss of appetite, pain is much less common than with ulcer disease. Coffee-ground vomitus is not typically associated with gastritis. (pp. 875-876) 3. Irritable bowel syndrome typically presents with prolonged abdominal pain that is relieved by bowel movements. Changes in stool associated with irritable bowel syndrome include hard, loose, or watery stool, rather than melena (dark, tarry stools), alterations in stool frequency, or difficulty in having a bowel movement. Left lower quadrant pain with constipation and diarrhea are commonly associated with this condition. (p. 891) 4. An inflammation of the appendix typically presents with a classic sequence of abdominal pain with vomiting (not coffee-ground vomitus, which is indicative of bleeding), low-grade fever, anorexia, and nausea. Pain associated with appendicitis is constant and may shift within 12 hours to the right lower quadrant at the McBurney point. Melena (dark stools) and pain referred to the right shoulder are not typically associated with appendicitis. (pp. 904-905)

A patient who has a long-term history of nonsteroidal antiinflammatory drug use reports back pain from the mid thoracic region to the right upper quadrant, including the posterior right shoulder. The patient also reports weight loss, loss of appetite, dark-colored stools, and episodes of epigastric pain within 3 hours of eating a meal. The patient reports an episode of vomiting material with a coffee-ground appearance prior to arriving for physical therapy. Which of the following gastrointestinal conditions is MOST likely responsible for these symptoms? 1. Peptic ulcer disease 2. Gastritis 3. Irritable bowel syndrome 4. Appendicitis

Correct Answer: 1 1. In spastic bowel dysfunction, the level of cord injury occurs above S2-S4, leaving the spinal defecation reflexes intact. 2. Spinal cord injuries at or below spinal segments S2-S4 result in flaccid bowel dysfunction, with loss of spinal defecation reflexes. 3. In spastic bowel dysfunction, the level of cord injury occurs above S2-S4, leaving the spinal defecation reflexes intact. 4. Spinal cord injuries at or below spinal segments S2-S4 result in flaccid bowel dysfunction, with loss of spinal defecation reflexes.

A patient who has a spinal cord injury reports having spastic (reflex) bowel function. Which of the following descriptions BEST characterizes the patient's neurologic injury? 1. Injury above spinal segments S2-S4, leaving spinal defecation reflexes intact 2. Injury at or below spinal segments S2-S4, leaving spinal defecation reflexes intact 3. Injury above spinal segments S2-S4, abolishing spinal defecation reflexes 4. Injury at or below spinal segments S2-S4, abolishing spinal defecation reflexes

Correct Answer: 1 1. The patient descriptors align with Stage 5 amyotrophic lateral sclerosis. A physical therapist should educate the family and patient on proper positioning and turning principles to avoid skin breakdown. 2. The patient descriptors align with Stage 5 amyotrophic lateral sclerosis. The patient would be unable to perform ambulation or mobility with or without orthoses due to severe lower extremity weakness. Orthotic support is more appropriate in Stage 2. 3. The patient descriptors align with Stage 5 amyotrophic lateral sclerosis. The patient would be unable to perform manual wheelchair propulsion due to moderate upper extremity weakness. Most patients are introduced to electronic or motorized mobility by Stage 4. 4. The patient descriptors align with Stage 5 amyotrophic lateral sclerosis. Upper extremity strength training is not appropriate in the setting of moderate weakness. Strength training is permissible in muscle groups with Fair plus (3+/5) strength during Stages 2 and 3 with caution to avoid excessive fatigue.

A patient who has amyotrophic lateral sclerosis exhibits severe lower extremity weakness and moderate upper extremity weakness. The patient has been increasingly dependent for activities of daily living. Which of the following interventions is MOST appropriate for the patient? 1. Education in positioning principles 2. Fitting with ankle-foot orthoses 3. Education in manual wheelchair propulsion 4. Strength training of the upper extremities

Correct Answer: 1 1. The piriformis arises from the anterior aspect of S2-S3 and S4 segments of the sacrum and attaches to the upper border of the greater trochanter of the femur. The piriformis primarily functions to produce lateral (external) rotation and abduction of the femur but is also thought to function as a medial (internal) rotator and abductor of the hip if the hip joint is flexed beyond 90°. The piriformis has been implicated as the source for a number of conditions in this area, including entrapment neuropathies of the sciatic nerve, trigger points, and tender points. (pp. 879, 1533) 2. The gluteus minimus is the major medial (internal) rotator of the femur. It also abducts the thigh as well as helps the gluteus medius with pelvic support. The muscle activation described in the scenario is performed by the piriformis muscle. The piriformis primarily functions to produce lateral (external) rotation and abduction of the femur but is also thought to function as a medial (internal) rotator and abductor of the hip if the hip joint is flexed beyond 90°. (pp. 879, 1533) 3. The obturator externus is an adductor and lateral (external) rotator of the hip in neutral (p. 880). The muscle activation described in the scenario is performed by the piriformis muscle. The piriformis primarily functions to produce lateral (external) rotation and abduction of the femur but is also thought to function as a medial (internal) rotator and abductor of the hip if the hip joint is flexed beyond 90° (pp. 879, 1533). 4. The obturator internus is normally a lateral (external) rotator of the hip in neutral and a medial (internal) rotator of the ilium but becomes an abductor of the hip at 90° of hip flexion (p. 879). The muscle activation described in the scenario is performed by the piriformis muscle. The piriformis primarily functions to produce lateral (external) rotation and abduction of the femur but is also thought to function as a medial (internal) rotator and abductor of the hip if the hip joint is flexed beyond 90° (pp. 879, 1533).

A patient who has buttock pain exhibits tenderness to palpation in the mid gluteal area. Pain is reproduced with resisted hip lateral (external) rotation when the hip is in the anatomical position and with resisted hip medial (internal) rotation when the hip is flexed greater than 90°. Which of the following muscles is MOST likely involved? 1. Piriformis 2. Gluteus minimus 3. Obturator externus 4. Obturator internus

Correct Answer: 2 1. This is a normal response in a patient who has chronic obstructive pulmonary disease and as such would not require discontinuation of the exercise session. Since elevated heart rates with increased hypoxemia are an expected compensatory response in patients who have pulmonary disease, there is no reason to cease exercise unless some other type of medical emergency exists. 2. During acute exercise, patients who have chronic obstructive pulmonary disease experience elevated heart rates and blood pressures with incremental exercise. The patient's response is an expected compensatory response, and exercise can continue as prescribed with continued monitoring of vital signs. 3. Since elevated heart rates with increased hypoxemia are an expected compensatory response in patients who have pulmonary disease, there is no reason to refer the patient back to the physician unless there is some other type of medical emergency. 4. This is an expected compensatory response in a patient who has chronic obstructive pulmonary disease and as such would not require discontinuation of the exercise session. There is no reason to refer the patient to the emergency department unless there is some other type of medical emergency.

A patient who has chronic obstructive pulmonary disease is participating in a mild graded exercise program at a level of 2 metabolic equivalents (METs). The patient's heart rate at rest is 80 bpm. During an incremental increase in exercise up to 4 metabolic equivalents (METs), the patient experiences an elevation in heart rate to 120 bpm. Which of the following actions is MOST appropriate? 1. Discontinue exercise while monitoring vital signs. 2. Continue the exercise session while monitoring vital signs. 3. Continue the exercise session and refer the patient to a physician to evaluate exercise response. 4. Discontinue the exercise session and refer the patient to an emergency department to evaluate exercise response.

Correct Answer: 1 1. The cognitive stage is the beginning of the learning process. Cues, instructions, and guidance are provided by the therapist, and demonstration is used. 2. Performing a task with little cognitive attention at variable speeds is characteristic of the autonomous stage of learning. 3. Problem-solving independent from the therapist's feedback is characteristic of the associative stage of learning. 4. Exploration of different strategies with little or no input from the therapist is characteristic of the associative stage of learning.

A patient who has hemiparesis is learning to propel a manual wheelchair. Which of the following interventions is MOST appropriate for the cognitive stage of learning this task? 1. The physical therapist guides the patient with hand-over-hand cues and demonstrates the propulsion technique. 2. The patient propels the wheelchair with variable speed through an obstacle course with supervision from the physical therapist. 3. The physical therapist allows the patient to problem-solve when steering errors occur and does not provide feedback. 4. The physical therapist allows the patient to independently explore strategies for propulsion and steering during a specific propulsion task.

Correct Answer: 3 1. Resistance training has been shown to be beneficial for patients who have primary progressive multiple sclerosis (p. 690). 2. Exercising to the point of fatigue is contraindicated and can result in worsening of symptoms, most notably increasing weakness (p. 689). 3. Patients who have primary progressive multiple sclerosis should exercise when the core body temperatures is the lowest and before fatigue sets in. Most commonly, this is in the morning hours. (p. 690) 4. Heart rate responses may be attenuated during exercise. A category-ratio rating of perceived exertion scale can be used to estimate central and peripheral exertion. Heart rate may be difficult to monitor due to dysautonomia; sensory loss in the fingers may make self-monitoring difficult. (pp. 690-691).

A patient who has primary progressive multiple sclerosis requests guidelines for an exercise plan. Which of the following recommendations would be MOST appropriate for the patient? 1. Resistance training should be avoided. 2. Exercise should be performed to the point of fatigue. 3. Exercise sessions should be scheduled for the morning. 4. Measuring heart rate is the best way to monitor exercise intensity.

Correct Answer: 2 1. Unilateral gaze preference is associated with unilateral neglect and results from the lesion itself, not patient positioning. A patient with visual unilateral neglect often avoids crossing the midline visually (p. 1198). This condition is much more common in patients who have left hemiplegia than in patients who have right hemiplegia (p. 1198). 2. Hemiplegic shoulder pain is a common complication after stroke. Poor positioning of the more affected upper extremity has been implicated in producing joint microtrauma and pain. Prolonged soft tissue injury can result in complex regional pain syndrome. (pp. 646-647) 3. Sidelying on the right side should produce elongation of the trunk on the right, not shortening of the trunk. 4. Risk factors for skin breakdown include decreased sensation and abnormal patterns of movement, neither of which should be present on the less involved (left) side in this case (pp. 622-623).

A patient who has right hemiparesis following a cerebrovascular accident is habitually positioned in right sidelying position. Which of the following problems may result from this positioning and should be of GREATEST concern to the physical therapist? 1. Left gaze preference 2. Chronic right shoulder pain 3. Trunk shortening on the right 4. Skin breakdown on the medial aspect of left knee

Correct Answer: 1 1. Right shoulder pain, bruising, palmar erythema, and confusion are all among the signs and symptoms of liver disease (p. 341). 2. Right shoulder pain, bruising, palmar erythema, and confusion are all among the signs and symptoms of liver disease (p. 341). Pancreatic pain is more likely to refer to the left, not right, shoulder (p. 329). 3. Right shoulder pain, bruising, palmar erythema, and confusion are all among the signs and symptoms of liver disease (p. 341). Renal pain may be referred to the shoulder; however, it is more commonly felt in the posterior subcostal region (p. 377). Associated symptoms include blood in the urine and fever/chills (p. 378). 4. Right shoulder pain, bruising, palmar erythema, and confusion are all among the signs and symptoms of liver disease (p. 341). Splenomegaly may occur as a result of chronic active hepatitis; however, a pathological condition of the spleen is not the primary cause of the other symptoms described (p. 345).

A patient who has right shoulder pain exhibits bruising, palmar erythema, and signs of confusion. Which of the following organs is MOST likely involved? 1. Liver 2. Pancreas 3. Kidney 4. Spleen

Correct Answer: 4 1. Variations of tapotement consist of brisk percussive movements with the goal of increasing alertness or stimulating airway clearance, not improving scar mobility (Benjamin, pp. 279-280). 2. Effleurage is a slide or glide over the skin with continuous motion and light to moderate pressure in order to increase relaxation as well as venous and lymphatic drainage, not to improve scar mobility (Dutton, p. 425). 3. Pétrissage involves compression of soft tissue structures by means of kneading, wringing, rolling, and picking up techniques to release areas of muscle fibrosis, not improve scar mobility (Dutton, p. 425). 4. Friction involves direct circular or cross-fiber massage applied to increase mobility of scar tissue (Dutton, p. 422; Benjamin, p. 279).

A patient who is 6 weeks post surgery for a patellar fracture has a well-healed scar with poor mobility. Which of the following massage techniques is BEST for addressing this problem? 1. Tapotement 2. Effleurage 3. Pétrissage 4. Friction

Correct Answer: 4 1. Exercise should be avoided during the peak activity time of insulin, because insulin is absorbed much faster with exercise, thus altering its effectiveness. 2. With greater effects of insulin associated with exercise, food intake should occur to avoid hypoglycemia. 3. It is important that the person avoid insulin injections within 1 hour of exercise because insulin is absorbed much more quickly in active extremities. This can cause blood glucose levels to drop. 4. Exercise typically increases insulin sensitivity and enhances the effect of insulin. Therefore, glucose intake should be increased to counter the effects of exercise. Insulin intake could be decreased to counter the effects of exercise. During prolonged activity, a snack is recommended for every 30 minutes of activity.

A patient with type 1 diabetes is planning to begin an exercise program. Which of the following actions is MOST appropriate for the patient to perform? 1. Inject insulin just prior to starting an exercise session. 2. Avoid food consumption just prior to an exercise session. 3. Complete an exercise session within 1 hour of receiving an insulin injection. 4. Increase food intake prior to an exercise session.

Correct Answer: 2 1. Most activities of daily living are performed at 50° of pronation and supination. Pronation and supination limited to 65° on the right would not delay return to work. (Magee, pp. 451-452) 2. One of the complications after an upper extremity trauma is the advent of complex regional pain syndrome. This complication often presents with burning pain with any movement of the body part, excessive sensitivity to light touch or minor stimulation, temperature changes, localized sweating, localized changes of the skin, or trophic changes of the skin, hair, and nails. Of the four options, this complication would result in longest delay in recovery and return to work. (Dutton) 3. The functional position of the wrist is between 20° and 35° of wrist extension, which this patient has (Magee, p. 445). The patient's pain is at end-range of wrist motion, and the patient should not have to push the wrist to end-range for most functional activities. 4. The grip strength on the right is 90% of the grip strength on the left. Although the right hand remains 10% weaker, this is an adequate strength for activities of daily living. (Magee, p. 455)

A patient who is a secretary has a well-healed fracture of the right scaphoid. The findings upon the initial physical therapy examination include 55° of wrist flexion and 45° of wrist extension with pain at end-range. Which of the following additional findings would result in the GREATEST delay in return to work? 1. Passive pronation and supination limited to 65° on the right 2. Pain with light touch and increased sweating of the right hand 3. Subjective pain of 3/10 with right wrist movement and 1/10 at rest 4. Grip dynamometer strength of 72 lb (32.7 kg) on the right and 80 lb (36.3 kg) on the left

Correct Answer: 4 1. A deep, throbbing pain is more suggestive of a systemic cause of joint pain (pp. 104, 114). Musculoskeletal pain is more often a unilateral, achy pain (p. 104). 2. Systemic pain is constant and often awakens the patient at night (p. 104). 3. Symptoms that are unrelieved by rest or change in position are characteristic of systemic pain, not musculoskeletal pain (p. 104). 4. Musculoskeletal pain is most likely intermittent and dependent on activity or position. Certain movements will aggravate the symptoms. (p. 104)

A patient who is experiencing knee pain originating from the musculoskeletal system is MOST likely to make which of the following subjective reports? 1. Throbbing pain is present. 2. Pain at night interrupts sleep. 3. Pain is the same with rest and activity. 4. Active motion provokes pain.

Correct Answer: 3 1. Independence with transfers would decrease the risk of skin breakdown due to friction compared to the risk present from tissue loading (p. 145). 2. While the presence of infection may cause a wound to worsen, the infection is unlikely to be the causative factor for wound development. Since development of infection is dependent on the ability of microorganisms to attach to the host's body, the likelihood that the attachment would occur without a pre-existing wound bed is less than the risk of wound development from tissue loading. Since the patient is not incontinent, risk of infection from urine is minimized. (pp. 145, 147) 3. In this patient, prolonged sitting due to the inability to walk leads to tissue loading and risk for skin breakdown on the ischial tuberosities (p. 145). 4. While maceration may contribute, patients who are nonambulatory are generally at risk for having issues with skin maceration in areas where skin is exposed to prolonged contact with urine or stools. Since the patient is independent with catheterization, the patient is unlikely to be experiencing prolonged contact with soiled clothing. (pp. 145-146).

A patient who is not able to walk has developed an ischial tuberosity pressure injury. The patient is able to perform independent intermittent catheterization and demonstrates independence in bed-to-chair transfers. Which of the following factors has MOST likely contributed to the formation of the pressure injury? 1. Friction 2. Infection 3. Tissue loading 4. Tissue maceration

Correct Answer: 2 1. The physical therapist should conduct further measures to rule in or out signs and symptoms that may show if the patient's problem can be addressed within the scope of practice. Results of further measures can also provide the physician with additional clinical information. (p. 3) 2. In view of the symptoms presented in the case, the therapist should screen for appendicitis by checking for the presence of a low fever as well as palpating the abdomen for tenderness (pp. 319-320). 3. Frequently, appendicitis pain is aggravated by movement. The patient tends to lie down, drawing legs up to relieve the pain. Strength testing the abdominal and hip muscles may exacerbate pain unnecessarily. (p. 319) 4. Early detection of a medical problem is key to preventing further medical complications. In this case, it would be more important for the therapist to screen for an acute condition than to provide nutrition education. (p. 3)

A patient who is overweight reports pain and tenderness in the right lower quadrant of the abdomen that worsens upon coughing or sneezing. Which of the following actions should a physical therapist perform FIRST? 1. Defer examination and immediately contact the physician. 2. Measure the patient's body temperature and check for rebound tenderness. 3. Strength test the abdominal and right hip muscles. 4. Provide the patient with educational materials on dietary modification.

Correct Answer: 3 1. Carpal tunnel syndrome is compression of the median nerve as it enters the hand. A positive result on the Phalen test might be present, and compression would be associated with weakness of the lumbricals. The flexor carpi radialis is innervated above the wrist and would not be weakened. (p. 483) 2. The cubital tunnel in the elbow region is a common site of entrapment neuropathy, but compression of the ulnar nerve would occur with entrapment at the cubital tunnel (p. 421). The weak muscles listed in the stem are innervated by the median nerve. 3. The flexor carpi radialis, flexor digitorum superficialis, and flexor pollicis longus and brevis are innervated by the median nerve (p. 413). The pronator teres can compress the median nerve as it passes through this muscle. Repetitive movements may be one cause of enlargement and entrapment. Symptoms would include weakness in the distribution of the median nerve distal to the pronator teres, affecting the wrist and hand (p. 413). However, because the entrapment is superior to the wrist rather than at the wrist, the result of the Phalen test would not likely be positive. 4. Compression of the median nerve that occurs with thoracic outlet syndrome is more proximal than with compression by the pronator teres and thereby affects the entire nerve distribution. However, symptoms are usually exacerbated with head movements, and weakness would be identified in all of the muscles and sensory distributions. (p. 261)

A patient who performs repetitive manual labor reports difficulty working with a screwdriver. A physical therapist identifies weakness of the flexor carpi radialis, flexor digitorum superficialis, and flexor pollicis longus and brevis. The patient has a negative result on the Phalen test. Which of the following locations is the MOST likely site of nerve entrapment? 1. Carpal tunnel 2. Cubital tunnel 3. Pronator teres 4. Thoracic outlet

Correct Answer: 4 1. While liver cirrhosis may present with pallor of the skin (Goodman, Differential Diagnosis, p. 341), hyperbilirubinemia more commonly presents with jaundice or yellowing of the eye sclera and in more advanced cases, of the skin. It is often the first and only clinical manifestation of liver disease. (Goodman, Differential Diagnosis, p. 337) 2. Hyperbilirubinemia leads to an increased excretion of bilirubin by the kidney and hence darker urine (Goodman, Pathology; Goodman, Differential Diagnosis, pp. 340-341). 3. Darkened stools would not occur because liver diseases impair the removal of bilirubin from the blood (hyperbilirubinemia) and reduce the amount of conjugated bilirubin (the compound that darkens fecal material) excreted by the liver into the gut. This set of events leads to lightening, not darkening, of the fecal material (Goodman, Pathology; Goodman, Differential Diagnosis, pp. 340-341). 4. The liver removes bilirubin from the blood, conjugates it, and excretes this complex into the intestine. It is this conjugated bilirubin that darkens fecal material. Liver disease that limits bilirubin transport into the liver increases serum bilirubin (hyperbilirubinemia) and limits the amount of conjugated bilirubin entering the gut. Stools become light in color. This sign helps to distinguish jaundice caused by liver disease from jaundice caused by extrahepatic causes. (Goodman, Pathology; Goodman, Differential Diagnosis, pp. 340-341)

A patient who reports a history of liver damage exhibits signs of hyperbilirubinemia. Which of the following clinical manifestations is MOST likely present? 1. Pallor 2. Clear urine 3. Darkened stools 4. Light-colored stools

Correct Answer: 2 1. Although pain inhibition causes some cervicothoracic muscles to adaptively shorten after a whiplash injury, stretching them will not eliminate the cause of the hypertonicity. Stretching may give temporary relief, but the patient needs to change movement patterns and improve proprioception for long-term gains. 2. For patients in the subacute phase of healing after a whiplash injury, cervical proprioception exercises along with deep neck flexor strengthening are recommended. 3. At 10 weeks after injury, the patient should be performing an active exercise program. Use of modalities for pain relief should be limited to the acute phase of healing. 4. Following a whiplash injury, there is a change in muscle fiber type from type I slow-twitch to type II fast-twitch in the deep cervical flexors. As the slow-twitch muscles function in the stabilization of the cervical spine, exercises to increase the endurance of the type I muscle fibers should be emphasized.

A patient who was in a motor vehicle accident 10 weeks ago sustained a whiplash injury. To facilitate long-term return to function, which of the following interventions would be MOST appropriate for the patient? 1. Levator scapulae stretching 2. Cervical proprioception exercises 3. Continuous ultrasound to the suboccipitals 4. Exercises for selective recruitment of type II muscle fibers

Correct Answer: 1 1. Myofascial tightness in the posterior structures is a common occurrence with chronic whiplash injuries (Dutton, p. 1320). 2. Although cervical facet joint pain is common after a motor vehicular collision, the exact cause, whether because of osteophytic spurring or another mechanism, remains unknown (Dutton, pp. 1320-1321). 3. Muscles that are short and tight are typically overactive, not weak. Weakness typically occurs in the deep posterior cervicothoracic musculature. (Giangarra, p. 482). 4. Patients who sustain a whiplash injury typically exhibit decreased cervical proprioception and sensorimotor deficits (Giangarra, p. 483).

A patient whose car was hit from behind developed superficial cervical paraspinal muscle guarding. Three months post injury, the patient continues to demonstrate muscle guarding, although the pain has diminished considerably. As a result of 3 months of muscle guarding, which of the following consequences is MOST likely to occur? 1. Adaptive shortening of the involved myofascia 2. Osteophytic spurring of the facet joints 3. Weakening of the muscles involved with the muscle guarding 4. Increased proprioceptive awareness in the cervical region

Correct Answer: 3 1. A second-degree prolapse is marked by the cervix as part of the uterus having descended through the introitus, or vaginal opening. Pelvic floor rehabilitation has become the recommended first course of treatment and should include a discussion of alternative positions for sexual intercourse. Additional functional questions should include those related to bladder and bowel habits. Questions about usual sitting patterns would not be most important to ask. 2. A second-degree prolapse is marked by the cervix as part of the uterus having descended through the introitus, or vaginal opening. Pelvic floor rehabilitation has become the recommended first course of treatment and should include a discussion of alternative positions for sexual intercourse. Additional functional questions should include those related to bladder and bowel habits. Questions about usual sleeping patterns would not be most important to ask. 3. A second-degree prolapse is marked by the cervix as part of the uterus having descended through the introitus, or vaginal opening. Pelvic floor rehabilitation has become the recommended first course of treatment and should include a discussion of alternative positions for sexual intercourse. Symptoms may be exacerbated by prolonged standing, walking, coughing, or straining (i.e., for a bowel movement). Urinary incontinence is also a common problem as a result of uterine prolapse. Additional functional questions should include those related to bladder and bowel habits. 4. A second-degree prolapse is marked by the cervix as part of the uterus having descended through the introitus, or vaginal opening. Pelvic floor rehabilitation has become the recommended first course of treatment and should include a discussion of alternative positions for sexual intercourse. Additional functional questions should include those related to bladder and bowel habits. Personal hygiene is a broad category and does not relate exclusively to the presence of a prolapse. Questions related to bowel and bladder habits would be more comprehensive but are not included as options.

A patient with a medical diagnosis of second-degree uterine prolapse is referred to a physical therapist for exercise. For the therapist to evaluate this case fully, questions about which of the following patient functional activities are MOST important to ask? 1. Sitting 2. Sleeping 3. Sexual activity 4. Personal hygiene

Correct Answer: 3 1. Buoyancy is the upward force equal to the volume of water displacement. Buoyancy provides the patient with relative weightlessness and joint unloading by reducing the force of gravity on the body. It does not affect blood flow. (p. 297) 2. Buoyancy is the upward force equal to the volume of water displacement. Buoyancy provides the patient with relative weightlessness and joint unloading by reducing the force of gravity on the body. It does not affect blood flow. (p. 297) 3. Increased hydrostatic pressure centralizes peripheral blood flow and increases venous return (p. 298). 4. Increasing, not decreasing, hydrostatic pressure increases blood pressure via centralizing blood flow and increasing venous return. Decreasing hydrostatic pressure does not increase the blood pressure. (p. 298)

A patient with hypertension has been referred for aquatic physical therapy following a total hip arthroplasty. Which of the following statements is the BEST reason for altering this plan of care? 1. The increased buoyancy of the water may increase the patient's blood pressure. 2. The decreased buoyancy of the water may decrease the patient's blood pressure. 3. The increased hydrostatic pressure of the water may increase the patient's blood pressure. 4. The decreased hydrostatic pressure of the water may increase the patient's blood pressure.

Correct Answer: 3 1. There is no evidence of a change in ventilation-perfusion ratio with pursed-lip breathing. 2. There is no evidence of a change in partial pressure of arterial oxygen (PaO2) with pursed-lip breathing. 3. The increase in exhalation time creates a decrease in respiratory rate. 4. There is no evidence of a change in strength of the ventilatory muscles with pursed-lip breathing.

A physical therapist is teaching a patient pursed-lip breathing. This intervention will MOST likely result in which of the following changes? 1. Decreased ventilation-perfusion ratio 2. Increased partial pressure of arterial oxygen (PaO2) 3. Decreased respiratory rate 4. Increased strength of the ventilatory muscles

Correct Answer: 2 1. The photograph depicts an anterior glide of the humerus, which helps improve glenohumeral extension, according to the concave-convex rule. Hanging clothes in a closet is a flexion-based task. Flexion is best improved through posterior gliding of the humerus. 2. The photograph depicts an anterior glide of the humerus, which helps improve glenohumeral extension, according to the concave-convex rule. Pulling a lawn mower cord is a movement into glenohumeral extension. 3. The photograph depicts an anterior glide of the humerus, which helps improve glenohumeral extension, according to the concave-convex rule. Reaching across the body describes horizontal adduction. Posterior gliding of the humerus can help improve this motion. 4. The photograph depicts an anterior glide of the humerus, which helps improve glenohumeral extension, according to the concave-convex rule. Bringing the arm out to the side and over the head is an abduction task that is best improved with inferior or caudal glenohumeral glides.

A physical therapist is MOST likely to perform the intervention shown in the photograph for a patient who has difficulty with which of the following functional activities? 1. Hanging clothes in a closet 2. Pulling the cord to start a lawn mower 3. Reaching across the body to scrub the back of the opposite shoulder 4. Bringing the arm out to the side and over the head

Correct Answer: 2 1. Vancomycin-resistant Enterococcus (VRE) requires contact precautions. Contact precautions require a gown. A mask is not needed unless droplet precautions are necessary. 2. Contact precautions are followed for vancomycin-resistant Enterococcus (VRE). A gown is needed for contact precautions. 3. Vancomycin-resistant Enterococcus (VRE) requires contact precautions. A gown is needed for contact precautions. 4. A gown is appropriate; however, the therapist should follow contact precautions, not droplet precautions.

A physical therapist is assisting with bed mobility for a patient who is receiving antibiotics for vancomycin-resistant Enterococcus (VRE). Which of the following precautions and personal protective equipment are indicated for physical therapy intervention? 1. Contact precautions; the therapist should wear a mask. 2. Contact precautions; the therapist should wear a gown. 3. Droplet precautions; the therapist should wear a mask. 4. Droplet precautions; the therapist should wear a gown.

Correct Answer: 3 1. Cross-sectional research involves studying a group at one point in time and generalizing the results to a population (p. 280). This is not the method described in the stem. 2. Correlational research is conducted for the purpose of determining the interrelationships among variables (pp. 281-282). This is not consistent with the stem. 3. Meta-analytical research is a process by which the results of several studies are synthesized in a quantitative way (pp. 357-358). This is consistent with the stem. 4. Methodological research is conducted to determine the reliability and validity of clinical and research measurements (p. 290). This is not consistent with the stem.

A physical therapist is conducting a research project that synthesizes the results of several studies in a quantitative process. This process represents which of the following research methods? 1. Cross-sectional 2. Correlational 3. Meta-analytical 4. Methodological

Correct Answer: 2 1. In the management of chronic low back pain, stretching, strengthening, and mobility should be promoted (p. 1480). 2. Aerobic exercise combined with specific strengthening may decrease the frequency of low back pain recurrence (p. 1480). 3. Traction is utilized in the acute phase for patients who have signs of nerve root impingement and for whom symptoms are not centralized with any lumbar movement (p. 1204). 4. Bracing is used for temporary pain relief during the acute phase of nerve root impingement (p. 1479).

A physical therapist is designing an independent home program for a patient who has a 10-year history of recurrent low back pain. The goal of the program is to reduce the recurrence rate and improve the patient's function. Which of the following recommendations is MOST appropriate? 1. Rest whenever pain is increased. 2. Start an aerobic conditioning program. 3. Place an order for a home traction unit. 4. Use a lumbar brace regularly.

Correct Answer: 2 1. Pes cavus is a high-arched foot and is not consistent with the description in the stem (p. 307). 2. Clubfoot (talipes equinovarus) is a congenital deformity (environmental and genetic) that includes components of forefoot adductus, hindfoot varus, and ankle equinus. The primary distinguishing factor is the equinus component. (p. 307) 3. Calcaneovalgus is an intrauterine "packaging" deformity. The ankle is in excessive dorsiflexion, the forefoot is curved out laterally, and the hindfoot is in valgus. (p. 304) 4. Metatarsus adductus is an intrauterine "packaging" deformity. The hindfoot is in valgus, and the forefoot is in varus. (p. 303)

A physical therapist is evaluating an infant who has bilateral ankle equinus, hindfoot varus, and forefoot adductus. The infant MOST likely has which of the following deformities? 1. Pes cavus 2. Talipes equinovarus 3. Calcaneovalgus 4. Metatarsus adductus

Correct Answer: 4

A physical therapist is testing the strength of a patient's middle deltoid. The patient is sitting with the arm at the side and the palm facing the trunk. When asked to move the arm out to the side, the patient turns the palm forward and brings the arm to 90° of abduction. What should the therapist do NEXT? 1. Apply resistance at the end of the movement. 2. Have the patient lie in supine position and repeat the movement. 3. Ask the patient to continue moving the arm into maximum abduction. 4. Have the patient repeat the movement, keeping the palm facing the trunk.

Correct Answer: 2 1. Requesting that the physician determine if urinary tract infection has reoccurred would be unnecessary because the frequency of urination is normal. 2. It is normal to urinate 6 to 8 times in a 24-hour period. A frequency beyond 8 times/day is considered abnormal. 3. Recommending reduced fluid intake would be inappropriate because the decreased fluid intake can increase urine concentration and irritation, resulting in increased frequency. Also, the patient's report of urinary frequency indicates a normal finding. 4. Coffee, alcohol, and nicotine are all bladder irritants and could increase frequency of urination, but the patient's frequency of urination is normal, so this advice is not pertinent to the findings.

A physical therapist is examining a patient who had a urinary tract infection that was successfully treated with antibiotics. The patient reveals a need to urinate 6 times/day. The patient reports normal fluid intake. Which of the following actions should the therapist take? 1. Request that the physician determine if urinary tract infection has reoccurred. 2. Document that the patient's report indicates normal function of the urinary system. 3. Recommend reducing fluid intake to determine if urinary frequency diminishes. 4. Advise the patient to avoid caffeine, alcohol, and nicotine.

Correct Answer: 2 1. Axial extension or chin retraction may assess the mobility of the upper cervical spine but not the function of the deep neck flexors. Axial extension causes excessive shear in the mid-cervical spine. 2. The longus colli and longus capitis are deep neck flexors. The craniocervical flexion test or the deep neck flexor endurance test is included in the examination of these muscles. 3. The cervical compression test is used to stress the ability of the neck to tolerate passive loading; it is not a test of muscle strength. 4. Neck flexion range of motion is not specific for the deep neck flexors and can be achieved with gravity assist (in sitting position) or with the superficial neck flexors such as the sternocleidomastoid.

A physical therapist is examining a patient who has a whiplash injury and a mid-cervical spine sprain. To determine the function of the patient's longus colli and longus capitis, which of the following assessments should be included in the examination? 1. Axial extension 2. Craniocervical flexion 3. Cervical compression test 4. Neck flexion range of motion

Correct Answer: 3 1. A normal blood pressure response to exercise would be an increase in systolic pressure of 20 mm Hg or more with diastolic pressure remaining the same or decreasing slightly. A decrease in systolic blood pressure from 140 to 120 mm Hg is abnormal. 2. Systolic blood pressure is expected to increase with exercise. Systolic pressure that remains the same is an abnormal response to exercise. 3. This is a normal response to exercise because systolic blood pressure increased and diastolic blood pressure stayed the same. A change to 170/80 mm Hg is a normal response to exercise since the systolic pressure increased and the diastolic pressure remained the same. 4. This is an abnormal response to exercise because diastolic blood pressure increased, when it is expected to stay the same or slightly decrease. An increase in the diastolic pressure is an abnormal response to exercise.

A physical therapist is monitoring a patient's blood pressure while the patient exercises on a stationary bike. Prior to exercise, the patient's blood pressure was 140/80 mm Hg. After the patient exercises for 10 minutes at a moderate intensity, which of the following blood pressure values represents a normal response? 1. 120/60 mm Hg 2. 140/80 mm Hg 3. 170/80 mm Hg 4. 180/100 mm Hg

Correct Answer: 1 1. A white blood cell count of 16,000/mm3 exceeds the normal range of 4,500 to 11,000/mm3 (Hillegass, p. 268). This elevated count suggests infection, which may compromise exercise tolerances (Goodman). 2. Normal response is for the oxygen saturation level to remain in the normal range of 98% to 100%. If the oxygen saturation level falls below 90% with activity, the exercise intensity should be decreased and caution used. (Hillegass, p. 558) 3. A hemoglobin level of 15 g/dL is within the normal range for males and females (Hillegass, p. 268). 4. Partial pressure of arterial oxygen (PaO2) of 80 mm Hg and partial pressure of arterial carbon dioxide (PaCO2) of 40 mm Hg are within the normal ranges (Hillegass, p. 354).

A physical therapist is preparing to treat a patient in an acute care setting following a total hip arthroplasty. The patient's past medical history includes chronic obstructive pulmonary disease. Which of the following concomitant conditions MOST indicates that the therapist should defer treatment? 1. White blood cell count of 16,000/mm3 2. Oxygen saturation level of 92% with activity 3. Postoperative hemoglobin level of 15 g/dL (compared with a preoperative level of 18 g/dL) 4. Partial pressure of arterial oxygen (PaO2) of 80 mm Hg and partial pressure of arterial carbon dioxide (PaCO2) of 40 mm Hg

Correct Answer: 4 1. Anemia would be diagnosed from iron and hemoglobin levels. 2. Leukocytosis is a total white blood cell count of greater than 11,000-15,000/mm3 (above normal range). 3. Immunosuppression causes leukopenia, which is a white blood cell count less than 4000/mm3. 4. The patient's white blood cell count is within the normal range of 4500-11,000/mm3, so the infection has resolved.

A physical therapist is reviewing the laboratory report of a patient who received a diagnosis of pneumonia 2 weeks ago. The patient's white blood cell count is currently 9,000 cells/mm3. Which of the following conditions does this value indicate for the patient? 1. Anemia 2. Development of leukocytosis 3. Immunosuppression 4. Resolution of the pneumonia infection

Correct Answer: 2 1. A physician's order for a Doppler study indicates possible deep vein thrombosis. A complete physical therapy evaluation and treatment should be deferred until a deep vein thrombosis has been ruled out or therapeutic levels of a prescribed anticoagulant to treat a deep vein thrombosis have been reached. 2. A physician's order for a Doppler study indicates possible deep vein thrombosis. Physical therapy should not be conducted until the Doppler study is completed and the results analyzed by the physician. 3. A physician's order for a Doppler study indicates possible deep vein thrombosis. Transfer from bed to chair is contraindicated due to possible deep vein thrombosis. 4. The nurse alone should not be providing clearance; the Doppler study must be completed and the results interpreted by the physician.

A physical therapist is reviewing the medical record of a patient in the intensive care unit. The patient was admitted the previous night through the emergency department after a motorcycle accident resulting in a fractured right femur. The therapist notes a physician's order for a Doppler study of the left leg. The therapist should: 1. proceed with the evaluation and intervention without any restrictions. 2. withhold physical therapy until results of the study are obtained and interpreted by the physician. 3. proceed with the evaluation and limit intervention to transfer to a bedside chair. 4. obtain clearance from the nurse to provide intervention for the patient.

Correct Answer: 1 1. Direct current is indicated with a maximum safe amplitude of 4 milliamperes. 2. Direct current is indicated, but an amplitude of 8-10 milliamperes is too high. 3. Pulsed current is not the correct type of electrical current to use with iontophoresis. Direct current should be used. 4. Pulsed current is not the correct type of electrical current to use with iontophoresis. Direct current should be used. The amplitude of 8-10 milliamperes is too high.

A physical therapist is treating a patient with bicipital tendonitis. The therapist has determined that iontophoresis with medication for a total treatment dosage of 80 milliampere-minutes is most appropriate. Which of the following current parameters should a physical therapist use when applying the iontophoresis to achieve the BEST results? 1. 3-4 milliamperes, direct current 2. 8-10 milliamperes, direct current 3. 3-4 milliamperes, pulsed current 4. 8-10 milliamperes, pulsed current

Correct Answer: 4 1. Wearing the harness during rest only is inadequate. It should be worn 18-23 hours/day. 2. Wearing the harness during activity only is inadequate. It should be worn 18-23 hours/day. 3. Wearing the harness for 2-4 hours per day is inadequate. It should be worn 18-23 hours/day. 4. The harness must be worn 18-23 hours/day.

A physical therapist is treating an infant who has a Pavlik harness for developmental dysplasia of the hip. The infant should wear the harness: 1. during rest only. 2. during activity only. 3. 2-4 hours/day. 4. 18-23 hours/day.

Correct Answer: 1 1. Advanced ankylosing spondylitis would cause loss of chest wall excursion, which compromises breathing. Aerobic exercise done consistently would be most important in order to optimize efficiency of oxygen transport and maintain cardiopulmonary function. (pp. 1333, 1337) 2. Balance exercises would likely be used for a patient who has ankylosing spondylitis, but they would not be as important as maintaining cardiopulmonary function in those who have advanced stages of this disease. In addition, activities requiring high levels of balance might need to be avoided to reduce risk of falls. (p. 1337) 3. Stretching and aerobic exercise would be more important than light resistance exercise for a patient who has advanced ankylosing spondylitis and likely fusion of involved joints. Energy conservation would also take precedence over resistance exercise. (p. 1337) 4. High-intensity and high-impact exercise should be avoided by a patient who has advanced ankylosing spondylitis, because intense exercise can potentially exacerbate the inflammatory process and be potentially harmful. Low-intensity aerobic exercise is recommended instead. (p. 1337

A physical therapist is using exercise as an intervention for a patient with advanced ankylosing spondylitis. Which of the following types of exercise would be MOST important for the patient? 1. Aerobic exercise 2. Balance exercises 3. Light-weight resistance exercises 4. Short-duration, high-intensity exercise

Correct Answer: 2 1. Bilateral touch, also known as sensory extinction, tests a patient's ability to perceive simultaneous stimuli. The patient states "one" or "two" to indicate the number of stimuli felt. (p. 96) 2. The assessment described in the stem tests the sensory modality of touch localization, which is the ability to localize the area tested after a stimulus was provided with vision occluded (p. 95) 3. The sensory modality that best fits the stem described is touch localization. If the patient were asked to indicate an affirmative response when the stimulus is felt, then the appropriate sensory modality tested would have been touch awareness. (p. 93) 4. The touch pressure threshold is tested using Semmes-Weinstein monofilaments. The patient indicates when the stimulus from the monofilaments is felt. (p. 548)

A physical therapist lightly touches a patient's skin with a cotton ball and asks the patient to point to the area that was touched. The patient's vision was occluded during the examination. Which of the following sensory modalities was being tested? 1. Bilateral touch 2. Touch localization 3. Touch awareness 4. Touch pressure threshold

Correct Answer: 3 1. Arthrogryposis comprises nonprogressive conditions characterized by multiple joint contractures found throughout the body at birth. Fatty deposits in the skin and tufts of hair are not characteristic of this disorder. (pp. 1209-1210) 2. A spondylolisthesis is a visible or palpable step-off suggesting forward slippage of one vertebra over another. Fatty deposits in the skin and tufts of hair are not characteristic of this disorder. (pp. 1313-1314) 3. An unusual patch of hair on the back may be evidence of a bony defect of the spine. Fatty masses appearing as lumps in the area of the low back may be a sign of spina bifida. (p. 1171) 4. Paget disease is a chronic bone condition characterized by disorder of the normal bone remodeling process. The bone that is formed is abnormal, enlarged, brittle, and prone to breakage. Fatty deposits in the skin and tufts of hair are not characteristic of this disorder. (pp. 1229-1230)

A physical therapist notes that a patient has a fatty mass and an unusual patch of hair on the low back. Which of the following conditions is MOST likely present? 1. Arthrogryposis 2. Spondylolisthesis 3. Spina bifida occulta 4. Paget disease

Correct Answer: 3 1. The key areas to examine to determine if returning to the workplace is appropriate include the patient's ability to access the environment, to perform the job with or without assistive technology, and to perform the job safely. It is not appropriate or within the scope of physical therapy practice to address issues of workplace satisfaction. 2. The current level of function as related to the job tasks, not the previous level of function, is key in this context. 3. A job analysis as a component of a workplace assessment is the identification of the specific components of job tasks and the work environment. 4. The key areas to examine to determine if returning to the workplace is appropriate include the patient's ability to access the environment, to perform the job with or without assistive technology, and to perform the job safely. It is the employer's role to establish salary and benefit levels.

A physical therapist plans to perform a job analysis for a worker who had a T10 complete spinal cord transection 6 months ago. Which of the following elements would be MOST appropriate to include in the job analysis? 1. Identify specific job tasks that increase the patient's satisfaction. 2. Determine the previous level of function related to the job tasks. 3. Identify specific components of the job tasks and environment. 4. Determine the appropriate level of salary and benefits related to the job tasks.

Correct Answer: 2 1. Interventions for diastasis recti include strengthening exercises, not soft tissue mobilization (Kisner, p. 996). 2. The mobilization described in the stem traces the location of the ascending, transverse, and descending colon and can address intestinal gas pain by promoting motility (Kisner, p. 1012). 3. The mobilization described in the stem does not effectively follow the fibers of the rectus abdominis muscle (Kisner, p. 424). 4. The most common cesarean incision is a horizontal incision through the lower uterine segment, which is below the locations shown in the photograph (Taber's). Furthermore, scar mobilization is not circular but uses cross-friction (Kisner, p. 1012).

A physical therapist provides soft tissue mobilization to a patient following a cesarean section. The mobilization is performed in a clockwise direction at the ascending colon. This intervention is MOST appropriate for which of the following conditions? 1. Diastasis recti 2. Intestinal gas pain 3. Rectus abdominis strain 4. Scar prevention

Correct Answer: 1 1. External validity is the degree to which the results of a study can be generalized to another situation. In this option, the only change in the procedure is the model of the electrical stimulation machine. External validity should be maximized in this situation. 2. The generalizability (external validity) of the findings is only high for patients from the same population (those who have acute disc herniation). 3. The generalizability (external validity) of the findings is only high for patients from the same population (those who have acute disc herniation). 4. The generalizability (external validity) of the findings is only high for patients from the same population (those who have acute disc herniation).

A physical therapist reads about a clinically based study in which electrical stimulation was used for subjects with acute disc herniation. The author gives details about the parameters used and reports that the intervention had a statistically significant effect. External validity is LEAST threatened if the study findings are applied to which of the following groups? 1. Patients from the same population, when the same parameters are used, even if the stimulation device is not the same model 2. Patients from a similar population, when the same stimulation device model and the same parameters are used 3. Patients with a variety of diagnoses as long as they have low back pain 4. Patients with a similar physical therapy problem list as long as the same stimulation device model is used

Correct Answer: 2 1. Persons who have diabetes frequently have distal symmetrical sensorimotor polyneuropathies. With the extent of motor involvement indicated by the manual muscle testing results, it is unlikely that a sensory loss would not be present. 2. Stocking glove distribution of sensory deficit is a hallmark of polyneuropathy, the presence of which is evidenced by the manual muscle testing results. 3. No single dermatome corresponds to the affected myotomes. 4. The manual muscle testing results suggest bilateral involvement of multiple neural structures; therefore, no single spinal nerve or peripheral nerve is involved.

A physical therapist tests the muscle strength of a patient who has had type 2 diabetes for 20 years and finds the following: Biceps: Good (4/5) Quadriceps: Good (4/5) Anterior tibialis: Poor (2/5) Posterior tibialis: Poor (2/5) Gastrocnemius: Poor (2/5) Which of the following is the MOST likely outcome of sensory testing? 1. No loss of sensation in any distribution 2. Loss of sensation in a stocking glove distribution 3. Loss of sensation in a dermatomal distribution 4. Loss of sensation in a peripheral nerve distribution

Correct Answer: 4 1. There are no hormonal changes that will compromise circulation while a pregnant woman is in supine position. Hormonal changes affect pulmonary secretions and rib cage position. 2. Blood volume increases 35% to 50% during pregnancy. 3. Oxygen consumption increases by 15% to 20% during pregnancy. 4. After the 20th week of gestation, the weight of the fetus can impair blood flow through the inferior vena cava when the woman is in supine position.

A pregnant patient reports discomfort and dizziness while lying in supine. Which of the following factors is the MOST likely explanation? 1. Hormonal changes 2. Decreased blood volume 3. Positional effect on oxygen consumption 4. Fetal pressure on the inferior vena cava

Correct Answer: 1 1. The first step in dealing with an angry patient who is not disruptive or a security risk is to validate the patient's feelings by listening and by acknowledging the patient's anger over the situation. This may diffuse the anger, allowing the therapist to carry on with the examination. 2. Attempting to change the direction of the questions is better used for a patient who is emotionally labile or excessively talkative. It does not acknowledge the patient's anger and thus may not diffuse the patient's anger. 3. Leaving the area would be appropriate if a patient is disruptive and appears to have violent intent. Stepping out of the area does not acknowledge the patient's anger. 4. Explaining the importance of collecting the same information may be an appropriate second step, but it does not acknowledge the patient's anger and thus may not diffuse the situation.

After beginning an initial interview with a patient, a physical therapist discerns that the patient is becoming angry. The patient declares that numerous other clinicians have asked the same questions and demands that the therapist contact the physician. What is the MOST appropriate FIRST response by the therapist? 1. Validate the patient's feelings of anger. 2. Attempt to change the direction of the examination questions. 3. Step out of the area and allow the patient to calm down. 4. Explain to the patient the importance of collecting the same information.

Correct Answer: 4 1. Lateral (external) rotation and abduction with flexion is not the most likely position to dislocate the reverse total shoulder arthroplasty. Combined medial (internal) rotation and adduction with extension is more likely to cause dislocation. 2. Abduction and flexion are not most likely to cause dislocation. Combined medial (internal) rotation and adduction with extension is more likely to cause dislocation. 3. Lateral (external) rotation is not most likely to cause dislocation. Combined medial (internal) rotation and adduction with extension is more likely to cause dislocation. 4. Patients are most likely to dislocate a reverse total shoulder arthroplasty by performing medial (internal) rotation and adduction in conjunction with extension. This position allows the prosthesis to escape anteriorly and inferiorly.

After undergoing a reverse total shoulder arthroplasty, a patient is MOST likely to dislocate the shoulder in which of the following positions? 1. Lateral (external) rotation and abduction with flexion 2. Medial (internal) rotation and abduction with flexion 3. Lateral (external) rotation and adduction with extension 4. Medial (internal) rotation and adduction with extension

Correct Answer: 4 1. Return from recline can create shear forces on the ischial tuberosities and sacrum that have the potential to tear skin and lead to pressure injuries. 2. Return from recline can create shear forces on the ischial tuberosities and sacrum that have the potential to tear skin and lead to pressure injuries. 3. Tilt systems do not create shearing forces on the ischial tuberosities or sacrum, but manual tilt-in-space requires assistance from another person for pressure relief. 4. Tilt systems do not create shearing forces on the ischial tuberosities or sacrum, and power tilt-in-space will allow independence in pressure relief without causing shear forces on the buttocks.

An 18-year-old patient who has a traumatic brain injury exhibits mild cognitive deficits, poor head and trunk control, and slowly healing pressure injuries on the ischial tuberosities. The patient has good voluntary movement and motor control in the right arm but has significantly increased tone and poor functional use of the left arm. Which of the following methods of wheelchair pressure relief is BEST for the patient? 1. Manual recline 2. Power recline 3. Manual tilt-in-space 4. Power tilt-in-space

Correct Answer: 4 1. Uncontrolled seizures within the last year are considered a contraindication for aquatic therapy, not a history of seizures in the previous 2 years. 2. Human immunodeficiency virus (HIV) is not a waterborne or airborne transmitted infection. Only waterborne and airborne transmitted infections are contraindications for aquatic therapy. 3. An open wound is not a contraindication as long as it is covered with an occlusive dressing. 4. Aquatic therapy is contraindicated for patients who have a vital capacity of 1 liter or less.

An aquatic-based rehabilitation program is MOST likely to be contraindicated for a patient who has which of the following conditions or characteristics? 1. History of seizures 2 years previously 2. Human immunodeficiency virus (HIV) 3. Leg ulcer covered in an occlusive dressing 4. Pulmonary fibrosis with a vital capacity of 0.8 liters

Correct Answer: 2 1. As gait develops, velocity increases, not decreases, and cadence decreases, not increases. 2. Velocity increases and cadence decreases as gait develops. 3. Single-limb stance time increases as gait develops. 4. Step length increases as gait develops.

As a child ages from 1 to 7 years, which of the following factors indicate maturing gait? 1. Velocity decreases and cadence increases. 2. Velocity increases and cadence decreases. 3. Single-leg stance time decreases and step length increases. 4. Single-leg stance time increases and step length decreases.

Correct Answer: 1 1. The photographs show the modified Thomas test. This test is used to assess iliopsoas complex flexibility. The extreme position of abduction should raise suspicion of possible tensor fasciae latae shortness. To confirm possible shortness of the tensor fasciae latae, the modified Ober test would be appropriate as a follow-up examination. (pp. 916-918) 2. Prone hip extension would be used to further investigate the flexibility of the iliopsoas complex. This test would not evaluate the suspicion of tensor fasciae latae shortness. (p. 917) 3. Prone knee flexion is used to assess the flexibility of the rectus femoris. It will not help clarify suspected shortness of the tensor fasciae latae. (p. 917) 4. The supine straight leg raise test is used to evaluate flexibility of the hamstring muscles. The supine straight leg raise test will not help confirm suspicion of tensor fasciae latae shortness. (pp. 918-919)

At the completion of the the Thomas Test, the patient's leg has extreme abduction. Which of the following tests would be MOST appropriate for follow-up? 1. Modified Ober test 2. Prone hip extension 3. Prone knee flexion 4. Supine straight leg raise

Correct Answer: 3 1. The role of calcitonin is to decrease plasma calcium concentration. There are no receptors on tendons that would alter their function in any way in response to changes in calcitonin levels. Calcitonin should not alter the way muscles and bones interact in a way that would cause chondromalacia, because any calcium changes in the muscle will occur in every muscle, so no imbalance should occur. (pp. 1012-1013) 2. The hormone progesterone is secreted by the placenta during pregnancy and has no known action on tendon laxity. It would not alter the way muscles and bones interact in a way that would cause chondromalacia. (p. 1061) 3. Chondromalacia is a roughening of the cartilage behind the kneecap, and relaxin causes an increase in tendon and ligament laxity, exacerbating any friction between the patella and the femur (p. 1062). 4. Insulin promotes glucose uptake. Although insulin receptors are found in most tissues, any insulin changes in the muscle or tendon will occur in every muscle or tendon, so no imbalance should occur in the interaction of muscles and bones such that chondromalacia would develop. (pp. 988-989)

Changes in the level of which hormone are MOST likely to contribute to development of chondromalacia patella in a pregnant woman? 1. Calcitonin 2. Progesterone 3. Relaxin 4. Insulin

Correct Answer: 4 1. Normal diastolic pulmonary arterial pressure ranges from 5 to 15 mm Hg (Hillegass, p. 423). A measure of 3 mm Hg is pathologically low and may indicate unstable hemodynamic status, which is a relative contraindication for percussion (Hillegass, p. 544). 2. The normal range of intracranial pressure is 0 to 10 mm Hg for adults and 0 to 5 mm Hg for children younger than age 6 years. High intracranial pressure correlates with low cerebral perfusion pressure. Percussion would be likely to further increase this value. (Hillegass, p. 427) 3. Coagulopathy is a precaution for percussion (Hillegass, p. 544). Percussion, like mechanical compression or soft tissue mobilization, increases the risk of injury for patients who have a low platelet count and should not be performed without the approval of the physician (Paz, p. 188). Although a platelet count of 30,000/mm3 is a precaution, a platelet count below 20,000/mm3 is a relative contraindication (Hillegass, p. 544). 4. Normal partial pressure of arterial oxygen is greater than 80 mm Hg (Hillegass, p. 354). As this value drops, the patient may become tachypneic and tachycardic (Paz, p. 64). Airway clearance techniques, such as percussion, can help to optimize ventilation and perfusion matching, increase gas exchange, and increase alveolar ventilation by mobilizing secretions (Hillegass, p. 541).

Chest percussion may be an appropriate intervention for a patient who has which of the following findings? 1. Diastolic pulmonary arterial pressure of 3 mm Hg 2. Intracranial pressure of 30 mm Hg 3. Platelet count of 30,000/mm3 4. Partial pressure of arterial oxygen (PaO2) of 70 mm Hg

Correct Answer: 3 1. Chorea-type movements are related to a pathological condition of the basal ganglia, not the cerebellum. 2. Chorea-type movements are related to a pathological condition of the basal ganglia, not the thalamus. 3. Hyperkinetic disorders such as chorea arise from a pathological condition of the basal ganglia. 4. Chorea-type movements are related to a pathological condition of the basal ganglia, not the limbic system.

Chorea-type movements are noted during an initial gait assessment of a patient referred to physical therapy following a stroke. This clinical finding is indicative of a lesion in the: 1. cerebellum. 2. thalamus. 3. basal ganglia. 4. limbic system.

Correct Answer: 3 1. Children who have spasticity are prone to poor flexibility and would benefit from exercises to improve flexibility (p. 468). 2. Children who have juvenile rheumatoid arthritis lose a substantial amount of flexibility. A stretching program to improve flexibility is important for children who have juvenile rheumatoid arthritis. (p. 146) 3. Children who have Down syndrome are extremely flexible (p. 433). 4. Children who have muscular dystrophy are prone to poor flexibility. A daily stretching program to improve flexibility is recommended for children who have muscular dystrophy. (p. 246)

Exercises to improve flexibility would be LEAST appropriate to include in a physical fitness program for children with which of the following diagnoses? 1. Spastic cerebral palsy 2. Juvenile rheumatoid arthritis 3. Down syndrome 4. Muscular dystrophy

Correct Answer: 4 1. Clubbing of the digits is associated with conditions that have interfered with tissue oxygenation and perfusion for a long period of time. Pleuritis is most often an acute condition that is managed quickly. Pleuritis is not commonly associated with digital clubbing. (pp. 775-776, 859) 2. Clubbing of the digits is associated with conditions that have interfered with tissue oxygenation and perfusion for a long period of time. Atelectasis is most often an acute condition that is managed quickly. Atelectasis is not commonly associated with digital clubbing. (pp. 775-776, 883) 3. Clubbing of the digits is associated with conditions that have interfered with tissue oxygenation and perfusion for a long period of time. Pleural effusion is most often an acute condition that is managed quickly. Pleural effusions are not commonly associated with digital clubbing. (pp. 775-776, 860-861) 4. Clubbing of the digits is associated with conditions that have interfered with tissue oxygenation and perfusion for a long period of time. A chronic disorder that causes hypoxemia, such as pulmonary fibrosis, can cause digital clubbing. (pp. 775-776, 812)

During an initial examination, a physical therapist observes that a patient has clubbing of the digits on bilateral upper extremities. The patient MOST likely has which of the following conditions? 1. Pleuritis 2. Atelectasis 3. Pleural effusion 4. Pulmonary fibrosis

Correct Answer: 2 1. The results given in the stem are describing egophony. Egophony is a voice sounds test and is the transmission of an "A" sound when the patient says "E." Voice sounds will increase in patients who have consolidation, whereas in a patient who has atelectasis, voice sounds will decrease since the lung tissue is deflated and will not transmit sounds. 2. When a patient has consolidated lung tissue or increased secretions, egophony will be present. Egophony is a voice sounds test and is the transmission of an "A" sound when the patient says "E." Voice sounds will increase in patients who have consolidation . 3. The results given in the stem are describing egophony. Egophony is a voice sounds test and is the transmission of an "A" sound when the patient says "E." Voice sounds will increase in patients who have consolidation, whereas in a patient who has pleural effusion, voice sounds will not be present over the effusion since the lung tissue is deflated. Voice sounds may be increased immediately above the line of the fluid of the effusion, but pleural effusion is not the most likely diagnosis since the stem describes a condition present in an entire lobe. 4. The results given in the stem are describing egophony. Egophony is a voice sounds test and is the transmission of an "A" sound when the patient says "E." Voice sounds will increase in patients who have consolidation, whereas in a patient who has a pneumothorax, voice sounds will decrease since the lung is collapsed and will not allow transmission of sounds.

During lung auscultation, a physical therapist asks a patient to continuously say "E." Transmission of an "A" sound is heard over the right lower lobe when the patient says "E." Which of the following conditions is MOST likely present in the right lower lobe? 1. Atelectasis 2. Consolidation 3. Pleural effusion 4. Pneumothorax

Correct Answer: 2 1. Taking the patient's dietary history would overlook the important opportunity to inquire whether the patient may be a victim of physical abuse. 2. Multiple lesions in various stages of healing in a broad number of areas is a sign of possible abuse. In the interest of patient protection, the issue should be pursued, because the patient may not be forthcoming with an admission that abuse has taken place. 3. Instructing the patient in safe movement patterns would overlook the important opportunity to inquire whether the patient may be a victim of physical abuse. 4. Reporting abuse to the appropriate authorities may ultimately be required, but it is not the first step.

During the initial screening of a patient, the physical therapist notes contusions in various states of healing on the chest, back, and face, as well as multiple scars. The patient reports falling often. Which of the following courses of action should the therapist take FIRST? 1. Take the patient's dietary history to assess for mineral deficiency. 2. Ask the patient if bruising was caused by being hit, kicked, or abused. 3. Instruct the patient in safe movement patterns to avoid falls. 4. Report the patient's caregivers to the appropriate authorities.

Correct Answer: 2 1. The myotome associated with L4 is ankle dorsiflexion. The dermatome associated with L4 is along the medial leg and foot. The stem describes involvement of the L5 nerve root. 2. The myotome associated with L5 is toe extension/dorsiflexion. The dermatome associated with L5 is the lateral leg and dorsum of the foot. 3. The myotome associated with S1 is ankle plantar flexion. The dermatome associated with S1 is along the posterior thigh and lateral foot. The stem describes involvement of the L5 nerve root. 4. The myotome associated with S2 is knee flexion, and the dermatome associated with S2 is the posterior thigh and medial ankle. The stem describes involvement of the L5 nerve root.

Examination of a patient's right lower extremity reveals weakness in great toe extension and decreased sensation along the lateral leg and dorsum of the foot. Which of the following nerve roots is MOST likely contributing to these findings? 1. L4 2. L5 3. S1 4. S2

Correct Answer: 1 1. New symptoms of redness, swelling, and increasing warmth following a surgical procedure are indicators of a possible infection. The surgeon should be notified. (O'Sullivan) 2. Redness, swelling, and increasing warmth are indicators of possible acute inflammation, which is a contraindication for superficial moist heat (Cameron, p. 155). 3. Ice is not the modality of choice to prepare a patient for exercise and gait training (Cameron, pp. 134-135). 4. Because the patient is demonstrating signs of a possible infection at the surgical site, the most appropriate action is to notify the surgeon. Inaction could cause potential harm to the patient. (O'Sullivan)

Following a total knee arthroplasty, a patient has been receiving moist heat to the knee prior to exercise and gait training. During the current visit, the physical therapist notes new redness, swelling, and increasing warmth surrounding the knee. Which of the following actions should the therapist take? 1. Notify the orthopedic surgeon of the changes. 2. Continue with the use of hot packs for 5 to 10 minutes prior to physical therapy. 3. Substitute ice instead of heat prior to physical therapy. 4. Discontinue physical therapy until the problems are resolved.

Correct Answer: 2 1. High Fowler position is defined as a position in which the head of the patient's bed is raised 80° to 90° with knees flexed (p. 173). Over time this position increases the risk of development of a knee flexion contracture (p. 190). 2. The residual limb supported by pillows and the knee resting in full extension is the preferred position, because it decreases the risk for knee flexion contracture (p. 190). 3. In this position the residual limb is supported, but the knee is still flexed, creating pressure on the distal end of the residual limb that could lead to pressure injuries (p. 190). 4. In this position the residual limb is in a dependent position, which promotes swelling, and the knee is flexed, which promotes knee flexion contracture (p. 190).

For a patient who has undergone a transtibial amputation, which of the following seated positions is MOST appropriate? 1. High Fowler position with the residual limb supported by pillows and the knee resting in 45° of flexion 2. Residual limb supported by pillows and the knee resting in full extension 3. Residual limb supported by pillows and the knee resting in 30° of flexion 4. Upright with bilateral hips and knees in 90° of flexion

Correct Answer: 4 1. Prone with the lower extremities raised 18 inches (45.7 cm) is the preferred position for postural drainage of the lower lobes. 2. Supine position with the lower extremities raised 12 inches (30.5 cm) is the preferred position for postural drainage of right middle lobe secretions. 3. Supine with the lower extremities raised 18 inches (45.7 cm) is the preferred position for postural drainage of anterior segment secretions. 4. The correct patient position for postural drainage of the right lateral segment is left sidelying with the legs raised 18 inches (45.7 cm).

For a patient who is undergoing postural drainage for secretions in the right lateral segment, which of the following positions would be MOST appropriate? 1. Prone with the lower extremities raised 18 inches (45.7 cm) 2. Supine with the lower extremities raised 12 inches (30.5 cm) 3. Supine with the lower extremities raised 18 inches (45.7 cm) 4. Left sidelying with the lower extremities raised 18 inches (45.7 cm)

Correct Answer: 4 1. The test shown in the photograph is the Phalen test, for which a positive result is indicative of carpal tunnel syndrome (p. 474). The Allen test is used to determine the patency of the radial and ulnar arteries and to determine which artery provides a majority of the blood supply to the hand; the Allen test is used to identify thoracic outlet syndrome, not carpal tunnel syndrome (p. 476). 2. The test shown in the photograph is the Phalen test, for which a positive result is indicative of carpal tunnel syndrome (p. 474). A positive result of the Froment test indicates ulnar nerve palsy (p. 473). It is not indicative of carpal tunnel syndrome. 3. The test shown in the photograph is the Phalen test, for which a positive result is indicative of carpal tunnel syndrome (p. 474). A positive result of the piano keys test is indicative of dysfunction of the distal radioulnar joint (p. 467). It is not indicative of carpal tunnel syndrome. 4. The test shown in the photograph is the Phalen test, for which a positive result is indicative of carpal tunnel syndrome. A positive result of the carpal compression test is also indicative of carpal tunnel syndrome. (p. 474)

If the test shown in the photograph has a positive result, which of the following tests would MOST likely also have a positive result? 1. Allen test 2. Froment test 3. Piano keys test 4. Carpal compression test

Correct Answer: 2 1. Energy delivery to the tissues with the use of ultrasound is a function of various parameters, including intensity, frequency, and duty cycle. Continuous mode (or 100% duty cycle) produces thermal effects compared to pulse mode (p. 185). Research indicates that a frequency of 3 MHz results in a higher maximal temperature than 1 MHz despite delivering a lesser depth of penetration (p. 175). Furthermore, higher intensities produce higher temperature increases in tissues (p. 175). 2. Energy delivery to the tissues with the use of ultrasound is a function of various parameters, including intensity, frequency, and duty cycle. Continuous mode (or 100% duty cycle) produces thermal effects, compared to pulsed mode (p. 185). Research indicates that a frequency of 3 MHz results in a higher maximal temperature than 1 MHz despite delivering a lesser depth of penetration (p. 175). Furthermore, higher intensities produce higher temperature increases in tissues (p. 175). 3. Energy delivery to the tissues with the use of ultrasound is a function of various parameters, including intensity, frequency, and duty cycle. Pulsed mode (less than 100% duty cycle) produces nonthermal effects, compared to continuous mode (p. 185). 4. Energy delivery to the tissues with the use of ultrasound is a function of various parameters, including intensity, frequency, and duty cycle. Pulsed mode (less than 100% duty cycle) produces nonthermal effects, compared to continuous mode (p. 185).

If treatment time and surface area are kept constant, which of the following ultrasound parameters would MOST likely deliver the GREATEST amount of energy through tissues? 1. 0.5 W/cm2 in continuous mode at 1 MHz 2. 0.8 W/cm2 in continuous mode at 3 MHz 3. 1.0 W/cm2 in 50% pulsed mode at 1 MHz 4. 1.2 W/cm2 in 25% pulsed mode at 3 MHz

Correct Answer: 2 1. Rigid trunk support can be used to support diaphragm function in people who have weak abdominal muscles, but the orthosis requires an anterior cut-out to allow the diaphragm to descend (DeTurk). A corset is recommended over a rigid orthosis when the goal is compression and not movement restriction (O'Sullivan). 2. Use of an abdominal corset in patients who have weak abdominal muscles can compensate for laxity and can improve respiratory function (Nichols-Larsen). 3. In sitting position, the abdominal contents shift inferiorly and anteriorly in patients who have weak abdominals. This causes the diaphragm to be pulled into a more horizontal position, where its mechanical function is at an extreme disadvantage. (Nichols-Larsen) 4. In sitting position, the abdominal contents shift inferiorly and anteriorly in patients who have weak abdominals. This causes the diaphragm to be pulled into a more horizontal position, where its mechanical function is at an extreme disadvantage. (Nichols-Larsen)

In a patient who has weak oblique, rectus abdominis, and transversus abdominis muscles, which of the following interventions is MOST likely to improve the mechanical efficiency of the diaphragm? 1. Use of a rigid trunk support 2. Use of an abdominal binder 3. Assuming an erect sitting position 4. Assuming a forward leaning sitting position

Correct Answer: 1 1. Standing on foam allows the ankle joint to at least begin rotating and prevents the somatosensory systems from relaying accurate information from the ankle joints and the skin. The eyes are open, and the vestibular system is also not compromised. Visual input dominates under this condition. 2. Standing on foam allows the ankle joint to at least begin rotating and prevents the somatosensory systems from relaying accurate information from the ankle joints and skin. Therefore, proprioception is not being relied upon. 3. Standing on foam allows the ankle joint to at least begin rotating and prevents the somatosensory systems from relaying accurate information from the ankle joints and the skin. Therefore, proprioception is not being relied upon. 4. Standing on foam allows the ankle joint to at least begin rotating and prevents the somatosensory systems from relaying accurate information from the ankle joints and the skin. Therefore, proprioception is not being relied upon.

In the photograph, which systems are PREDOMINANTLY being used by the patient to maintain balance? 1. Visual and vestibular 2. Visual and proprioceptive 3. Proprioceptive and vestibular 4. Vestibular, visual, and proprioceptive

Correct Answer: 3 1. With the wrist in neutral, the finger flexors are not lengthened to the optimal length for tenodesis action and, as a result, are not as likely to generate the greatest force. 2. In 15° of flexion, the finger flexors are shortened and, as a result, are not as likely to generate the greatest force. 3. A muscle has maximal ability to generate force (or tension) when the muscle is contracted at its optimal length. Finger flexors involved in grip cross the wrist, so wrist position affects the length of the finger flexors and consequently also affects the ability of the flexors to generate force. The optimal length of the finger flexors is maintained when the wrist is held at approximately 30° of extension. 4. In 60° of extension, the wrist flexors are lengthened beyond their optimal length and, as a result, are not as likely to generate the greatest force.

In which of the following wrist positions would maximal grip strength MOST likely be generated? 1. 0° (neutral) 2. 15° of flexion 3. 30° of extension 4. 60° of extension

Correct Answer: 1 1. Lesions in the skin are often the first sign of an underlying rheumatic disease. Scleroderma is accompanied by many skin changes. (p. 445) 2. Thromboangiitis obliterans or Buerger disease is vasculitis affecting the peripheral blood vessels. Clinical manifestations of pain and tenderness in the affected part are caused by occlusion of arteries, reduced blood flood, and subsequent reduced oxygenation. Symptoms of cold sensitivity, rubor (redness from dilated capillaries under the skin), cyanosis, and thin, shiny, hairless skin (trophic changes) occur from chronic ischemia. Thus, integumentary changes occur in the latter part of the disease process, when chronic ischemia and oxygenation may eventually lead to ischemic ulceration and possible development of gangrene. (p. 637) 3. Mild anemia often causes only minimal and usually vague symptoms such as fatigue. As anemia progresses, the general signs and symptoms caused by the inability of anemic blood to supply the body tissues with enough oxygen may include pallor or yellowness of skin, especially of the palms of the hands and fingernails, mucosa, and conjunctiva; leg ulcers; and occasionally koilonychia or spoon-shaped nails. (p. 715) 4. Integumentary clinical manifestations of hypothyroidism include carotenosis, poor wound healing, and dry, flaky skin (pp. 488-489).

Lesions of the skin are the FIRST clinical sign of underlying disease for which of the following diagnoses? 1. Scleroderma 2. Thromboangiitis obliterans 3. Anemia 4. Hypothyroidism

Correct Answer: 3 1. The gate control theory of pain states that activation of large-diameter fibers will also activate the substantia gelatinosa, which closes the gate. Activation of the A fibers alone would not have this effect. 2. The gate control theory of pain states that small-diameter fiber cells are stimulated by a noxious stimulus, not by superficial nonnoxious stimulation, such as low-intensity conventional transcutaneous electrical stimulation. 3. The gate control theory of pain states that large-diameter fibers are activated with nonnoxious stimulation, such as transcutaneous electrical nerve stimulation. This will cause activation of the substantia gelatinosa, which will close the gate and block pain signals being sent by T-cells. 4. The gate control theory of pain states that small-diameter fibers are activated by noxious stimulation, which would then inhibit substantia gelatinosa (not activate it) and thereby open the gate to allow pain perception to be transmitted.

Low-intensity conventional transcutaneous electrical nerve stimulation is being used to control pain. Which of the following physiological responses is MOST likely to occur? 1. Activation of the A (large-diameter) fibers only 2. Activation of the C (small-diameter) fibers only 3. Activation of the A (large-diameter) fibers and substantia gelatinosa 4. Activation of the C (small-diameter) fibers and substantia gelatinosa

Correct Answer: 2 1. Manual lymph drainage is useful to stimulate circulation to the extremities and is an appropriate treatment for chronic edema (pp. 248-249). 2. Manual lymph drainage facilitates the removal of fluid from the extremities back into the circulatory system. If the patient has heart failure, the heart will not be able to handle the additional fluid and the heart failure will worsen. (p. 253) 3. Manual lymph drainage is indicated for venous insufficiency. Excess fluid in the extremities can be managed by this technique. (pp. 248-249) 4. Effects of manual lymph drainage include pain relief and relaxation. It is, therefore, a useful treatment for complex regional pain syndrome (also called reflex sympathetic dystrophy). (pp. 248-249).

Manual lymphatic drainage is CONTRAINDICATED for a patient who has which of the following conditions? 1. Chronic edema 2. Congestive heart failure 3. Chronic venous insufficiency 4. Complex regional pain syndrome

Correct Answer: 1 1. Ocular torticollis may result from a lesion to the trochlear nerve. Damage to the trochlear nerve results in diplopia. Patients will frequently compensate for the diplopia by tilting the head anteriorly and laterally toward the side of the normal eye. (pp. 1080-1081) 2. Damage to the facial nerve leads to Bell palsy. Clinical presentation includes loss of taste, paralysis of the muscles of facial expression, and loss of salivation and tear production. It does not lead to diplopia or a compensatory head tilt. (p. 1081) 3. Damage to the vestibulocochlear nerve results in hearing loss and vestibular symptoms, including loss of balance, vertigo, nystagmus, and impaired ability to maintain gaze. It does not lead to diplopia or a compensatory head tilt. (p. 1082) 4. Damage to the hypoglossal nerve results in deviation of the tongue to one side during protrusion of the tongue. It does not lead to diplopia or a compensatory head tilt. (p. 1082)

Palsy of which of the following cranial nerves would MOST likely lead to the presentation of torticollis in a child? 1. Trochlear nerve (CN IV) 2. Facial nerve (CN VII) 3. Vestibulocochlear nerve (CN VIII) 4. Hypoglossal nerve (CN XII)

Correct Answer: 2 1. Pleural effusion is a precaution for postural drainage but not a contraindication. 2. Recent head trauma is a contraindication for postural drainage. 3. The only surgeries that are contraindications for postural drainage are spinal fusions and eye surgeries. 4. A history of pulmonary embolism is a precaution to postural drainage but not a contraindication.

Postural drainage for the posterior segments of the lower lobes would be CONTRAINDICATED for which of the following patients? 1. A patient who has a pleural effusion. 2. A patient who had a recent head trauma. 3. A patient who had a recent total knee arthroplasty. 4. A patient who has a history of pulmonary embolism

Correct Answer: 1 1. When the dominant temporal lobe is involved, receptive aphasia may be present. The magnetic resonance imaging (MRI) shows a lesion in the parietal and temporal areas. (p. 1514) 2. Control of executive function is in the frontal lobe (pp. 1381-1382). 3. Ataxia occurs with a cerebellar lesion (p. 1380). 4. Greater upper extremity involvement is seen in anterior cerebral artery syndrome, and the magnetic resonance imaging (MRI) shows a lesion where the middle cerebral artery supplies blood (p. 1514).

On the basis of the magnetic resonance imaging data shown in the photograph, which of the following findings is MOST likely to be found during examination of a patient who is left-handed? 1. Receptive aphasia 2. Executive function changes 3. Bilateral ataxia observed with intentional movement 4. Lower extremity involvement greater than upper extremity involvement

Correct Answer: 1 1. Herpes zoster (shingles) is a painful, blistering skin rash caused by the varicella-zoster virus. The first symptom is usually one-sided pain, tingling, or burning followed by development of a rash that usually involves a narrow area from the spine around to the front of the chest or abdomen. A typical location for occurrence of shingles rash is the T11-T12 dermatome along the iliac crest. The location of the rash, postsurgical onset, and symptoms of pain all suggest herpes zoster. (Goodman, p. 177) 2. The surgical incision for a lumbar laminectomy is located along the lumbar spine. An infected surgical incision would be associated with pain, erythema, edema, and increased temperature over the wound site. (Bryant) 3. The area of rash described in the stem includes the area where the pelvic (lower) strap of the thoracolumbosacral orthosis is placed. However, contact dermatitis would not present with the dermatomal pattern described and would be more itchy than painful. (Goodman, p. 171) 4. An allergic response to medication would not be localized to a specific dermatome but would most likely be a generalized response. Also a rash due to medication is generally not painful. (Goodman, pp. 171, 435)

One day after lumbar laminectomy surgery, a patient refuses to wear a thoracolumbosacral orthosis because of a painful and itching rash that extends in a narrow path from the central low back along the iliac crest to the right lateral trunk. Which of the following conditions is MOST likely present? 1. Herpes zoster 2. Infected surgical incision 3. Contact dermatitis 4. Allergic response to medication

Correct Answer: 1 1. Pain from urinary calculi results from the ureter contracting in the attempt to dislodge the calculi. 2. Pain from urinary calculi is more likely to result from obstruction in the ureter than from obstruction in the urethra. 3. Pain from urinary calculi results from obstruction in the ureter, not the bladder. 4. Pain from urinary calculi results from obstruction in the ureter, not the kidneys.

Pain associated with urinary calculi MOST often occurs because of blockage of which of the following structures? 1. Ureter 2. Urethra 3. Bladder 4. Kidney

Correct Answer: 3 1. The area highlighted in the photograph is the C7 nerve root distribution. Absent sensation due to a nerve root lesion would cause muscles innervated by the C7 nerve root to be affected as well (p. 24). The deltoid, which is innervated by the axillary nerve (C5-C6), would be largely unaffected (p. 263). 2. The area highlighted in the photograph is the C7 nerve root distribution. Absent sensation due to a nerve root lesion would cause muscles innervated by the C7 nerve root to be affected as well (p. 24). The brachialis, which is innervated by the musculocutaneous nerve (C5-C6), will be largely unaffected (p. 177). 3. The area highlighted in the photograph is the C7 nerve root distribution. Absent sensation due to a nerve root lesion would cause muscles innervated by the C7 nerve root to be affected as well (p. 24). The triceps, which is innervated by the radial nerve (C7-C8), would most likely be affected as well (p. 24). 4. The area highlighted in the photograph is the C7 nerve root distribution. Absent sensation due to a nerve root lesion would cause muscles innervated by the C7 nerve root to be affected as well (p. 24). The supinator is innervated by the radial nerve from the C6 nerve root and would be largely unaffected (p. 177).

Sensation in the area highlighted in the photograph is absent due to a nerve root lesion. Which of the following muscles is MOST likely to be affected? 1. Deltoid 2. Brachialis 3. Triceps 4. Supinator

Correct Answer: 3 1. The use of superficial cooling over superficial nerves is a precaution (p. 134). 2. Hypertension is listed as a precaution for use of superficial cooling (p. 134). 3. Raynaud syndrome is listed as a contraindication for use of superficial cooling (p. 133). 4. Poor cognition is listed as a precaution for use of superficial cooling (p. 134).

Superficial cooling is CONTRAINDICATED in which of the following situations? 1. For a patient with an injury near the location of the fibular nerve 2. For a patient who has a blood pressure of 176/90 mm Hg 3. For a patient who has periodic vasoconstriction in the fingers 4. For a patient who has a Mini-Mental State Examination score of 20/30

Correct Answer: 2 1. To correct for a forward loss of balance, the patient would use a forward sway ankle strategy, recruiting the gastrocnemius, quadriceps femoris, and paraspinal trunk extensors. This strategy is not being tested in the photograph. 2. An ankle strategy in response to a posterior perturbation is demonstrated in the photograph. The correct muscle recruitment for a posterior displacement of center of gravity for an ankle strategy is tibialis anterior, quadriceps femoris, and abdominals. This is the body motion used to reestablish stability in response to backward instability. 3. An ankle strategy is used in situations when the support surface is firm; in the case of uneven surfaces, the patient would utilize a hip strategy. This strategy is not being tested in the photograph. 4. A patient standing with a narrow base of support would utilize a hip strategy for correction of the center of mass. This strategy is not being tested in the photograph.

Testing of reactive balance as shown in the photograph is MOST likely to provide useful findings for a patient who reports which of the following problems? 1. Tendency to fall forward 2. Tendency to fall backward 3. Tendency to fall on uneven surfaces 4. Tendency to fall when standing with a narrow base of support

Correct Answer: 2 1. The axillary nerve innervates the deltoid and teres minor muscles. Injury would result in difficulty abducting the arm with neutral rotation. (pp. 80-81) 2. The long thoracic nerve innervates the serratus anterior. Injury to this nerve could result in winging of the scapula. (pp. 76-77) 3. The suprascapular nerve innervates the supraspinatus and infraspinatus muscles. Injury would likely result in increased scapular elevation, not winging. (pp. 78-79) 4. The musculocutaneous nerve innervates the biceps, brachialis, and coracobrachialis muscles. Injury would most likely result in weak elbow flexion with the forearm in supination. (pp. 79, 81)

The clinical presentation of the patient in the photograph is MOST likely to result from compression of which of the following nerves? 1. Axillary 2. Long thoracic 3. Suprascapular 4. Musculocutaneous

Correct Answer: 1 1. Initial paresis grade is an important predictor of motor recovery. Of patients admitted with complete paralysis, less than 15% experience complete motor recovery. 2. Paresis status upon hospital admission has not been found to be a predictor for proprioception. 3. Paresis grade has not been found to be a predictor of executive functions. 4. The risk of seizures is more related to the location of the occlusion or infarct than to the degree of hemiparesis.

The degree of hemiparesis initially noted after a cerebrovascular accident is MOST predictive of which of the following? 1. Motor recovery 2. Proprioception 3. Executive functions 4. Risk for seizures

Correct Answer: 1 1. The device shown in the photograph is a chest tube collection device. Removal of a chest tube may result in pneumothorax. A radiograph of the chest is needed to rule out pneumothorax prior to mobilization of the patient. (p. 439) 2. The device shown in the photograph is a chest tube collection device. A radiograph of the chest is needed to rule out pneumothorax prior to mobilization of the patient (p. 439). An electrocardiogram represents electrical impulses of the heart and provides information regarding the heart's function (p. 305). 3. The device shown in the photograph is a chest tube collection device. A radiograph of the chest is needed to rule out pneumothorax prior to mobilization of the patient (p. 439). Pulmonary function tests can show changes in expiratory or inspiratory flow but are not definitive tests for pneumothorax (pp. 346-347). 4. The device shown in the photograph is a chest tube collection device. A radiograph of the chest is needed to rule out pneumothorax prior to mobilization of the patient (p. 439). A computed tomography scan is not required to determine the presence of a pneumothorax (p. 277).

The device shown in the photograph has been removed from a patient. Which of the following tests is required prior to mobilizing the patient? 1. Radiograph 2. Electrocardiogram 3. Pulmonary function test 4. Computed tomography scan

Correct Answer: 1 1. The patient in the photograph demonstrates a lordotic posture of the lumbar spine, which increases compressive forces on the disc (Kisner). The position of the arms outstretched and away from the body increases the flexion moment around the body's center of mass (Samuels). 2. The lordotic posture of the lumbar spine increases, not decreases, compressive forces on the disc (Kisner). The position of the arms outstretched and away from the body increases the flexion moment, not decreases it (Samuels). 3. The lordotic posture of the lumbar spine increases compressive forces (Kisner). 4. The position of the arms outstretched and away from the body increases the flexion moment, not the extension moment, around the body's center of mass (Samuels).

The individual in the photograph (lumbar extended, arms outstretched forward away from body) is demonstrating how she lifts her laundry basket. What impact does the position of the basket have on the forces at the lumbar spine and disc? 1. The position increases the compressive forces and flexion moment of the disc. 2. The position decreases the compressive forces and flexion moment of the disc. 3. The position decreases the compressive forces and increases the flexion moment of the disc. 4. The position increases the compressive forces and extension moment of the disc.

Correct Answer: 2 1. A Stemmer sign is positive if the skin on the dorsum of the toes and fingers cannot be lifted, or is difficult to lift, compared to the opposite side. Presence of a Stemmer sign is considered to accurately diagnose the presence of lymphedema (p. 73.) Therefore, presence of a Stemmer sign, in the absence of peripheral arterial disease, would indicate that multilayered compression bandaging may be beneficial (pp. 272-273). 2. Multilayered compression bandaging or "lymphedema bandaging" is a technique used to provide compression therapy during the first phase of complete decongestive therapy (pp. 272-273). A wound that is deep, regularly shaped, and dry is most likely an arterial wound (p. 141). Compression therapy is contraindicated for management of edema in the presence of peripheral arterial disease (p. 267). 3. A superficial, irregularly shaped, highly exudative wound is most likely a venous ulcer (p. 141). Compression therapy is considered one of the most important therapeutic measures for prevention and treatment of venous leg ulcers (pp. 272-273). When applying multilayered compression bandaging to an extremity with a venous ulcer, the physical therapist may incorporate materials or medications prescribed by the physician into the bandaging to manage the wound (p. 82). Therefore, the presence of a venous ulcer is not a contraindication to using multilayered compression bandaging. 4. Papillomas and hyperkeratosis are common dermal abnormalities that occur in the presence of chronic lymphedema (pp. 79-80). These dermal changes do not prohibit the use of multilayered compression bandaging.

The intervention shown in the photograph is CONTRAINDICATED in the presence of which of the following clinical signs? 1. A positive Stemmer sign 2. A deep, dry, regularly shaped wound 3. A superficial, irregularly shaped, exudative wound 4. Presence of papillomas and hyperkeratosis

Correct Answer: 2 1. Rotator cuff dysfunction is identified better with the Hawkins-Kennedy test (Dutton, pp. 630-631; Magee, p. 315). The Speed test is shown in the photograph (Dutton, p. 640; Magee, p. 341). 2. The Speed test is shown in the photograph. Deep shoulder pain produced during this maneuver suggests a labral tear, (Dutton, p. 640) or a superior labrum anterior to posterior (SLAP) (Type II) lesion (Magee, p. 341). 3. Subacromial bursitis is identified better with the Hawkins-Kennedy test (Dutton, pp. 630-631; Magee, p. 315). The Speed test is shown in the photograph (Dutton, p. 640; Magee, p. 341). 4. Supraspinatus tendinitis is identified better with the Empty Can test, in which the arm would be in maximum medial (internal) rotation (Dutton, p. 637; Magee, p. 341). The Speed test is shown in the photograph (Dutton, p. 640; Magee, p. 341).

The patient being assessed in the photographs reports a deep pain in the shoulder when a physical therapist applies a downward force to the forearm. The patient MOST likely has which of the following conditions? 1. Rotator cuff tear 2. Superior labrum tear 3. Subacromial bursitis 4. Supraspinatus tendinitis

Correct Answer: 1 1. The test shown in the photograph is used to assess the integrity of the anterior cruciate ligament. Jumping and landing with stability are required for the patient's functional goals associated with treatment of an anterior cruciate ligament injury. 2. Although a nonantalgic gait pattern is a goal for a patient recovering from an anterior cruciate ligament injury, the presence of a nonantalgic gait does not demonstrate safety with higher-level tasks required for return to athletic activity. 3. Although functional range of motion is restored, pain can be an indicator of continued inflammatory response and swelling, indicating that the patient is not ready for return to athletic activity. 4. Controlling joint effusion and pain and strengthening the quadriceps are part of the program of rehabilitation after anterior cruciate ligament injury, but these criteria are not functionally sufficient for the patient to resume athletic activity.

The patient in the photograph sustained a twisting injury to the knee 1 week ago. The physical therapist notes laxity when performing the test demonstrated in the photograph. The patient wants to resume athletic activity as soon as possible. Which of the following criteria MUST be met before the patient returns to athletic activity?

Correct Answer: 3 1. The patient demonstrates tightness in the iliopsoas, which will not affect sitting posture. Sitting places the hip in flexion and the iliopsoas in a shortened position, so the muscle will not pull at its lumbar attachments. (Reese, p. 422) 2. The patient demonstrates tightness in the iliopsoas, which will not affect sitting posture. The position of the knees will not affect the length of the iliopsoas, because this muscle does not cross the knee joint. (Reese, p. 422) 3. The result shown in the photograph indicates hip flexor tightness (Reese, p. 423). Tight hip flexors contribute to an increase in lumbar lordosis (Fruth). 4. Patients who have a flat back have shortened hamstring muscles, not tight hip flexors (Fruth)

The patient in the photograph was asked to slowly lower and relax the left leg. The photograph indicates the final position in a relaxed state. How will the findings indicated in the photograph MOST likely influence the patient's posture? 1. Sitting with the knees flexed will cause hyperlordosis. 2. Sitting with the knees straight will cause a flat lumbar spine. 3. The patient will have a tendency to stand with hyperlordosis. 4. The patient will have a tendency to stand with a flat back.

Correct Answer: 4 1. The radiograph clearly shows spondylolisthesis, the forward displacement of one vertebra upon the stationary vertebra beneath it. This displacement would not be seen with spinal stenosis. In addition, magnetic resonance imaging is considered the best choice for characterizing spinal stenosis. (pp. 336-339, 341) 2. The radiograph clearly shows spondylolisthesis, the forward displacement of one vertebra upon the stationary vertebra beneath it. This displacement would not be seen in a patient who has spondylolysis; instead, a line would be seen across the pars interarticularis. (pp. 334, 336-339) 3. The radiograph clearly shows spondylolisthesis, the forward displacement of one vertebra upon the stationary vertebra beneath it. This displacement would not be seen with a disc herniation. In addition, magnetic resonance imaging is the study of choice for disc herniation. (pp. 336-339, 344-345) 4. Spondylolisthesis is the forward displacement of one vertebra upon the stationary vertebra beneath it, as shown in the radiograph (pp. 336-339).

The patient whose radiograph is shown MOST likely has which of the following conditions? 1. Stenosis 2. Spondylolysis 3. Intervertebral disc herniation 4. Spondylolisthesis

Correct Answer: 2 1. The photograph depicts passive assessment of hip medial (internal) rotation range of motion. An anterior glide would be limited if lateral (external) hip rotation was limited, not medial (internal) rotation. 2. The photograph depicts passive assessment of hip medial (internal) rotation range of motion. A posterior glide facilitates this motion because of the convex-concave rule and would be most likely be restricted given the limitation. 3. The photograph depicts passive assessment of hip medial (internal) rotation range of motion. An inferior glide would be limited if hip abduction, not medial (internal) rotation, range of motion was limited. 4. The photograph depicts passive assessment of hip medial (internal) rotation range of motion. A superior glide would be limited if hip adduction was limited, not medial (internal) rotation.

The photograph depicts the end range of a patient's hip motion. Which of the following arthrokinematic motions is MOST likely restricted? 1. Anterior glide 2. Posterior glide 3. Inferior glide 4. Superior glide

Correct Answer: 2 1. Hands are placed over the anterior portion of the first four ribs with the fingertips extended over the upper trapezius. This is the location to assess upper lobe excursion, not movement of the diaphragm. (p. 525f) 2. Hands are placed over the anterior chest with the thumbs over the costal margin so the tips of fingers almost meet at the xiphoid process. This is the proper position to palpate the diaphragm, which is located anatomically in the inferior aspect of the thorax. (p. 527f) 3. Hands are positioned with fingers over the trapezius with palms on the posterior aspect of thorax. This position is used to assess the excursion of the upper posterior lobes of the lungs, not the movement of the diaphragm. (p. 525f). 4. Hands are placed over the lower ribs. This is the location to assess the middle lobe lung excursion, not movement of the diaphragm. (p. 525f)

The photographs indicate locations for hand placement to assess various aspects of breathing. Which of the photographs indicates the MOST appropriate hand placement for assessing movement of the diaphragm? 1. A 2. B 3. C 4. D

Correct Answer: 3 1. Posterior pelvic tilts performed in supine position would not be contraindicated for a patient who has irritable bowel syndrome (pp. 890-891). 2. Posterior pelvic tilts performed in supine position would not be contraindicated for a patient who has a duodenal ulcer (pp. 878-879). 3. When a client has a known hiatal hernia, the supine position and any exercise (such as posterior pelvic tilts) for which the patient might use the Valsalva maneuver should be avoided (pp. 901-902). 4. Diverticulosis presents with diverticula in the wall of the colon or small intestine, but no infection or inflammation. Posterior pelvic tilts performed in supine position would not be contraindicated for a patient who has diverticulosis, although the patient should not hold his or her breath (perform Valsalva maneuver) when exercising. (pp. 893-894)

The physical therapy plan of care for a patient with low back pain includes posterior pelvic tilts performed in the supine position. Modification of the plan of care would be MOST needed if the patient were found to have which of the following conditions? 1. Irritable bowel syndrome 2. Duodenal ulcer 3. Hiatal hernia 4. Diverticulosis

Correct Answer: 4 1. The photograph shows a radial nerve glide (Kisner, p. 399). Cubital tunnel syndrome involves entrapment of the ulnar nerve at the elbow and would most appropriately be treated by ulnar nerve glides (Brody, p. 821). 2. The photograph shows a radial nerve glide (Kisner, p. 399). Carpal tunnel syndrome involves the median nerve and would be exacerbated by the full wrist flexion in the radial glide shown in the photograph (Brody, pp. 818-819). 3. The photograph shows a radial nerve glide (Kisner, p. 399). Medial epicondylalgia would be exacerbated by full wrist flexion demonstrated in the photograph (Brody, p. 823). 4. The photograph shows a radial nerve glide (Kisner, p. 399). This maneuver is used to treat symptoms related to shoulder girdle depression, radial nerve distribution, and disorders such as tennis elbow (lateral epicondylalgia) and de Quervain syndrome (Kisner, pp. 399, 643).

The position demonstrated in the photograph is BEST used to treat a patient who has which of the following conditions? 1. Cubital tunnel syndrome 2. Carpal tunnel syndrome 3. Medial epicondyle tendinopathy 4. Lateral epicondyle tendinopathy

Correct Answer: 2 1. The infraspinatus is a key lateral (external) rotator of the shoulder. Infraspinatus function is examined by means of a manual muscle test and the lateral rotation lag sign. Infraspinatus function is not implicated in the findings from the lift-off test. 2. The photograph demonstrates the lift-off sign or Gerber test which is considered a test for integrity of the subscapularis. Although several muscles contribute to medial (internal) rotation of the humerus at the shoulder, the subscapularis is thought to be best isolated with this special test. If a patient is able to lift the hand away from the lower back (through active shoulder medial [internal] rotation), subscapularis function is intact. A positive finding (subscapularis compromise) is indicated by at least one of the following: frank inability to lift the dorsum of the hand from the back or straightening of the elbow to move the hand from the lower back but inability to "lift-off." In the photograph, the patient is maintaining elbow flexion while lifting the dorsum of the hand away, which is a negative test result. 3. The supraspinatus generates shoulder abduction. Integrity of the supraspinatus is tested a number of ways, including the supraspinatus manual muscle test, empty can test, and drop-arm test. Function of the supraspinatus muscle is not implicated in the findings from the lift-off test. 4. Latissimus dorsi functions include shoulder extension, with some adduction and medial (internal) rotation. It is also active with numerous functions of scapular movement (e.g., depression and protraction). The integrity of the latissimus dorsi is best appreciated with a manual muscle test isolating the muscle. Latissimus function is not directly implicated in the findings from a lift-off test. Although the latissimus dorsi is a contributor to medial (internal) rotation, subscapularis activity is optimized in the lift-off test position.

The test shown in the photograph is used to assess the integrity of which of the following muscles? 1. Infraspinatus 2. Subscapularis 3. Supraspinatus 4. Latissimus dorsi

Correct Answer: 3 1. The test in the photograph is the jaw reflex test, which tests the integrity of the trigeminal nerve (CN V), which is a cranial nerve, not cartilage. 2. The test in the photograph is the jaw reflex test, which tests the integrity of the trigeminal nerve (CN V), which is a cranial nerve, not a ligament. 3. The test in the photograph is the jaw reflex test, which tests the integrity of the trigeminal nerve (CN V), which is a cranial nerve. 4. The test in the photograph is the jaw reflex test, which tests the integrity of the trigeminal nerve (CN V), which is a cranial nerve, not a peripheral nerve.

The test shown in the photograph should be used for assessment of which of the following structures? 1. Cartilage 2. Ligament 3. Cranial nerve 4. Peripheral nerve

Correct Answer: 4 1. A pulse oximeter is used to determine oxygen saturation and is an indicator of oxygenation/perfusion in the lungs (p. 52). It is not the best method for determining exercise intensity. 2. The Karvonen formula uses a patient's resting heart rate and should not be used to determine exercise intensity because a patient who is taking metoprolol will have a blunted heart rate response. Metoprolol is a beta-blocker that lowers the maximum heart rate and, therefore, lowers the target heart rate zone. (p. 47) 3. Heart rate should not be used to determine exercise intensity because a patient who is taking metoprolol will have a blunted heart rate response. Metoprolol is a beta-blocker that lowers the maximum heart rate and, therefore, lowers the target heart rate zone. (p. 48) 4. Metoprolol is a beta-blocker that lowers the maximum heart rate and, therefore, lowers the target heart rate zone. The rating of perceived exertion is the most appropriate method to determine exercise intensity since it is not affected by metoprolol. Rating of perceived exertion is a subjective rating of intensity of exertion used to quantify effort during exercise. (p. 48)

To determine exercise intensity for a patient who is taking metoprolol (Lopressor), which of the following methods is MOST appropriate to use? 1. Pulse oximetry 2. Karvonen formula 3. Percentage of maximum heart rate 4. Rating of perceived exertion

Correct Answer: 3 1. The Tinel test at the wrist is performed by light percussions of the median nerve distally to proximally. The point at which paresthesia is elicited is the level of axonal regrowth. The test is used to determine the presence of carpal tunnel syndrome, not paratenonitis in the thumb (1st digit). 2. With the Allen test, the patient is asked to close and open the hand several times as quickly as possible and then squeeze the hand tightly. The examiner's thumb and index finger (1st and 2nd digits) are placed over the radial and ulnar arteries, compressing them. The patient then opens the hand while pressure is being maintained over the arteries. One artery is tested by releasing the pressure and seeing if the hand flushes. Then the other artery is released. Both hands should be tested for comparison. This test determines the patency of the radial and ulnar arteries and determines which artery provides the major blood supply to the hand. The test does not examine paratenonitis of the thumb (1st digit). 3. The Finkelstein test is used to determine the presence of de Quervain or Hoffmann disease, a paratenonitis of the thumb (1st digit). 4. The Phalen test consists of the examiner flexing the patient's wrists maximally and holding this position for 1 minute by pushing the patient's wrists together. A positive test result is indicated by tingling in the thumb, index finger, and middle finger (1st, 2nd, and 3rd digits) and lateral half of the ring finger (4th digit) and is indicative of carpal tunnel syndrome caused by pressure on the median nerve, not paratenonitis of the thumb (1st digit).

To determine if paratenonitis is present in the thumb (1st digit), which of the following tests is MOST appropriate to perform? 1. Tinel 2. Allen 3. Finkelstein 4. Phalen

Correct Answer: 4 1. Face validity is based on the validation of a test without comparison to an already validated test. 2. Construct validity is based on abstract concepts and is not observable or measurable. 3. Content validity of a test is measured to determine the extent of coverage of a concept. It is not determined by comparison with a reliable/valid (gold) standard. 4. Criterion-related validity of a new tool is tested by using practical and objective comparisons to a reliable/valid (gold) standard measure already in use.

What form of validity is measured by comparing results obtained with a test to results obtained using an already well-established and validated tool? 1. Face 2. Construct 3. Content 4. Criterion-related

Correct Answer: 1 1. A negative correlation shows an inverse direction, and a correlation of 0.32 is fair in strength. 2. A correlation of 0.32 is fair in strength, and a direct relationship would have a positive correlation. 3. The direction of the correlation is inverse, but a correlation of 0.32 is fair in strength, not "good." 4. A correlation of 0.32 indicates a relationship that is fair in strength, and a direct relationship would have a positive correlation.

What is the BEST characterization of the relationship between the score on the Functional Reach assessment (FR) and the score on the Fear-Avoidance Beliefs Questionnaire (FABQ) if the correlation is -0.32? 1. Fair in strength, inverse in direction 2. Excellent in strength, direct in direction 3. Good in strength, inverse in direction 4. No relationship, direct in direction

Correct Answer: 4 1. The photograph shows the Bakody sign. If arm pain and referred symptoms into the arm are relieved by placing the arm on the head, it is indicative of a disc lesion at the C4 or C5 level (Magee, pp. 185-186). Since the pain is increased in this position, it is not likely that a C5 disc lesion is present. 2. Dural irritability would be assessed with cervical distraction with the patient in seated position, not with the maneuver shown in the photograph (Dutton, pp. 1293-1294). 3. The position shown in the photograph puts the ulnar nerve in a partly stretched position, since there are components of ulnar nerve stretching in this position, not radial nerve stretching (Dutton, p. 456). To stress the radial nerve, the arm would have to be abducted and medially (internally) rotated with elbow extension and wrist and thumb (1st digit) flexion (Dutton, p. 455). 4. With thoracic outlet syndrome, the position shown in the photograph would increase symptoms, since it would increase pressure on the interscalene triangle (Dutton, p. 1322). The patient would have pain when the arm is in abduction and lateral (external) rotation (Magee, p. 344), an arm position is similar to the arm position for the hyperabduction test, Roos test, and Wright test (Dutton, pp. 1299-1300).

When a patient assumes the position shown in the photograph, pain increases in the raised upper extremity. The patient MOST likely has which of the following conditions? 1. C5 disc lesion 2. Dural irritability 3. Radial nerve irritation 4. Thoracic outlet syndrome

Correct Answer: 2 1. The general order for donning personal protective equipment is gown, then face mask or respirator, then goggles or face shield, then gloves. The gown is the first piece of equipment applied. 2. The general order for donning personal protective equipment is gown, then face mask or respirator, then goggles or face shield, then gloves. Gloves are the last piece of equipment applied. 3. The general order for donning personal protective equipment is gown, then face mask or respirator, then goggles or face shield, then gloves. Goggles are the third piece of equipment applied. 4. The general order for donning personal protective equipment is gown, then face mask or respirator, then goggles or face shield, then gloves. The respirator is the second piece of equipment applied.

When donning personal protective equipment prior to working with a patient who has tuberculosis, which of the following items should be applied LAST? 1. Gown 2. Gloves 3. Goggles 4. N-95 respirator

Correct Answer: 4 1. Pitting is generally more pronounced in the early stages of lymphedema (p. 73). 2. If lymphedema management starts in Stage 3, reduction of lymphedema can still be expected; it may just take longer (p. 74). 3. There is no specific period of time for each stage (p. 72). 4. There is no cure or permanent remedy for lymphedema (p. 71).

When educating a patient regarding the prognosis of lymphedema, which of the following concepts is MOST appropriate to communicate? 1. Pitting becomes more pronounced in the later stages of the condition. 2. Management strategies started in the later stages of the condition are ineffective. 3. The condition progresses in stages that develop at regular intervals. 4. Management strategies exist but are not a permanent cure.

Correct Answer: 3 1. Stage 1 pressure injuries are characterized by nonblanchable erythema of intact skin. In this scenario, the skin is not intact. 2. Stage 2 pressure injuries are characterized by partial-thickness skin loss involving the epidermis, dermis, or both (e.g., abrasion, blister, or shallow crater). 3. Stage 3 pressure injuries are characterized by full-thickness skin loss involving damage to or necrosis of subcutaneous tissue that may extend down to but not through underlying fascia (deep crater with or without undermining). 4. Stage 4 pressure injuries are characterized by full-thickness skin loss with extensive destruction, tissue necrosis, or damage to muscle, bone, or supporting structures (e.g., tendon or joint capsule).

When examining a patient's pressure injury, a physical therapist notes that in the area of the wound, the patient has complete loss of skin and intact underlying fascia. The therapist should recognize this as a: 1. Stage 1 wound. 2. Stage 2 wound. 3. Stage 3 wound. 4. Stage 4 wound.

Correct Answer: 4 1. Cervical rotation follows the same arthrokinematic motions as side bending; therefore, side bending to the right would be limited. Right side bending and rotation mechanics include a superoanterior glide of the left superior facet and a posteroinferior glide of the right facet, which would be restricted with decreased cervical motion and translation of the mid cervical spine to the left. Flexion requires a superoanterior glide of both superior facets, not just one; therefore, a deviation to the side that is not gliding superiorly is more likely. (pp. 1267-1268) 2. Cervical rotation follows the same arthrokinematic motions as side bending; therefore, side bending to the right would be limited. Right side bending and rotation mechanics include a superoanterior glide of the left superior facet and a posteroinferior glide of the right facet, which would be restricted with decreased cervical motion and translation of the mid cervical spine to the left. Extension requires an inferoposterior glide and approximation of the superior facets of the zygapophyseal joints. This motion would not be limited in the patient described in the stem, since backward bending does not involve a superoanterior glide, which is a component of the restriction described. (pp. 1267-1268) 3. Cervical rotation follows the same arthrokinematic motions as side bending; therefore, side bending to the right would be limited, not side bending to the left. Side bending mechanics include a superoanterior glide of the left superior facet and a posteroinferior glide of the right facet, which would be restricted with decreased cervical motion and translation of the mid cervical spine to the left. (p. 1268) 4. Cervical rotation follows the same arthrokinematic motions as side bending; therefore, side bending to the right would be limited. Side bending mechanics include a superoanterior glide of the left superior facet and a posteroinferior glide of the right facet, which would be restricted with decreased cervical motion and translation of the mid cervical spine to the left. (p. 1268)

When performing active cervical rotation range of motion, a patient moves with limited right cervical rotation. Translation testing of the mid-cervical spine reveals no restrictions to the right and moderate restrictions to the left. Which of the following cervical motions will MOST likely be restricted in the patient? 1. Forward bending 2. Backward bending 3. Side bending to the left 4. Side bending to the right

Correct Answer: 3 1. Debris in the right posterior semicircular canal produces symptoms of transient upbeating nystagmus and/or right ocular torsion. 2. Debris in the right superior semicircular canal produces symptoms of persistent downbeating nystagmus and/or right ocular torsion. 3. The canalith repositioning maneuver for the left posterior semicircular canal is performed to move free-floating debris in the posterior semicircular canal back into the vestibule, thus resolving the signs and symptoms of nystagmus and dizziness. Debris in the left posterior semicircular canal produces symptoms of transient upbeating nystagmus and/or left ocular torsion. 4. Debris in the left superior semicircular canal produces symptoms of persistent downbeating nystagmus and/or left ocular torsion.

When treating a patient who has transient upbeating nystagmus and left ocular torsion, canalith repositioning maneuvers should be targeted to which of the following structures? 1. Right posterior semicircular canal 2. Right superior semicircular canal 3. Left posterior semicircular canal 4. Left superior semicircular canal

Correct Answer: 4 1. The symmetrical biphasic waveform is not a polar current and thus cannot be used in wound healing per the theory of galvanotaxis (pp. 271-272). For symmetrical biphasic waveforms, the polarity is irrelevant since polar effects are not demonstrated. A frequency of 35 pps is more relevant for a nonfatiguing muscle stimulation. The symmetrical biphasic waveform is primarily used in neuromuscular stimulation for functional support or muscle strengthening. (p. 263) 2. Interferential current is not a polar current and thus cannot be used in wound healing per the theory of galvanotaxis (pp. 271-272). For interferential current, the polarity is irrelevant since polar effects are not demonstrated. Interferential current is thought to be more comfortable and is used primarily for pain control. (pp. 222-223) 3. High-voltage pulsed current is a polar current that is used for wound healing. However, the positive electrode is used when the wound is clean and not when it is infected. The low pulse frequency has not been demonstrated to cause healing and would not be comfortable to the patient. (pp. 272, 278) 4. The principle behind wound healing with electrical stimulation is galvanotaxis. The current's polarity introduced into the wound attracts cells that promote healing. Thus, a generator that has polarity must be used. High-voltage pulsed currents are polar currents. The other stimulators are not polar because they have a counter-pulse in the opposite direction that cancels out any polar effects. The negative electrode will attract neutrophils and is used in infected wounds. The positive electrode attracts macrophages and epidermal cells and is used in treating noninfected wound states. The frequency of 100 pps provides a continuous, comfortable current and has been demonstrated to promote healing. (pp. 271-272, 278)

When using electrical stimulation to treat a patient's nonhealing, infected wound, which of the following waveforms and parameters will be MOST helpful in facilitating wound closure? 1. Symmetrical biphasic waveform, 35 pps 2. Interferential current waveform, 100 beats/second 3. High-voltage pulsed current waveform, positive electrode in wound, 4 pps 4. High-voltage pulsed current waveform, negative electrode in wound, 100 pps

Correct Answer: 1 1. For best biofeedback results, electrode placement should be as close to the muscle as possible (Prentice). Hook grasp requires finger flexion (Lippert, p. 215). The finger flexors are located proximal to the anterior forearm (Lippert, pp. 201-202). 2. The proximal posterolateral forearm is the location of the finger extensors (Lippert, pp. 204-205). 3. No muscles originate from the distal anterior, medial forearm area (Lippert, pp. 201-206). 4. The distal posterolateral forearm is the location of thumb and index finger (1st and 2nd digit) extensors (Lippert, pp. 203-205).

Where on the forearm should a physical therapist place electrodes for biofeedback therapy in order to facilitate hook grasp? 1. Proximal anteromedial 2. Proximal posterolateral 3. Distal anteromedial 4. Distal posterolateral

Correct Answer: 2 1. T-scores of -2.5 and lower indicate osteoporosis. 2. T-scores falling in the range -1.0 to -2.5 indicate low bone mass, which is osteopenia. 3. T-scores of -1.0 and higher indicate normal bone mass. 4. T-scores of -1.0 and higher indicate normal bone mass.

Which of the following T-scores for bone density indicates that the patient has osteopenia? 1. -3.0 2. -2.0 3. +2.0 4. +3.0

Correct Answer: 4 1. Frequent feedback is detrimental when learning a skill and would not be done first when teaching a new skill (p. 263). 2. Massed practice is less effective than distributed practice and would not be done first (p. 255). 3. It may be best to tailor written instructions to fit the patient's learning needs (p. 212). 4. It is best to identify and use the patient's preferred learning style (p. 24).

Which of the following actions should be done FIRST when teaching a new motor skill? 1. Provide frequent verbal feedback on performance. 2. Utilize massed practice. 3. Distribute written instructions with standardized formatting. 4. Identify and utilize learning preferences.

Correct Answer: 3 1. After open-heart surgery, such as coronary artery bypass surgery, patients are encouraged to gradually increase walking, with a goal of 30 minutes of ambulation 1-2 times/day by 4-6 weeks post-surgery (p. 488). 2. Elevating both arms overhead results in the least sternal separation (p. 487). 3. Patients who are at risk for sternal instability (e.g. following open-heart surgery) demonstrate the greatest separation when pushing up from a chair (p. 487). 4. Postsurgical patients benefit from splinting the incision with a hand or pillow when laughing, coughing, or sneezing (p. 488).

Which of the following activities is CONTRAINDICATED for a patient who is 2 weeks post coronary artery bypass surgery? 1. Walking for 15 minutes 2 times/day 2. Performing bilateral arm lifts using 2-lb (0.9-kg) weights 3. Pushing up from the armrest of a chair during sit-to-stand transfers 4. Holding a pillow against the sternal incision when coughing or sneezing

Correct Answer: 1 1. Trace (1/5) strength in the anterior tibialis indicates the ankle is unable to move into dorsiflexion, resulting in foot drop (Avers, pp. 284-285). The posterior leaf spring is designed to help lift the foot for adequate clearance during the swing phase of gait (O'Sullivan, p. 1293). 2. A floor (ground) reaction orthosis has an anterior shell that provides a posteriorly directed force to resist knee flexion during stance phase (O'Sullivan, p. 1296). This patient does not need assistance with knee control because the patient has weakness of the tibialis anterior, which affects dorsiflexion of the ankle (Avers, p. 284). 3. A patellar tendon-bearing orthosis is designed to lessen the load on the foot (O'Sullivan, p. 1296). This patient requires assistance with ankle dorsiflexion due to tibialis anterior weakness (Avers, p. 284). 4. A solid ankle-foot orthosis limits all foot and ankle motion (O'Sullivan, p. 1294). This patient requires only assistance with ankle dorsiflexion due to tibialis anterior weakness (Avers, p. 284).

Which of the following ankle-foot orthoses is MOST appropriate for a patient who exhibits Trace (1/5) strength of the tibialis anterior muscle? 1. Posterior leaf spring 2. Floor reaction 3. Patellar tendon-bearing 4. Solid ankle

Correct Answer: 4 1. Use of a hip strategy is more appropriate for activities such as tandem and single limb stance (Umphred). 2. Use of a hip strategy is more appropriate for activities such as tandem and single limb stance (Umphred). 3. Small perturbations backward (anterior to posterior) would create a forward weight-shift, then a return to midline in response (O'Sullivan). 4. The ankle strategy is appropriate for small perturbations on a stable surface (Umphred). Small perturbations forward (posterior to anterior) would create a backward weight-shift, then a return to midline in response (O'Sullivan).

Which of the following automatic postural responses will a patient MOST likely use to maintain equilibrium following a small perturbation on a stable surface from a posterior to anterior direction? 1. Hip strategy to control sway and move the body anterior to midline 2. Hip strategy to control sway and move the body posterior to midline 3. Ankle strategy to control sway and move the body anterior to midline 4. Ankle strategy to control sway and move the body posterior to midline

Correct Answer: 4 1. Spinal cord lesions at the level of S2 and below lead to bladder areflexia and dysfunction of the external sphincter. Surgical interventions for neurogenic bladder exist; sacral nerve modulation is used for incomplete lesions. (Goodman, pp. 991-992) The patient has a complete lesion. 2. Spinal cord lesions at the level of S2 and below lead to bladder areflexia and dysfunction of the external sphincter (Goodman, p. 991). S2-S4 innervate muscles in the perineum and external sphincter; therefore, complete loss of innervation would lead to paralysis of the respective musculature (Moore). Biofeedback is for muscle identification would be an inappropriate intervention for this patient. 3. Spinal cord lesions at the level of S3 and below lead to bladder areflexia and dysfunction of the external sphincter (Goodman, p. 991). S2-S4 innervate muscles in the perineum and external sphincter; therefore, complete loss of innervation would lead to paralysis of the respective musculature (Moore). Strengthening exercises would be an inappropriate intervention for this patient. 4. Spinal cord lesions at the level of S2 and below lead to bladder areflexia and dysfunction of the external sphincter. Bladder tone is preserved, but bladder compliance decreases with time. Catheterization is a commonly employed intervention to avoid excessive bladder distention. (Goodman, p. 991)

Which of the following bladder management techniques is MOST likely to be used for a patient with bladder dysfunction due to a cauda equina lesion? 1. Sacral nerve modulation 2. Pelvic floor biofeedback 3. Pelvic floor strengthening exercises 4. Intermittent catheterization

Correct Answer: 2 1. Common moles and other normal skin changes usually have smooth, even borders or edges. Malignant melanomas have uneven, notched borders. 2. A single lesion with more than one shade of black, brown, or blue may be a sign of malignant melanoma. 3. Round, symmetric skin lesions, such as common moles, freckles, and birthmarks, are considered normal. 4. The average mole is less than 0.25 inch (6.3 mm) in diameter. Anything larger than this should be inspected carefully.

Which of the following characteristics of a skin lesion is MOST likely to require referral to a physician? 1. Smooth and even borders 2. Black and brown coloration 3. Round and symmetrical shape 4. 0.20 inches (5 mm) in diameter

Correct Answer: 4 1. An individual with hypoglycemia would exhibit pallor, not a flushed appearance. 2. An individual with hyperglycemia would potentially display deep respirations, while an individual with hypoglycemia would likely experience shallow respirations. 3. An individual with hyperglycemia would experience a diminished pulse response, while an individual with hypoglycemia would experience tachycardia. 4. With the onset of hypoglycemia, an individual will experience a sudden change in appearance from normal coloration to pallor.

Which of the following clinical manifestations MOST indicates the onset of hypoglycemia? 1. Flushed appearance 2. Deep respirations 3. Slow pulse 4. Pallor

Correct Answer: 2 1. Resistance training is an essential component of an exercise program for a patient who has osteoporosis (Moore) and is lacking from this option. Balance training, although helpful in preventing falls, is not most beneficial for maintaining bone density. 2. This question presents two comparisons and asks which is better: 1) treadmill walking or swimming and 2) resistance or balance training. The correct answer is treadmill walking and resistance training. Walking is a beneficial weight-bearing aerobic activity. Resistance training is important to maintain bone mineral density or prevent loss of bone mineral density. (Moore) 3. Swimming, because of the lack of a weight-bearing component, has limited value for a person who has osteoporosis (Goodman). Balance training, although helpful in preventing falls, is not most beneficial for maintaining bone density. 4. Swimming, because of the lack of a weight- bearing component, has limited value for a person who has osteoporosis (Goodman).

Which of the following combinations of activities would be MOST beneficial for maintaining bone density in a patient with osteoporosis? 1. Treadmill walking and balance training 2. Treadmill walking and resistance training 3. Swimming and balance training 4. Swimming and resistance training

Correct Answer: 1 1. When communicating with a person who has difficulty speaking, physical therapists should intensify their listening skills and provide feedback to the individual to indicate understanding. 2. A person who is hearing impaired may need to have tactile or visual cues, but this would not be most appropriate for a person who has difficulty speaking. 3. When communicating with a person who has difficulty speaking, the physical therapist should use questions that require brief responses. 4. The physical therapist should speak directly to the patient rather than to a companion or family member.

Which of the following communication strategies is MOST appropriate for a physical therapist to use with a patient who has dysarthria following a recent cerebrovascular accident? 1. Provide feedback indicating understanding of the patient's speech. 2. Utilize an increased level of tactile or visual cueing. 3. Use open-ended questions to elicit responses. 4. Speak and interact with the patient's family rather than the patient.

Correct Answer: 3 1. Uveitis is commonly found in patients who have ankylosing spondylitis (p. 1134). 2. Urethritis is commonly found in patients who have Reiter syndrome (p. 1344). 3. Skin rashes, fever, fatigue, malaise, photosensitivity, dyspnea, cough, and peripheral neuropathies are all common findings in patients who have systemic lupus erythematosus (pp. 307-308). 4. Psoriasis is commonly seen in patients who have psoriatic arthritis (pp. 1341-1342).

Which of the following conditions is MOST likely to be associated with systemic lupus erythematosus? 1. Uveitis 2. Urethritis 3. Photosensitivity 4. Psoriasis

Correct Answer: 1 1. A systematic review of literature entails a secondary analysis of individual studies with similar characteristics in order to generate a combined conclusion (p. 338). 2. A systematic review of literature entails a secondary analysis of individual studies with similar characteristics in order to generate a combined conclusion (p. 338). 3. A meta-analysis, not a systematic literature review, is an aggregation of raw data from multiple studies to increase the sample size and generate a conclusion based on a larger subject population (pp. 339-341). 4. Generation of a hypothesis is the initial step in research and can be based on conclusions from other research. Aggregating data from multiple studies does not lead to new hypotheses but can generate a combined conclusion from a larger population size (e.g., meta-analysis). (pp. 84-85)

Which of the following definitions BEST describes a systematic literature review? 1. Secondary analysis of individual studies with similar characteristics 2. Secondary analysis of individual studies with divergent characteristics 3. Aggregation of raw data from multiple studies to increase the sample size 4. Aggregation of raw data from multiple studies to generate new hypotheses

Correct Answer: 4 1. This pattern describes bradypnea. Bradypnea is associated with impairment of the respiratory control center and may occur with an increased intracranial pressure, drug intake, or metabolic disorder. (p. 57) 2. This pattern describes tachypnea. Tachypnea is associated with respiratory insufficiency and fever as the body attempts to rid itself of excess heat. (p. 57) 3. This pattern is characteristic of Biot respirations (p. 58). 4. This is a typical Cheyne-Strokes respiratory pattern, which is an irregular respiration pattern characterized by a period of apnea followed by gradually increasing depth and frequency of respirations (pp. 57-58). This breathing pattern is often observed with depression of the cerebral hemisphere (e.g., coma), in basal ganglia disease, and occasionally with congestive heart failure.

Which of the following descriptions BEST depicts the Cheyne-Stokes respiratory pattern? 1. Regular respiration pattern characterized by a rate of less than 10 breaths/minute 2. Regular respiration pattern characterized by a rate of more than 24 breaths/minute 3. Irregular respiration pattern characterized by highly variable respiratory depth and intermittent periods of apnea 4. Irregular respiration pattern characterized by a period of apnea followed by gradually increasing depth of respirations

Correct Answer: 1 1. Patients who have sustained a C1-C4 spinal cord injury are dependent in bed mobility and transfers and use a power wheelchair independently as the primary means of mobility (p. 470). 2. Patients who have sustained a complete C5 spinal cord injury should be able to achieve transfers and bed mobility with assistance, or at maximum are dependent for transfers and bed mobility. Power wheelchair mobility is the recommended mode of mobility, with modified independence as the highest level. (p. 471) 3. Patients who have sustained a complete C6 spinal cord injury should be able to perform transfers with some assistance and are likely to be able to perform bed mobility independently with assistance needed only for leg management at times. Manual wheelchair mobility will be possible over level surfaces, but assistance will be required over unlevel surfaces such as rough terrain and curbs. (p. 472) 4. Patients who have sustained a complete C7-C8 spinal cord injury should be able to perform transfers with modified independence and may not need a transfer board. They will be able to perform bed mobility without assistance and wheelchair mobility over most surfaces, including ramps and rough terrain. (pp. 473, 474)

Which of the following descriptions BEST represents the highest potential of function for a patient who sustained a C4 spinal cord injury (ASIA Impairment Scale A)? 1. Level transfers with total assistance, bed mobility with total assistance, power wheelchair mobility 2. Level transfers with moderate assistance, bed mobility with maximum assistance, power wheelchair mobility with modified independence 3. Unlevel transfers with assistance, bed mobility with minimal assistance, manual wheelchair mobility over level surfaces 4. Unlevel transfers without assistance, bed mobility without assistance, manual wheelchair mobility over unlevel surfaces with modified independence

Correct Answer: 2 1. A foam dressing is absorptive but also creates an occlusive environment for moist wound healing. In the case of infection, a less occlusive dressing would be a better choice. (pp. 563, 589-590) 2. An alginate dressing is best to use in this case because this type of dressing provides both hemostasis and is appropriate for use over an infected wound (pp. 565, 589). 3. A transparent film is not the best dressing to use in this case because it does not provide hemostasis or infection control. Films are more appropriate for friction reduction. (pp. 563, 589) 4. A hydrocolloid dressing is not the best dressing to use with an infected wound that also requires hemostasis because it is the most occlusive dressing type. An alginate dressing is better for hemostasis. (pp. 564, 589-590)

Which of the following dressings is MOST appropriate to use with an infected wound that also requires hemostasis? 1. Foam 2. Alginate 3. Transparent film 4. Hydrocolloid

Correct Answer: 3 1. Standing with one foot in front of the other with eyes closed is an intervention to increase reliance on vestibular input to maintain balance and is not a vestibuloocular reflex test (p. 938). 2. Assessment of ability to move eyeballs laterally is used to test the integrity of the abducent nerve (CN VI), not the vestibuloocular reflex (p. 151). 3. The vestibuloocular reflex keeps the visual environment in focus during rapid head movements. The vestibuloocular reflex is essential for gaze stabilization during high-frequency, high-velocity, and high-acceleration head movements. (p. 921) 4. Cervical extension with rotation is a position in the Dix-Hallpike maneuver and is not used in testing of the vestibuloocular reflex (p. 927).

Which of the following examination activities is used in testing of the vestibuloocular reflex? 1. Tandem stance with eyes closed 2. Lateral movement of the eyes 3. Rapid movement of the head 4. Cervical extension with rotation

Correct Answer: 2 1. Exercise should be avoided during peak insulin times (Goodman; Porcari). 2. Patients who have diabetes should exercise regularly and consistently (Goodman; Porcari). 3. Below 100 mg/dL (5.6 mmol/L) is the cautionary range for exercise (Goodman; Porcari). 4. Above 250 mg/dL (13.9 mmol/L) is the cautionary range for exercise (Goodman; Porcari).

Which of the following exercise guidelines is MOST appropriate for a patient who has diabetes? 1. Exercise during peak insulin times. 2. Exercise at the same time each day. 3. Avoid exercise if the blood glucose level is less than 150 mg/dL (8.3 mmol/L). 4. Avoid exercise if the blood glucose level is greater than 200 mg/dL (11.1 mmol/L).

Correct Answer: 2 1. Lap swimming will not typically promote context-dependent responses. 2. Exercises such as karate and other martial arts, dancing, and walking outdoors promote context-dependent responses and are important to incorporate in treatment of patients who have movement disorders because these exercises require patients to adjust the response to a specific task. 3. Unsupervised treadmill walking will not typically promote context-dependent responses. 4. Riding a stationary bike will not typically promote context-dependent responses.

Which of the following exercises promotes context-dependent responses in a patient who has Parkinson disease? 1. Lap swimming 2. Walking outdoors 3. Unsupervised treadmill walking 4. Riding a stationary bicycle

Correct Answer: 3 1. Exercises that are normally performed from a prone position should be modified in pregnancy. Standing push-ups will help maintain upper limb strength and promote good posture. 2. Modified squatting is incorrect because it is indicated for a pregnant woman. This exercise helps maintain lower limb strength for good body mechanics and also helps stretch the perineal area for increased flexibility during the delivery process. 3. Bilateral straight-leg raising places a great deal of stress on the abdominal muscles and low back. It may cause injury or diastasis recti and should not be included in a physical therapy program for pregnant women. 4. Quadruped pelvic tilt will help with correct posture and maintenance of mobility of the lumbar spine, as well as help to maintain the strength of abdominal muscles.

Which of the following exercises would be CONTRAINDICATED during pregnancy? 1. Standing push-ups 2. Modified squatting 3. Bilateral straight leg raises 4. Quadruped pelvic tilts

Correct Answer: 4 1. Although family involvement in the patient's care is important, there is no evidence that family involvement improves outcomes specifically in upper extremity functional tasks. 2. There is no evidence to support the use of blocked practice specifically to improve outcomes in upper extremity functional tasks. 3. Although the patient's cognitive understanding of the impairments and the intervention program is important, there is no evidence that the patient's understanding improves outcomes specifically in upper extremity functional tasks. 4. Research evidence supports that patients who possess active wrist and finger extension have improved upper extremity functional outcomes after a cerebrovascular accident.

Which of the following factors BEST predicts improvements in upper extremity functional outcomes for a patient following a cerebrovascular accident? 1. Family involvement in the patient's care 2. Use of blocked practice during intervention 3. Patient's cognitive understanding of the impairments and intervention program 4. Presence of active wrist and finger extension

Correct Answer: 4 1. With advanced age, functional liver tissue diminishes and hepatic blood flow decreases, not increases. 2. With advanced age, functional liver tissue diminishes and hepatic blood flow decreases. Consequently, the capacity of the liver to break down and convert drugs and their metabolites declines, not increases. 3. As people age, there is a decrease in lean body mass and an increase in the proportion of body fat. 4. A decrease in lean body mass and an increase in the proportion of body fat results in a decrease in body water. As a result, water-soluble drugs have a lower volume of distribution, which speeds up onset of action and raises peak concentration.

Which of the following factors MOST contributes to adverse reactions to medications in aging adults? 1. Increase in hepatic blood flow 2. Increase in metabolic activity 3. Decrease in proportion of body fat 4. Decrease in total body water

Correct Answer: 4 1. Traction is contraindicated in the presence of instability, which is implied by the diagnosis of a labral tear (p. 378). 2. Ultrasound with a 3-MHz frequency will only penetrate superficial tissue and will only cover a small area; therefore, it is not appropriate to treat the injury in question (p. 184). 3. Cryotherapy reduces chemical mediators being released in order to modulate pain. Ice packs are helpful in the initial, or acute, stage of healing but are not as effective in later stages of healing. (p. 4). 4. Sensory-level electrical stimulation can cover a large area and is effective for treating chronic pain (p. 7).

Which of the following modalities is MOST appropriate to administer to a patient who has hip joint pain secondary to a labral tear that occurred 6 months ago? 1. Traction 2. 3-MHz thermal ultrasound 3. Ice pack 4. Sensory-level electrical stimulation

Correct Answer: 4 1. The use of side-rails is a restraint. Their use on the bed of a mobile person may lead to a number of negative consequences, such as increasing the distance the patient could fall from the bed, creating an obstruction of vision, and creating a sense of being trapped. The use of side-rails with a patient who has dementia is a restraint and requires a physician's order. 2. Lap cushions, trays, and seat belts are considered restraints if the patient is unable to remove them independently. 3. Geriatric recliners are considered restraints when they restrict a patient's normal mobility. 4. The use of restraints has become a concern for nursing home caregivers, who must comply with Medicare guidelines and foster prevention of elder abuse. An electronic monitoring device is not a restraint. This option would facilitate safety through improved supervision and would allow the patient to maintain functional mobility.

Which of the following fall prevention strategies is MOST appropriate for a resident of a nursing home who has dementia, poor balance, and often wanders? 1. Place the patient in bed, with the side-rails up and secured. 2. Place the patient in a wheelchair with a seat belt that the patient is unable to remove independently. 3. Seat the patient in a geriatric recliner to reduce the likelihood of wandering. 4. Use an electronic monitor that will remotely alert staff when the patient gets out of bed.

Correct Answer: 1 1. Swelling that feels hard with palpation describes brawny edema. Brawny edema is associated with Stage 2 (spontaneously irreversible) lymphedema. 2. Edema that forms a dimple when fingertip pressure is removed describes pitting edema, which is associated with Stage 1 (reversible) lymphedema, which is less severe than Stage 2 lymphedema. 3. Edema reduced by elevation is associated with Stage 1 (reversible) lymphedema, which is less severe than Stage 2 lymphedema. 4. A Stemmer sign, which is the inability to pinch a fold of skin over the dorsum of the foot, is an indication of Stage 2 or Stage 3 lymphedema. Therefore, the ability to pinch a fold of skin indicates that there is either no edema or that Stage 1 lymphedema is present.

Which of the following findings is MOST associated with Stage 2 lymphedema? 1. Swelling that feels hard with palpation 2. Edema that forms a dimple when fingertip pressure is removed 3. Signs and symptoms that are improved with elevation of the limb 4. Skin on the dorsum of the foot that can be pinched in a fold

Correct Answer: 1 1. Excursion is decreased in patients who have chronic obstructive pulmonary disease due to hyperinflation of the chest and a resultant flattened diaphragm. Normal excursion of the diaphragm is 1.2 to 2 inches (3 to 5 cm); therefore, 0.4 to 0.8 inch (1 to 2 cm) would be below the normal excursion value. 2. Excursion is decreased in patients who have chronic obstructive pulmonary disease due to hyperinflation of the chest and a resultant flattened diaphragm. 3. Normal excursion of the diaphragm is 1.2 to 2 inches (3 to 5 cm). 4. Excursion is decreased in patients who have chronic obstructive pulmonary disease due to hyperinflation of the chest and a resultant flattened diaphragm.

Which of the following findings is MOST commonly associated with patients who have chronic obstructive pulmonary disease? 1. Below normal diaphragmatic excursion of 0.4 to 0.8 inch (1 to 2 cm) 2. Above normal diaphragmatic excursion of 0.4 to 0.8 inch (1 to 2 cm) 3. Below normal diaphragmatic excursion of 1.2 to 2 inches (3 to 5 cm) 4. Above normal diaphragmatic excursion of 1.2 to 2 inches (3 to 5 cm)

Correct Answer: 2 1. Having a patient find an object among similarly shaped objects is a better test for form discrimination rather than figure-ground discrimination, as it assesses whether subtle differences in shape can be perceived (p. 1208). Finding a toothbrush amongst an array of utensils of various shapes and sizes would be a better test for figure-ground discrimination. 2. An impairment in figure-ground discrimination is the inability to visually distinguish a figure from the background in which it is embedded. A functional test that can be given to the patient to assess figure-ground discrimination is to ask a patient to point out a white button on a white shirt. Compensatory techniques to be used with patients who lack figure-ground perception are placing red tape over the Velcro strap of the shoe to aid the patient in locating it or using bright red tape to mark the edges on stairs. (pp. 1207-1208) 3. Tactile agnosia is the inability to recognize forms by handling them, although tactile sensation may be intact. If a patient is handed an object while the patient's vision is occluded, the patient will fail to recognize the object. This would not be the best test for an impairment of figure-ground discrimination. (p. 1213) 4. A patient who has depth and distance perception problems may have difficulty grasping an object. The impaired patient will overshoot or undershoot the object. This would not be the best test for an impairment of figure-ground discrimination. (pp. 1210-1211)

Which of the following functional tests would be MOST appropriate to verify that a patient lacks figure-ground discrimination? 1. Have the patient find an object, such as a toothbrush, among similarly shaped objects. 2. Have the patient locate a white button on a white shirt. 3. Ask patient to identify an object, such as a key, with eyes closed. 4. Ask the patient to reach for a bright blue paper located on a white desk.

Correct Answer: 3 1. The recommended ramp length is 12 inches (30 cm) for each inch (2.5 cm) of rise (p. 324). 2. Wheelchairs can cross thresholds of up to 0.5 inch (1.3 cm) (p. 326). 3. Wheelchairs require a minimum of 32 to 34 inches (81 to 86 cm) in doorway width and would be unable to go through a door 28 inches (71 cm) in width (p. 326). 4. A door opening space of 5 feet by 5 feet (1.5 m x 1.5 m) is sufficient for a door opening that swings toward the patient (p. 325)

Which of the following household measurements requires modification to allow proper wheelchair accessibility? 1. A ramp of 5 ft (1.5 m) with a 5 in (13 cm) rise 2. A threshold of 0.5 in (1.3 cm) 3. A doorway with a width of 28 in (71 cm) 4. A door opening space of 5 ft x 5 ft (1.5 m x 1.5 m)

Correct Answer: 3 1. Stretching the gluteus maximus to 90° of hip flexion will not improve long sitting, which involves the hamstrings (Umphred). 2. Patients who have a complete T4 injury do not have control of abdominal or low back muscles (O'Sullivan, p. 886). 3. Stretching of the hamstring muscles prevents overstretching of the back during long sitting. Passive low back muscle tightness is important to develop for passive trunk stability. Straight leg raises less than 100° to 110° in long sitting put a passive pull on the pelvis, resulting in posterior pelvic tilt and stretching of the low back. (O'Sullivan, p. 894) 4. Patients who have a complete T4 injury do not have control of abdominal or low back muscles (O'Sullivan, p. 886).

Which of the following interventions is MOST appropriate to improve stability in long sitting for an individual with a complete T4 spinal cord injury? 1. Stretch the gluteus maximus to 90° of hip flexion. 2. Strengthen the abdominal muscles to grade Fair (3/5). 3. Stretch the hamstrings to 110° of straight leg raising. 4. Strengthen the erector spinae muscles to grade Good (4/5).

Correct Answer: 2 1. Cervical flexion at 10° does not achieve the greatest opening at the intervertebral foramina, although cervical flexion in supine position allows for more patient comfort and muscle relaxation than cervical flexion in sitting position. 2. Flexion at 30° results in greater separation of the posterior structures, which include the facet joints and the intervertebral foramina. Cervical flexion in supine position allows for more patient comfort and muscle relaxation than cervical flexion in sitting position. 3. Cervical flexion in sitting position does not allow for as much muscle relaxation, since the head is still in a weight-bearing position. Cervical flexion in supine position allows for more patient comfort and muscle relaxation than cervical flexion in sitting position. Facing the door does allow for more flexion of the cervical region. 4. Cervical flexion in sitting position does not allow for as much muscle relaxation, since the head is still in a weight-bearing position. Cervical flexion in supine position allows for more patient comfort and muscle relaxation than cervical flexion in sitting position. Facing away from the door gives more extension of the cervical region, but this would not open the cervical foramina, which would be necessary for treatment of cervical stenosis.

Which of the following interventions would be BEST for a patient who has cervical stenosis and right upper extremity radicular symptoms? 1. Supine cervical traction in 10° of flexion 2. Supine cervical traction in 30° of flexion 3. Sitting cervical traction facing toward the door 4. Sitting cervical traction facing away from the door

Correct Answer: 1 1. Sever disease is a calcaneal apophysitis and will benefit from stretching to improve flexibility of the gastrocnemius and soleus and use of a heel wedge to decrease the stress and traction of the Achilles insertion. 2. Sever disease affects the Achilles area, not the plantar fascia. 3. Sever disease affects the Achilles area, not the quadriceps tendon. 4. Sever disease affects the Achilles area, not the tibialis posterior muscle.

Which of the following interventions would be MOST appropriate for a child who has Sever disease? 1. Stretch the gastrocnemius and soleus, and use a heel wedge. 2. Stretch the plantar fascia, and use an arch support. 3. Stretch the quadriceps, and use a patellar tendon band. 4. Stretch the tibialis posterior, and use a medial heel wedge.

Correct Answer: 4 1. Use of tuning forks and vibrators would facilitate reeducation of vibratory sense. This sensory function is related to the dorsal column/medical lemniscus system and may not be affected in this patient. 2. Active movement with visual feedback would facilitate reeducation of proprioceptive sense. This sensory function is related to the dorsal column/medical lemniscus system and may not be affected in this patient. 3. Sensory reeducation focuses on all sensory modalities, but these techniques would emphasize reeducation of discriminative touch functions. These sensory functions are related to the dorsal column/medical lemniscus system and may not be affected in this patient. 4. With lesions to the anterolateral system, a patient may exhibit sensory deficits in both light touch and hot/cold discrimination. Failure to distinguish extremes in temperature could result in the patient sustaining thermal injuries. Instruction in techniques to protect against these injuries would be of primary importance.

Which of the following interventions would be MOST appropriate for a patient with a spinal cord lesion to the anterolateral sensory system? 1. Tactile stimulation using tuning forks and vibrators of varying frequencies 2. Active movement using visual feedback for facilitation of position sense 3. Sensory re-education utilizing objects of various sizes, shapes, and textures 4. Patient education concerning protection from hot/cold injuries

Correct Answer: 1 1. The purpose of humeroulnar distraction is to increase flexion (or extension) of the elbow joint. 2. The purpose of humeroradial posterior glide is to increase extension, not flexion, of the elbow joint. 3. The purpose of radioulnar anterior glide is to increase supination of the forearm, not elbow joint flexion. 4. The purpose of radioulnar posterior glide is to increase pronation of the forearm, not elbow joint flexion.

Which of the following joint mobilization techniques would MOST effectively increase elbow joint flexion? 1. Humeroulnar distraction 2. Humeroradial posterior glide 3. Radioulnar anterior glide 4. Radioulnar posterior glide

Correct Answer: 2 1. Stridor is a continuous monophonic high-pitched crowing sound heard during inspiration. It is usually caused by upper airway obstruction. (O'Sullivan, p. 57) 2. High-pitched wheezes are continuous musical sounds of variable pitch and duration that are heard on inspiration, expiration (most common), or both and are usually caused by narrow airways or stenosis (Main; O'Sullivan, p. 449). 3. A pleural rub is inspiratory and expiratory grating, creaking sound like sandpaper or leather being rubbed together, usually due to pleural inflammation (Main). 4. Crackles (rales) are discontinuous, nonmusical, crackling sounds similar in sound to several hairs being rubbed together. It is most often heard on inspiration and usually caused by sudden opening of closed airways or movement of secretions. (O'Sullivan, p. 449)

Which of the following lung sounds heard during auscultation would BEST be described as continuous musical sounds during exhalation for a patient who has difficulty breathing and who has an increased respiratory rate? 1. Stridor 2. High-pitched wheezes 3. Pleural rub 4. Crackles (rales)

Correct Answer: 1 1. Chest wall motion can be assessed using a tape measure with the patient in supine position. The therapist should measure the chest's circumference at the levels of the axillae and the xiphoid process to assess motions of the upper and middle chest, respectively. (O'Sullivan; DeTurk) 2. Lower chest wall excursion measurements should be taken circumferentially at the midpoint between the xiphoid process and the umbilicus, not at the level of the umbilicus (DeTurk). 3. Measuring the distance between the xiphoid and umbilicus during inhalation would be done to gain some understanding of the diaphragm's functioning in this patient, but not as a measure of chest excursion (O'Sullivan). 4. The recommended method to measure chest wall excursion is circumferentially around the thorax, not by using the vertical distance between two anatomical points (DeTurk).

Which of the following methods is MOST appropriate for evaluation of chest excursion in a patient with an incomplete spinal cord injury at the C7 level who is in supine position? 1. Use a tape measure circumferentially at the levels of the axillae and the xiphoid process. 2. Use a tape measure circumferentially at the level of the umbilicus. 3. Measure distance between the xiphoid and umbilicus during inhalation. 4. Measure distance between the xiphoid and umbilicus during an air shift maneuver.

1. The photograph shows the lower trapezius manual muscle test where the patient is placed in prone position with the arm extended diagonally overhead and the shoulder is medially (externally) rotated (p. 339). 2. The rhomboid major would be isolated with the arm at 90° of abduction and medial (internal) rotation of glenohumeral joint (p. 340). 3. The latissimus dorsi is tested with the patient in prone position with the shoulder extended, adducted, and medially (internally) rotated (p. 342). 4. The middle trapezius muscle is tested with the glenohumeral joint at 90° of abduction and laterally (externally) rotated (p. 339).

Which of the following muscles is MOST likely being tested in the photograph? 1. Lower trapezius 2. Rhomboid major 3. Latissimus dorsi 4. Middle trapezius

Correct Answer: 2 1. Tensing of the skin in the gluteal area is due to the gluteal reflex, which is evoked by stroking the back. The gluteal reflex comes from nerve roots of L4-L5, S1-S3. 2. For the cremasteric reflex text, the patient lies in supine position while the examiner strokes the inner side of the upper thigh with a pointed object. The test result is negative if the scrotal sac on the tested side pulls up. Unilateral absence of this response indicates a lower motor neuron lesion between L1 and L2. 3. Contraction of the anal sphincter muscles is due to the superficial anal reflex. The examiner tests the superficial anal reflex by touching the perianal skin. A normal result is shown by contraction of the anal sphincter muscles. This reflex comes from the S2-S4 nerve roots. 4. Movement of the umbilicus down and toward the area being stroked is due to the superficial abdominal reflex. The examiner uses a pointed object to stroke each quadrant of the abdomen of the supine patient in a triangular fashion around the umbilicus. Absence of the reflex (movement of the skin) indicates an upper motor neuron lesion; unilateral absence indicates a lower motor neuron lesion from T7-L2, depending on where the absence in noted, as a result of segmental innervation.

Which of the following options BEST describes a normal response to the cremasteric reflex test? 1. Skin tenses in the gluteal area. 2. Ipsilateral scrotum elevation 3. Contraction of the anal sphincter muscles 4. Umbilicus moves down and toward area being stroked.

Correct Answer: 3 1. A center coordinator for clinical education may be a physical therapist or physical therapist assistant however it may be another professional besides a physical therapy practitioner (p. 91). 2. Reviewing daily student documentation is not part of the role of the center coordinator for clinical education but is rather the role of the clinical instructor (pp. 91-92). 3. Serving as a liaison between clinical education sites and the academic institution is the primary role of the center coordinator for clinical education (p. 91). 4. Acting as a clinical instructor for physical therapy students is a role that is separate and distinct from the role of the center coordinator for clinical education, although a center coordinator for clinical education may also be a clinical instructor (p. 92).

Which of the following options BEST describes the role of the center coordinator for clinical education? 1. Practices as a physical therapist 2. Reviews daily student documentation 3. Serves as a liaison to the academic institution 4. Acts as a clinical instructor for physical therapy students

Correct Answer: 4 1. Supramalleolar orthoses would not provide enough support for an individual who has a history of S1 myelomeningocele, given the poor strength of the gastrocnemius. 2. Knee-ankle-foot orthoses provide too much support for an individual who has a history of S1 myelomeningocele, because the muscles of the knee are not weak. 3. Reciprocating gait orthoses provide too much support for an individual who has a history of S1 myelomeningocele, because the muscles of the knee and hip are not weak. 4. S1 myelomeningocele would cause weakness in the muscles of the posterior lower leg and tibia, without affecting muscle strength in the hips and knees. Solid ankle-foot orthoses would provide the appropriate support at the foot and ankle.

Which of the following orthoses would be MOST appropriate for a child who has a history of myelomeningocele at the S1 level and has Poor (2/5) gastrocnemius strength? 1. Supramalleolar 2. Knee-ankle-foot 3. Reciprocating gait 4. Solid ankle-foot

Correct Answer: 3 1. Current density is the amount of current per unit of contact area, which is inversely proportional to electrode contact area. Although current density may influence the strength of stimulation felt beneath the electrode, it is not the primary variable used to recruit peripheral axons. 2. Symmetry of waveform affects the way in which current amplitude varies over time. In symmetrical biphasic (balanced) waveforms, the total amount of current for one phase equals the absolute value of the total current in the second phase (versus unequal currents in each phase in asymmetrical biphasic/unbalanced waveforms). From a clinical perspective, differences in symmetry (balanced or unbalanced) results in noticeable differences in sensation of stimulus under surface electrodes and thus are manipulated for comfort. 3. The process by which increasing numbers of nerve fibers are activated by increasing the amplitude or the duration of the stimulus is called fiber recruitment. 4. Duty cycle refers to the ratio of on time to the sum of on time plus off time, expressed as a percentage. It characterizes the interrupted cycle of stimulation; this allows for relaxation between stimulation and assists with comfort during stimulation.

Which of the following parameters of electrical stimulation control the recruitment of peripheral axons during therapeutic electrical stimulation? 1. Current density 2. Symmetry of waveform 3. Duration of stimulus 4. Duty cycle

Correct Answer: 3 1. Patient A is experiencing respiratory acidosis and would not benefit from rebreathing carbon dioxide because this would make the condition worse (p. 215). 2. Patient B is experiencing metabolic alkalosis and would be treated with administration of potassium chloride (p. 212). 3. Patients who have respiratory alkalosis will have the following laboratory test values: pH greater than 7.45, PaCO2 less than 35 mm Hg, and HCO3- of 22-26 mEq/L (p. 1709). A patient who has respiratory alkalosis would benefit from rebreathing carbon dioxide (p. 215). 4. Patient D is experiencing metabolic acidosis and would be treated by correcting the electrolyte imbalance (p. 212).

Which of the following patients whose laboratory test values are shown in the table is MOST likely to benefit from rebreathing in a paper bag: Patient A: 7.2 pH, 68 paCO2, 22 HCO3 Patient B: 7.5 pH 35 paCO2 29 HCO3 Patient C: 7.53 pH 26 PaCO2 24 HCO3 Patient D: 7.33 pH 37 PaCO2 17 HCO3 1. Patient A 2. Patient B 3. Patient C 4. Patient D

Correct Answer: 2 1. Supine position is best to drain the upper lobes, anterior segments. 2. Leaning forward over a pillow is the best position to allow for drainage of the upper lobes, posterior segments. 3. Long sitting, leaning back is best used to drain the upper lobes, apical segments. 4. Prone position with the bed flat is best to drain the lower lobes, superior segments.

Which of the following positions is BEST for postural drainage of the posterior segments of the upper lobes? 1. Lying in a supine position with the bed flat 2. Sitting in a chair, leaning forward over a pillow 3. Long-sitting position, leaning back 4. Lying in a prone position with the bed flat

Correct Answer: 2 1. The correct way to expose the supraspinatus tendon is with extension and medial (internal) rotation. Flexion and lateral (external) rotation does not give access to the tendon. 2. Extension and medial (internal) rotation of the shoulder puts the supraspinatus tendon in the most accessible position. 3. The correct way to expose the supraspinatus tendon is with extension and medial (internal) rotation. Abduction and lateral (external) rotation does not give access to the tendon. 4. The correct way to expose the supraspinatus tendon is with extension and medial (internal) rotation. Flexion and medial (internal) rotation does not give access to the tendon.

Which of the following positions of the humerus is BEST for application of an ultrasound treatment to the supraspinatus tendon insertion? 1. Flexion and lateral (external) rotation 2. Extension and medial (internal) rotation 3. Abduction and lateral (external) rotation 4. Flexion and medial (internal) rotation

Correct Answer: 1 1. A patient who has lower crossed syndrome will have tight erector spinae and iliopsoas muscles and weak abdominal and gluteus maximus muscles. This results in an anterior pelvic tilt, an increased lumbar lordosis, and a slight flexion of the hip. 2. A patient who has lower crossed syndrome will have tight erector spinae and iliopsoas muscles and weak abdominal and gluteus maximus muscles. This results in an anterior pelvic tilt, an increased lumbar lordosis, and a slight flexion of the hip, not slight extension of the hip. 3. A patient who has lower crossed syndrome will have tight erector spinae and iliopsoas muscles and weak abdominal and gluteus maximus muscles. This results in an anterior, not posterior pelvic tilt, an increased lumbar lordosis, and slight flexion of the hip. 4. A patient who has lower crossed syndrome will have tight erector spinae and iliopsoas muscles and weak abdominal and gluteus maximus muscles. This results in an anterior, not posterior pelvic tilt, an increased lumbar lordosis, and a slight flexion, not extension, of the hip.

Which of the following postural characteristics are MOST likely to be seen in a patient who has lower crossed syndrome? 1. Anterior pelvic tilt and slight hip flexion 2. Anterior pelvic tilt and slight hip extension 3. Posterior pelvic tilt and slight hip flexion 4. Posterior pelvic tilt and slight hip extension

Correct Answer: 1 1. The photograph depicts the median nerve distribution, which is commonly disrupted in carpal tunnel syndrome. Tapping the anterior wrist describes the Tinel sign at the carpal tunnel. 2. The photograph depicts median nerve distribution, which is commonly disrupted in carpal tunnel syndrome. Applying direct pressure to the base of the thumb (1st digit) describes the digital nerve compression test for bowler's thumb, not carpal tunnel syndrome. 3. The photograph depicts the median nerve distribution, which is commonly disrupted in carpal tunnel syndrome. Palpation of the pronator teres checks for pronator syndrome and is not as specific as testing for the Tinel sign at the carpal tunnel. 4. The photograph depicts the median nerve distribution, which is commonly disrupted in carpal tunnel syndrome. Compressing the cervical spine with ipsilateral rotation and extension describes the Spurling test for cervical radiculopathy and is not as specific as the Tinel sign at the carpal tunnel.

Which of the following provocative maneuvers is MOST likely to elicit symptoms in the area highlighted in the photograph? 1. Tapping the anterior wrist moving from proximal to distal 2. Applying direct pressure to the base of the thumb (1st digit) 3. Palpating along the pronator teres 4. Compressing the cervical spine with ipsilateral rotation and extension

Correct Answer: 1 1. The test shown in the photograph would be necessary if a patient reports a pulsing or pounding sensation in the abdomen during increased activity or when lying in supine position. A positive finding would be a palpable pulsating mass and would indicate an abdominal aneurysm. (p. 201) 2. Reproduction of back or flank pain is a positive finding for possible renal involvement when percussion is done posteriorly over the costovertebral angle of the back (p. 201). 3. Distention on the costal margin is descriptive of an abnormal finding for the spleen, which is palpated typically below the left costal margin. A positive finding occurs in conditions such as mononucleosis and trauma. This warrants immediate medical attention. (p. 200) 4. The presence of costovertebral tenderness during the Murphy percussion test is a positive sign of renal involvement; however, the test is performed on the back, not the abdomen (p. 201).

Which of the following responses is a positive finding for the test shown in the photograph? 1. Palpation of a pulsating mass 2. Presence of back or flank pain 3. Distention on the costal margin 4. Costovertebral tenderness

Correct Answer: 1 1. The presence of clonus is indicative of an upper motor neuron lesion (p. 377). 2. The presence of paresis is indicative of a lower motor neuron lesion (p. 378). 3. The presence of areflexia is indicative of a lower motor neuron lesion (p. 378). 4. The presence of hypotonia is indicative of a lower motor neuron lesion (p. 378).

Which of the following signs is MOST characteristic of an upper motor neuron lesion? 1. Clonus 2. Paresis 3. Areflexia 4. Hypotonia

Correct Answer: 2 1. An decrease, not increase, in hair growth in the lower extremities is expected in patients who have chronic lower limb ischemia. 2. With chronic ischemia due to arterial insufficiency, a cardinal sign is that the skin in the lower extremities becomes thin, scaly or shiny, and transparent due to inadequate blood flow. 3. The lower extremity nail beds are typically thick and brittle, not thin and supple, in patients who have chronic lower limb ischemia. 4. With ischemia, there is decreased blood flow, so the skin feels cooler, not warmer.

Which of the following signs would be MOST indicative of a patient who is experiencing chronic lower limb ischemia? 1. There is an increase in the hair growth in the lower extremities. 2. The skin of the lower extremities has become transparent and appears dehydrated. 3. The nail beds of the toes have become thin and supple in texture and strength. 4. There is an increase in skin temperature in the lower extremity.

Correct Answer: 4 1. Stopping the process that leads to the development of disease describes primary prevention. 2. Providing education to give people greater control over their own health describes health promotion and wellness. 3. Limiting the degree of disability and improving function in patients who have chronic disease describes tertiary prevention. 4. Secondary prevention involves early detection of disease and health conditions through regular screening.

Which of the following statements is the BEST description of a physical therapist's role in secondary prevention of disease? 1. Stopping the process that leads to the development of disease. 2. Providing education to give people greater control over their own health. 3. Limiting the degree of disability and improving function in patients who have chronic disease. 4. Performing early detection of disease and health conditions through regular screening.

Correct Answer: 3 1. Principles of documentation require that comments be clear, objective, and measurable. General statements that are too vague should be avoided. 2. The statement includes non-standardized abbreviations, such as "recip." and "min," which may not be facility-approved or widely recognized. 3. Principles of documentation require the use of objective statements that are clearly measurable. This statement also avoids non-standardized abbreviations and fully spells out terms. 4. Principles of documentation require that comments be clear, objective, and measurable. General statements that are too vague should be avoided.

Which of the following statements is the MOST appropriate example of patient care documentation? 1. Patient ambulated up and down stairs with a reciprocal stepping pattern without difficulty. 2. Patient ambulated up and down 6 steps using a right handrail and recip. stepping pattern with min assist. 3. Patient ambulated up and down 6 steps using a right handrail and a reciprocal stepping pattern with minimal assistance. 4. Patient ambulated up and down stairs using a right handrail and reciprocal stepping pattern with minimal assist.

Correct Answer: 1 1. The levator ani muscles consist of the pubococcygeus, iliococcygeus, and puborectalis, which actively compress the urethra, vagina, and rectum, thus maintaining continence. 2. The levator ani muscles consist of the pubococcygeus, iliococcygeus, and puborectalis, which actively compress the urethra, vagina, and rectum, thus maintaining continence. The obturator internus is a lateral (external) hip rotator. 3. The levator ani muscles consist of the pubococcygeus, iliococcygeus, and puborectalis, which actively compress the urethra, vagina, and rectum, thus maintaining continence. The pubovesical ligament provides passive support. 4. The levator ani muscles consist of the pubococcygeus, iliococcygeus, and puborectalis, which actively compress the urethra, vagina, and rectum, thus maintaining continence. The anococcygeus ligament provides passive support.

Which of the following structures provide active compression of the urethra? 1. Pubococcygeus, iliococcygeus, and puborectalis 2. Pubococcygeus, obturator internus, and puborectalis 3. Iliococcygeus, puborectalis, and pubovesical ligament 4. Pubococcygeus, iliococcygeus, and anococcygeus ligament

Correct Answer: 4 1. When measuring forearm supination, lateral (external) rotation of the shoulder or adduction past 0° should be avoided (p. 92). Shoulder medial (internal) rotation and abduction should be avoided when measuring forearm pronation (p. 94). 2. When measuring forearm supination, lateral (external) rotation of the shoulder or adduction past 0° should be avoided (p. 92), not shoulder medial (internal) rotation. 3. When measuring forearm supination, lateral (external) rotation of the shoulder or adduction past 0° should be avoided (p. 92), not shoulder abduction. 4. When measuring forearm supination, lateral (external) rotation of the shoulder or adduction past 0° should be avoided (p. 92

Which of the following substitution patterns should be prevented when measuring active forearm supination? 1. Shoulder medial (internal) rotation and shoulder abduction 2. Shoulder medial (internal) rotation and shoulder adduction past 0° 3. Shoulder lateral (external) rotation and shoulder abduction 4. Shoulder lateral (external) rotation and shoulder adduction past 0°

Correct Answer: 1 1. Water displacement has been regarded as the most sensitive and accurate standard for volume measurement. 2. Circumference measurements taken at various points of a body part are used most frequently to quantify lymphedema, but several problems exist, including limitations for acceptable differences between repeated circumferential measurement of the normal adult and control of intra- and interrater reliability. Circumferential measurement, although used clinically, is not considered a reliable/valid (gold) standard. 3. Single frequency bioelectrical impedance has become more frequently used in the clinical setting to measure limb fluid, but it is not considered the reliable/valid (gold) standard. 4. Optoelectronic volumetry calculates limb volume by using infrared light and has been conceptualized as a continuous variable, supporting a more robust test of the severity of lymphedema, but it is not considered a reliable/valid (gold) standard.

Which of the following tests is MOST accurate for assessing volume reduction in a patient who has lymphedema? 1. Water displacement 2. Limb circumference 3. Bioelectrical impedance 4. Optoelectronic volumetry

Correct Answer: 4 1. Rate pressure product reflects myocardial oxygen demand, not skin integrity (DeTurk, p. 56). 2. Pulse pressure (difference between systolic and diastolic pressures) reflects the pressure responsible for perfusion of organs and tissue but not skin integrity (DeTurk, p. 312). 3. Capillary refilling time reflects adequacy of arterial skin perfusion but not necessarily risk of developing a skin ulceration (Baranoski, p. 381). 4. Because peripheral neuropathy is a common secondary complication of diabetes, it is imperative for a physical therapist to assess a patient's ability to detect light touch and presence of protective sensation to determine risk of skin ulceration (Baranoski, pp. 381-382).

Which of the following tests or measurements is BEST to assess risk for skin ulceration in a patient who has diabetes? 1. Rate pressure product 2. Pulse pressure 3. Capillary refill 4. Light touch

Correct Answer: 1 1. Symptoms consistent with angina are transient/temporary pain that radiates down the left arm into the ulnar border of the hand with the ring and little fingers (4th and 5th digits) affected. It is generally increased with exertion and decreased with rest. (pp. 572-573) 2. Symptoms of left-sided heart failure include progressive dyspnea and a productive spasmodic cough due to pulmonary edema. There is also cerebral hypoxia, which would cause confusion and anxiety. (pp. 593-594) 3. Symptoms of right-sided heart failure include edema of the lower extremity, jugular venous distention, and abdominal pain and distention (p. 593). 4. Symptoms associated with myocardial infarction are a sensation of pressure, dizziness, shortness of breath, and sweating that is persistent, lasting more than 30 minutes. Some of the symptoms described in the stem may be present, but the transient nature of the symptoms is not consistent with a myocardial infarction. (p. 586)

While exercising on a stationary bicycle, a patient experiences sudden pain behind the sternum. The pain radiates down the left upper extremity into the ring and little fingers (4th and 5th digits). The pain subsides with rest but returns several times throughout the session. Which of the following conditions is the MOST likely source of these symptoms? 1. Angina 2. Left-sided heart failure 3. Right-sided heart failure 4. Myocardial infarction

Correct Answer: 3 1. The pectoralis major does not have attachments to the lumbar spine. 2. The pectoralis minor does not have attachments to the lumbar spine. 3. The latissimus dorsi is the only muscle listed that has an attachment to the lumbar spine (through the thoracolumbar fascia). Therefore, this is the only muscle listed that could be affected by the position of the lumbar spine. If the latissimus dorsi is short, the lumbar spine is not allowed to extend and shoulder flexion/elevation will be limited. 4. The serratus anterior does not have attachments to the lumbar spine.

While performing an assessment of a patient's active shoulder flexion in supine, a physical therapist notices that the patient cannot complete the motion unless the lumbar spine is allowed to extend. Shortness in which of the following muscles is MOST likely the problem? 1. Pectoralis major 2. Pectoralis minor 3. Latissimus dorsi 4. Serratus anterior

Correct Answer: 4 1. The lift-off test is used to test for the presence of a rotator cuff tear (involving the subscapularis) and would not be used to detect subacromial impingement (p. 523). 2. The Hornblower sign is used to test for the presence of a rotator cuff tear (involving the teres minor) and would not be used to detect subacromial impingement (p. 522). 3. The apprehension sign is a test for anterior glenohumeral instability and not a test for subacromial impingement (p. 520). 4. The procedure described in the stem is the Neer test, which tests for the presence of subacromial impingement. The patient's response would be indicative of a subacromial impingement of the supraspinatus or long head of the biceps tendon. The Hawkins-Kennedy test is also a test for the presence of subacromial impingement. (p. 519)

With a patient in a sitting position, a physical therapist stabilizes the patient's scapula while passively moving the shoulder into flexion and medial (internal) rotation. This movement reproduces the patient's shoulder pain. Which of the following assessments is MOST likely to confirm the suspected diagnosis? 1. Lift-off test 2. Hornblower sign 3. Apprehension sign 4. Hawkins-Kennedy test


Ensembles d'études connexes

I NEED 120 QUESTION CORRECT TO PASS TEST WITH A 70

View Set

GCSE physics Topic 3 - Conservation of energy

View Set

Blood and Immune FINAL STUDY AID

View Set

Courts: Defense Attorneys Quiz 7

View Set

Geography: Volcanoes and Earthquakes.

View Set

Chapter 5 - Philosophy - STOP 337

View Set

Slope Intercept Form with Graph and Standard Form

View Set